143
Vol. 15 2011 年度西澤ゼミ学生論文集 同志社大学法学部・政治学科 2012 3 II

2011 年度西澤ゼミ学生論文集 - WWW1 Server Indexynishiza/ynishiza2014/...iii もくじ はしがき 西澤由隆 i 2 年生論文 1. 選挙運動と投票参加 岩崎朝恵

  • Upload
    vuongtu

  • View
    213

  • Download
    0

Embed Size (px)

Citation preview

Vol. 15 2011 年度西澤ゼミ学生論文集

同志社大学法学部・政治学科

2012 年 3 月

多 岐 亡 羊 II

i

はしがき

中国の学者・楊子(ようし)が、分かれ道が多くて、逃げた

羊を見つけることができなかったことを深く悲しんだ。「分か

れ道が多いために羊を逃してしまったのと同じように、学問の

道もあまりに多方面にわたっているために、真理を見失ってし

まうことを悲しんだのだ」と、後にその弟子の一人・心都子(し

んとし)が説明したそうだ。

現代日本が抱える政治的な問題や日本あるいは外国の有権

者の投票行動・価値観について、多くの立場の研究に触れ、ま

た、興味を持った具体的な疑問点について、各自が独自な視点

で分析を行ったわけだが、それには1年はあまりにも短かった。

おかげで、政治のメカニズムが理解できたというより、むしろ

よけいに分からなくなったというのが正直なところだろう。ま

さしく、「多岐亡羊」である。

さて、2011 年度は、例年とは異なる意味において、「忘れる

ことのない 1 年」となった。言うまでもなく、3 月の東日本大

震災とそれに続く原発事故のためである。3 月 11 日以降、測り

知れないプレッシャーのもとでだれもが日々を暮らすことに

なった。そして、10 ヵ月以上もたつ今でも、原発事故による放

射能汚染への脅威は、収まるどころかむしろ拡大しているよう

に感じる。ゼミの諸君は、一人ひとり、そのことについての心

配は顔にこそ出さなかったが、この「戦い」が長期にわたるこ

とを皆が承知で、しかも、それを担う中心的な世代が自分たち

であることを自覚しているだけに、「プレッシャー」は大きか

ったに違いない。大げさなようだが、「非常事態」でのゼミ活

動だった。そんな中での諸君の努力・協力の結晶がここに収め

られた論文である。だから、これがまとまったことに、心から

敬意を表したい。本当に、君たちは頑張った。

例年のことだが、ゼミの諸君は仲がいい。(私の「厳しい=愛

情たっぷりの」トレーニングが奏効しているのだと自分には言

い聞かせているのだが、)すばらしい連帯感である。その絆の

強さを感じるエビソードは数え切れない。たとえば、この論集

の最終稿を 3 年生が中心となり校正をしたが、その丁寧さには、

2 年生の後輩達も驚いたことだろう。本当の意味での「共同研

究」が成立している。きっと、この絆は、社会に出てからの一

生の宝になると確信している。そして、こんなゼミで一緒でき

たことは、私の誇りでもある。

ii

ところで、大学で学ぶことの究極の目的は、「真理とは何か

(あるいは人間とはどういう生き物なのか)」という問いに対

する答えを見いだすことである。「3.11」は、まさしく、その問

を世界中の人に突きつけたと言ってもよい。それにしても、大

学の 4 年は短すぎる。今年度も、「真理を問う」ことのおもし

ろさを、君たちが尐しは体験できたと私は信じているが、4 年

生の諸君は、卒業後もこの問いに対して自問を続けることにな

るだろう。また、在学生は、残る時間を大切にして、今年以上

に勉学に励んでほしい。来年度のさらなる飛躍を期待したい。

西澤由隆 2012 年 1 月 21 日

光塩館の研究室にて

iii

もくじ

はしがき 西澤由隆 i

2 年生論文

1. 選挙運動と投票参加 岩崎朝恵 1 -なぜ、候補者の経歴放送の視聴が

投票参加を促すのか-

2. 候補者志向と後援会 浦岡教之 7 -なぜ小選挙区制導入後も、候補者志向が

かなりの程度残っているのか-

3. 投票義務感をもたらすものはなにか 浦田祐香 12

4. 政党支持 塩谷侑己 17 -なぜ、無党派層は生まれるのか?-

5. 政治的有効性感覚と投票 林香織 23

6. 制度認知と関係する要因とは 水田紗希子 29 -学歴・政治関心との相関関係-

7. なぜ世代間で投票率に差が生じるのか 山内逸平 35 -尐子高齢化に着目して-

8. 生活満足度が政治満足度に

及ぼす影響について 渡邊智弘 40 -所得は生活満足度に影響するか-

3 年生論文

9. フィリピンの市民が 池田洋 50

モンストレーションに参加するのはなぜか

10. 2005 年衆議院選挙の投票率上昇は

何によってもたらされたのか 岡本怜子 59 -誰がマスメディアの影響を受けたのか-

11. 争点投票を可能とする要因は何か 岡山義信 68 -ソーシャル・キャピタルの可能性-

12. 「無風選挙」と有権者の投票行動 日裏瑠奈 85

13. なぜ職業が投票政党を規定するのか山岸未加 95

4 年生論文

14. 有権者のメディア効果意識 西山千尋 104 -メディアの影響を感じているのは誰か-

15. チェコにおける政治関心 林香織 118

16. 選挙制度改革によって候補者投票と 廣田奈央 127

政党投票の効果はどのように変化したのか

2 年生論文

1

1 選挙運動と投票参加 なぜ、候補者の経歴放送の視聴が投票参加を

促すのか

岩崎 朝恵

1. はじめに 選挙における投票率の低下が叫ばれる近年、有権者の投票参加

意欲をあげる効果的な方法はないのか。そう考えたときに、政党

や候補者が行う選挙運動に触れることが、一番政治を身近に感じ

られて効果的なのではないかと私は考えた。そこで本稿では、選

挙運動、特に候補者の経歴放送に焦点を当てて、有権者の投票行

動について分析していく。

2. 研究意義 選挙運動の効果について、三宅(1989)の分析を紹介する。三宅は、

Lazarsfeld らの選挙の古典的研究を用いて、選挙運動の効果を「活

性化(結晶化)効果」・「補強効果」・「改変効果」の 3 つに分類してい

る。しかしこれら 3 つの効果は投票の方向先を決定するさいに作

用するものであり、有権者が投票に参加するかどうかに作用する

ものではない。そこで三宅はさらに動員効果をあげている。動員

効果は、「活性化効果に似ているが、活性化効果はもともと選挙へ

の関心あるいは党派心を先有傾向としてもたない人たちをそのま

まの状態で投票へと動員する効果である」(三宅 1989,209)。

本稿では、この動員効果のように、選挙運動との接触は、投票

方向の決定だけでなく投票参加への効果もあるのかについて、選

挙運動の中から候補者の経歴放送をとりあげて分析していく。候

補者の経歴放送をとりあげた理由は、テレビが有権者にとってマ

スメディアのなかでも最も普及しており、さらに経歴放送は候補

者が主体的に行う点で、一般にマイナスな印象が強いテレビの報

道よりも、有権者が政治を身近に感じられ効果が期待できるので

はないかと考えたからである。

候補者の経歴放送の視聴と投票参加との関係については、有力

な先行研究は見つけられなかった。しかし表 1 より、候補者の経

歴放送の視聴と投票参加の間には、正の相関関係があることが分

かる。

2

表 1 候補者の経歴放送 と 投票参加 (%)

投票参加

合計(%) 投票した 投票しなかった

候補者の経歴放

観た 87.1 12.9 100

観てない 80.8 19.2 100

出所:第 44 回衆議院議員総選挙についての意識調査1

しかし上で述べたように、選挙運動との接触は投票方向の決定

へとつながるのが一般的であり、一概に投票参加へ矢印を伸ばす

ことはしがたい。このことから、候補者の経歴放送を観ることで、

投票参加への意欲を上げる何か別の潜在的要因が働き、投票行動

につながるのではないかと私は考えた。

この潜在的要因として、蒲島(1986)の政治参加の研究を参考に、

政治的態度の向上を考えた。蒲島によると、政治的関心や政治シ

ステムへの信頼度、政治シニシズム、政策意見、政党支持、候補

者情報、候補者への近接度(ここでは候補者に対する知識や、有権

者との関係)といった「政治的態度と政治参加には高い相関関係が

ある」(蒲島 1986,200)という。ここでいう政治参加には、投票・

選挙運動・地域・接触活動の 3 次元がある。本稿ではこれら政治

的態度のうち政治システムへの信頼度と政治的関心の 2 つと、政

治参加のうちの投票との関係を見る。

3. 命題と仮説

3-1. 命題

以上より、命題を「候補者の経歴放送を観ることは、政治的態

度を向上させ、投票参加に効果を及ぼす」とする。

3-2. 仮説

本稿では、この命題を実証するために、「政治への信頼度」と「今

回の選挙への関心」の 2 つの政治的態度を命題の変数として用い、

それぞれについて検証する。この 2 つの政治的態度の設定は、上

で述べた蒲島による政治参加の研究を参考に、政治システムへの

信頼を「政治への信頼度」へ、政治的関心を「選挙への関心」へ

と置き換えて考えることとした。

(1)「候補者の経歴放送を観ると、政治への信頼度が高まり投票

参加する。」

これを『政治信頼度要因説』と呼ぶことにする。

(2)「候補者の経歴放送を観ると、選挙への関心が高まり投票参

加する。」

これを『選挙関心要因説』と呼ぶことにする。

3

図 1

相関関係はあるが、直接の関係性があるのか?

図 2 自仮説のモデル図

〈政治的態度の向上〉

・政治への信頼

・選挙への関心

4. 分析 本稿では、明るい選挙推進委員会「第 44 回衆議院議員総選挙に

ついての意識調査」を基に、クロス表を用いて分析を行う。

4-1. 分析の手順

(1)

①候補者の経歴放送を観ると、政治への信頼度が高まる、という

ことを実証するため、「経歴放送の視聴の有無」と「政治への信

頼度」をクロス表にする。(表 1-1)

②政治への信頼度が高まると、投票参加する、ということを実証

するため、「政治への信頼度」と「投票参加」をクロス表にする。

(表 1-2)

(2)

①候補者の経歴放送を観ると、選挙への関心が高まる、というこ

とを実証するため、「経歴放送の視聴の有無」と「選挙への関心」

をクロス表にする。(2-1)

②選挙への関心が高まると、投票参加する、ということを実証す

るため、「選挙への関心」と「投票参加」をクロス表にする。(表

2-2)

経歴放送の視聴 投票参加

経歴放送の視聴 投票参加

潜在的な要因

4

4-2. 分析結果

表 1-1 候補者の経歴放送 と 政治信頼度(%)

政治信頼度

合計(%) 信頼している 信頼していない

候補者の経歴放

観た

観ていない

40.7

34.0

59.3

66.0

100

100

カイ二乗値=6.954 危険率=.008

表1-2 政治信頼度 と 投票参加(%)

投票参加

合計(%) 投票した 投票しなかった

政治信頼度 信頼している

信頼していない

88.6

81.9

11.4

18.1

100

100

カイ二乗値=12.107 危険率=.001

表2-1 候補者の経歴放送 と 選挙関心(%)

選挙関心

合計(%) 関心あり 関心なし

候補者の経歴放送 観た

観ていな

90.2

82.1

9.8

17.9

100

100

カイ二乗値=19.675 危険率=.000

表2-2 選挙関心 と 投票参加 (%)

投票参加

合計(%) 投票した 投票しなかった

選挙関心 関心あり

関心なし

90.2 9.8 100

43.5 56.5 100

カイ二乗値=317.704 危険率=.000

(1)

表 1-1 より、経歴放送を観た場合と観ていない場合とを比べると、

政治を「信頼している」は観た 40.7%、観ていない 34.0%で経歴

放送を観た人の方が観ていない人より政治信頼度は高い。また「信

頼していない」は観た 59.3%、観てない 66.0%で観ていない人の

方が観た人より政治信頼度が低い。よって経歴放送を観た人の方

が観ていない人より政治信頼度は高く、観ていない人の方が観た

人より政治信頼度が低いことが分かり、これより両者は相関関係

5

にある。

表 1-2 より、政治を信頼している人としていない人を比べると、

「投票した」は信頼している 88.6%、していない 81.9%で政治を

信頼している人の方がしていない人よりもわずかではあるが投票

に参加した人が多い。また「投票しなかった」は信頼している 11.4%、

していない 18.1%で政治を信頼していないほど投票に参加する人

が尐ない。よって政治を信頼している人の方がしていない人より

も投票に参加し、信頼していないと投票に参加しないことが分か

り、これより両者は相関関係にある。

以上より、「候補者の経歴放送を観ると、政治への信頼度が高ま

り投票参加する。」という政治信頼度要因説は実証された。

(2)

表 2-1 より、経歴放送を観た場合と観ていない場合を比べると、

今回の選挙に「関心あり」は観た 90.2%、観てない 82.1%で経歴

放送を観た人の方が観ていない人より選挙関心が高い。特に経歴

放送を観た人はほとんどが選挙への関心を持っていることが、高

い数値から分かる。また「関心なし」は観た 9%、観てない 17.9%

で観ていない人の方が観た人より選挙関心が低い。よって経歴放

送を観た人の方が観ていない人より選挙への関心は高く、観てい

ない人の方が観た人より選挙への関心が低いことが分かり、これ

より両者は相関関係にある。

表 2-2 より、選挙に関心がある人とない人とを比べると、「投票

した」は関心あり 90.2%、関心なし 43.5%で選挙に関心のある人

の方がない人よりも明らかに投票に参加した人が多い。また「投

票しなかった」は関心あり 9.8%、関心なし 43.5%で選挙に関心が

ないと全体の半数以上の人が投票に参加していない。よって選挙

に関心のある人の方がない人よりも投票に参加した人が多く、関

心がないと投票に参加しないことが分かり、これより両者は相関

関係にある。

以上より、「候補者の経歴放送を観ると、選挙への関心が高まり

投票参加する。」という選挙関心要因説は実証された。

5.まとめ

分析の結果から、(1)「候補者の経歴放送を観ると、政治への信

頼度が高まり投票参加する。」という政治信頼度要因説も、(2)「候

補者の経歴放送を観ると、選挙への関心が高まり投票参加する。」

という選挙関心要因説も、数字のばらつきが尐ない部分もあった

が、一定の支持を得ることができた。すなわち、投票参加には政

治への信頼度と選挙への関心度という政治的態度が影響すること

が明らかになり、この 2 つの要因に経歴放送の視聴が関係してい

ることも判明した。だが、この分析には課題が存在する。

一つは (1)も(2)も、「経歴放送を観る→信頼度が上がる or 関心が

高まる」という結果に対する原因の時間的先行をこちらで指定し

たということである。両者に相関はあったものの、もしかしたら

「信頼度が高い or 関心が高い→経歴放送を観る」という時間軸で、

まったく逆の相関関係にあるかもしれない。今回の分析ではそこ

6

まで考察することはできなかったが、特に選挙への関心の場合、

関心があるから積極的に選挙運動に触れ、投票に参加するという

流れはとても自然である。

もう一つは経歴放送の視聴・政治的態度の向上・投票参加の 3

変数の関係の考察が不十分になってしまったことである。この 3

つの変数を用いて多重クロス表分析を行ったところ、2 仮説とも一

定の相関を得ることができた。本稿では、経歴放送の視聴と投票

参加は間接的な関係にあると考えたが、さらに深く分析を行えば

もっと別の要因が見いだせたかもしれない。3 つの関係は、今後の

課題として考えていきたい。

(1)なお、これらのデータについては、レヴァイアサン・データ

バンクより同志社大学・法学部が購入したものを、教材とし

て西澤教授より提供を受けた。いずれも、同志社大学・法学

部の西澤由隆先生のご指導と便宜により利用ができた。それ

ぞれのデータを公開・寄託され、利用できるようにしてくだ

さった西澤教授に感謝いたします。

<補遺>

分析に使用した質問は以下の通りである。

<投票参加>

Q5,あなたは今回の衆議院選挙では、投票しましたか、しませ

んでしたか。(コード化済み)

・投票した

・投票しなかった

<政治信頼>

Q24,あなたは、日本の政府が「正しいことをしてくれる」と、

どれくらい信頼していますか。(コード化済み)

・信頼している

・信頼していない

Q2, 今回の選挙について、あなた自身は、どれくらい関心をも

ちましたか。(コード化済み)

・関心あり

・関心なし

Q7,参考になったかどうかは別として、今回の選挙で、あなたが

直接見たり、聞いたりしたものが、この中にありましたら、

全部おっしゃってください。(コード済み)

・候補者の経歴放送を観た

・候補者の経歴放送を観ていない

<参考文献>

・三宅一郎 1989. 『投票行動』 東京大学出版社

・綿貫譲治、三宅一郎、猪口孝、蒲島郁夫 1986. 『日本人の選

挙行動』 東京大学出版会.

7

2 候補者志向と後援会 なぜ小選挙区制導入後も、候補者志向がかな

りの程度残っているのか 浦岡 教之

1. はじめに 1994 年の選挙制度改革により、従来の中選挙区制にかわって小

選挙区制が導入された。この改革の目的は、候補者志向による利

益誘導政治をなくし、政党志向による政党政治を実現させること

であった。しかし、本当にこの目的は達成されたのだろうか。

表 1 は、1994~2005 年の衆議院選挙において、有権者が政党か

候補者どちらを重視したのかを示したものである。表1を見ると、

選挙制度改革以後、確かに候補者志向の有権者は減尐し、逆に政

党志向の有権者は増加している。だが、2005 年の段階においても

まだ 41.0%という比較的高い割合で候補者志向が残存している。

この割合を見る限り、選挙制度改革の目的が達成されたとは言い

難い。

では、なぜ選挙制度改革後も候補者志向の投票がかなりの程度

残っているのだろうか。以下、その要因について検証していく。

表 1 衆議院選挙において、政党か候補者どちらを重視したか (%)

1993 1996 2000 2003 2005

政党 45.2 49.7 51.8 56.3 59.0

候補者 54.8 50.3 48.2 43.7 41.0

出所:明るい選挙推進協議会データ 衆院選 1993~2005

2. 仮説 本稿では、有権者の候補者志向を裏付ける要因として、後援会

への参加を取り上げる。「後援会に参加している有権者は、その候

補者を支持しているので、候補者志向の投票をする」という仮説

である。

3. 統制変数 次に、「後援会への参加」という変数だけが候補者志向に影響を

与えているかどうかを確認しなければならない。なぜなら、他の

変数が候補者志向に影響を及ぼしている可能性が否めないからで

ある。

ここでは、その確認のために「支持政党の有無」という統制変

数を投入する。有権者は支持政党があれば政党志向の投票をし、

支持政党がなければ候補者志向の投票をすると考えたからである。

統制変数投入後も、二変数の相関関係が残っているならば、仮説

は支持されたということができる。

8

自仮説のモデル図

支持政党の有無

4. 検証方法 本稿で使用するデータは、「明るい選挙推進協議会 衆議院選挙

2005」によるものである。以下、このデータを用いて仮説を検証

していく。1

まず、「投票時に重視したもの」と「後援会への参加」をクロス

表にする。次に、そのクロス表に「支持政党の有無」という統制

変数を投入する。なお、具体的な作業定義については<補遺>に

掲載しているので確認して頂きたい。

5. 検証

5-1. 仮説の検証

まず、候補者志向と後援会への参加に相関関係が見られるか確

認する。「投票時に重視したもの」と「後援会への参加」をクロス

表にしたものが、以下の表 2 である。

表 2 投票時に重視したものと後援会(%)

投票時に重視したも

政党 候補者 計 N

後援会 参加 54.1 45.9 100 133

不参加 59.8 40.2 100 1008

カイ二乗値 1.572 危険率.210

表 2 を見ると、後援会に参加している人のうち過半数の 54.1%

が政党志向の投票をしている。それに対して、候補者志向の投票

をしているのは 45.9%にすぎない。後援会に参加している人が候

補者志向の投票をしているとは必ずしも言えない。また、危険率

も.210 であり有意な結果ではない。つまり、後援会に参加してい

る有権者は候補者志向の投票をするという仮説は支持されなかっ

た。

5-2. 統制変数の投入

5-1 では自仮説は支持されなかった。しかし、「後援会への参加」

と候補者志向の相関関係が偶然みられなかった場合も考えられる。

そこで、先ほどまでのクロス表に統制変数を投入することによっ

て、再度検証を行いたい。以下、5-1 のクロス表に、「支持政党の

有無」という統制変数を投入し、検証をおこなっていく。

候補者志向 後援会への参加

9

表 3 投票に重視したものと後援会と支持政党のクロス表 (%)

①支持政党あり

投票時に重視したもの

政党 候補者 計 N

後援会 参加 55.4 44.6 100 112

不参加 62.3 37.7 100 677

②支持政党なし

投票時に重視したもの

政党 候補者 計 N

後援会 参加 55.6 44.4 100 18

不参加 54.6 45.4 100 293

① カイ二乗値 1.973 危険率.160

② カイ二乗値 0.006 危険率.937

まず、①支持政党がある有権者について確認する。①では、後

援会に参加している人のうち 44.6%しか候補者志向の投票をして

おらず、残りの 55.4%が政党志向の投票をしている。また、危険

率も.160 と高く、有意な結果を得ることができなかった。

次に、②支持政党が無い有権者について確認する。②の場合も

同様で、後援会に参加している人のうち 44.4%しか候補者志向の

投票をしていなかった。それに対して、過半数の 55.6%が政党志

向の投票をしていることがわかった。加えて、危険率も.937 と非

常に高いので、有意な結果であるとは言えない。

以上のように、統制変数投入後も、「後援会への参加」と候補者

志向に相関関係は見られなかった。

6. まとめ 本稿では、小選挙区制が導入された後もなぜ候補者志向がかな

りの程度残っているのかという疑問に対し、「後援会に参加してい

る有権者は、その候補者を支持しているので、候補者志向の投票

をする」という仮説をたてた。しかし、この仮説は支持されなか

った。その理由は主に 2 つある。

1 つ目は、今回の分析で用いたデータにおいて、後援会に参加し

ている有権者の数が尐なかった点である。表 2 では、後援会に参

加している有権者は 133 人しかいない。また、表 3 の①は 112 人、

②は 18人とどちらもわずかの有権者しか後援会に参加していない

ことがわかる。母集団が尐なければ、正確なデータを得ることは

できないだろう。以上が 1 つ目の理由である。

2 つ目は、有権者が後援会に参加する理由をつかめていなかった

点である。本稿では、後援会に参加している有権者はその後援会

を主催している候補者に投票するという前提で仮説を検証してき

た。しかし、本当にそうだろうか。

確かに、候補者自体を支持しているため、その後援会に参加す

る有権者もいる。だが、中には小選挙区制という制度の都合上、

やむなく後援会に参加する人もいるはずである。

小選挙区制では 1 つの選挙区から 1 人の候補者しか当選できな

10

い。そのため、候補者自体を支持しているわけではないが、自分

の支持政党がその候補者しか公認していないため、仕方なくその

後援会に参加する人がいても不思議ではない。つまり、後援会に

参加している有権者が必ずしも候補者志向の投票をしていると明

言することはできない。以上が 2 つ目の理由である。

今回の分析は小選挙区制のもと検証を行ってきた。しかし、中

選挙区制のもと検証を行ったらどうなるだろうか。中選挙区制で

は 1 つの選挙区から 3~5 人当選できるので、同政党から複数の候

補者が出馬する。それゆえ、自分の支持している政党から 1 人の

候補者しか出馬していないため、仕方なくその後援会に参加する

という有権者は尐なくなるだろう。中選挙区制のもとでは、後援

会に参加している有権者は候補者志向の投票をしているかもしれ

ない。中選挙区制における自仮説の検証が今後の課題である。

(1)なお、これらのデータについては、レヴァイアサン・データバ

ンクより同志社大学・法学部が購入したものを、教材として西

澤教授より提供を受けた。いずれも、同志社大学・法学部の西

澤由隆教授のご指導と便宜により利用ができた。それぞれのデ

ータを公開・委託され、利用できるようにしてくださった西澤

教授に感謝いたします。

<補遺>

本稿における作業定義

◇投票時に重視したもの

SQ10. あなたは小選挙区選挙で、政党の方を重く見て投票しまし

たか、それとも候補者の方を重く見て投票しましたか。

1 : 政党を重く見て

2 : 候補者を重く見て

3 : 一概に言えない

9 : わからない

(1 : 政党を重く見て 2 : 候補者を重く見て 3・9 は欠損値)

◇後援会

Q13. あなたは、議員や候補者を後援する団体に加入されていま

すか国政選挙だけでなく、どの選挙のさいの団体でも結構です。

1 : 加入している

2 : 加入していない

9 : わからない

(1 : 加入している 2 : 加入していない 9 は欠損値)

◇支持政党の有無

Q20. あなたは、ふだん何党を支持していらっしゃいますか。

1 : 自由民主党

2 : 民主党

3 : 公明党

4 : 日本共産党

11

5 : 社会民主党

6 : 国民新党

7 : 新党日本

8 : 新党大地

9 : その他

10 : 支持政党なし

11 : わからない

(1~9= 1 : 支持政党あり 10= 2 : 支持政党なし 11 は欠損値)

<参考文献>

・三宅一郎 1989『投票行動』 東京大学出版会

12

3 投票義務感をもたら

すものはなにか 浦田 祐香

1. はじめに

「投票義務感」とは、「支持政党や候補者が勝つ見込みがないと

きは投票しても無駄と思うかどうか、自分一人くらい投票しなく

てもよいと思うかどうかに対する態度」である。「投票義務感」が、

高いほど、有権者の政治参加は高くなる。(蒲島 1988 p.88)議員

内閣制の日本において、選挙は、国民の数尐ない政治参加のうち

のひとつであるため、投票に行くことは、軽視してはならないも

のであり、各々が「投票義務感」を持ち、どんな場合でも投票に

行ことは大変重要である。投票率を向上させ、国民に投票に積極

的な態度を持たせるためにも、「投票義務感」を向上させることは

不可欠であろう。そのような「投票義務感」をもたらすものはな

んであるのか。社会的属性の要素を仮説に挙げ、分析していく。

2. 仮説 有権者に「投票義務感」をもたらすものとして、「教育程度の高

さ」と「有職者であるか否か」という二つの仮説を提示する。

【モデル図】

2-1. 教育程度の高さ

教育程度が高いほど「投票義務感」も高くなると考える。教育

程度が高いほど、学問的に政治や選挙制度獲得の歴史的背景など

について学ぶ機会が多く、投票する重要性を感じるからである。

2-2. 有識者であるか否か

有職者のほうが「投票義務感」が高くなると考える。有職者は、

社会を自分が担っているという意識から生じる責任感があり、ま

有権者であるか否か

投票義務感

教育程度の高さ

13

た、社会に出て、政治や経済に常に関わるため、政策によって受

ける影響が比較的大きくなる。そのため、政治・経済問題により

関心をもつからである。

3. 分析方法 分析は、財団法人 明るい選挙推進委員会によって行われた「第

44 回 衆議院議員総選挙についての意識調査」を用いる。1

そして、そこで調査された質問文と回答を用いて、表 1「投票義

務感」と「教育程度の高さ」のクロス表分析、表 2「投票義務感」

と「有職者であるか否か」のクロス表分析を行う。

3-1. 分析の手順

「投票義務感」については、“あなたはふだん、選挙の投票につ

いて、この中のどれに近い考えをもっていますか。”を用いる。な

お、回答項目の“わからない”は欠損値として処理した。

「教育程度と高さ」については、“あなたは学校はどこまでいら

っしゃいましたか。(在学中・中退は卒業とみなす)”を用いる。

分析の際、回答を三つのグループに分けた。一つ目は、小・高小・

新中卒の教育程度 1 のグループ、二つ目は旧中・新高卒、新高専・

短大・専修学校卒の教育課程 2 のグループ、三つ目は、旧高専大・

新大卒、大学院(修・博)卒の教育課程 3 のグループである。な

お、“わからない”は欠損値として処理した。

「有職者であるか否か」については、“あなたのご職業は何です

か(どんな仕事をなさっているのですか。ご自分で経営していら

っしゃるのですか。勤めていらっしゃるのですか)。を用いた。そ

して、回答を四つのグループに分けた。一つ目は、有職者、二つ

目は、学生、三つ目は、主婦、四つ目は無職である。なお、“わか

らない”は欠損値として処理した。

4. 実証 表 1「投票義務感」と「教育程度の高さ」、表 2「投票義務感」

と「有識者であるか否か」のクロス表分析を行い、以下の結果が

得られた。

表1「投票義務感」と「教育程度の高さ」のクロス表分析

教育程度の高さ

教育程度 1 教育程度 2 教育程度

3

投票義務感 義務である 63.6 55.8 53.8

権利であるが義務ではな

17.4 21.5 31.8

自由である 19.0 22.6 14.3

合計(%) 100 100 100

出所:第 44 回衆議院議員総選挙についての意識調査 カイ二乗値:.30.31 危率:.000

14

表 2「投票義務感」と「有権者か否か」のクロス表分析

有識者であるか否か

有識者 学生 主婦 無職

投 票 義 務

義務である 52.7 37.0 61.4 57.0

権利であるが義務ではない 23.1 29.6 22.6 22.6

自由である 24.2 33.3 16.0 20.5

合計(%) 100 100 100 100

出所第 44 回衆議院議員総選挙についての意識調査 カイ二乗値:.25.88 危率:.000

5. 考察 表 1「投票義務感」と「教育程度の高さ」のクロス表分析の“義

務である”の分布は、教育程度 1 が 66.3%、教育程度 2 が 55.8%、

教育程度 3 が 53.8%を占めた。この値から教育程度 1、つまり、教

育程度が最も低いグループ、最も投票を“義務である”と考えて

いることになる。この結果は、教育程度が高いほど「投票義務感」

も高くなる、という仮説が支持されないということを表す。

“自由である”の分布において、教育程度 3、つまり教育程度が

最も高いグループが 14.3%と、最も低い値を示しており、“権利

ではあるが棄権すべきではない”の分布では、教育程度 3 が 31.8%

という値で最も高い。この結果より、教育程度の高いグループは、

投票が重要でなく、政治参加する必要はないと考えているわけで

はなく、投票は国民の権利ではあり、棄権すべきではないと考え

ていることがわかる。

では、なぜ仮説は支持されなかったのか。これは、教育程度が

高いほど、投票するためにかける時間というコストを使いたがら

ない、つまり、政治参加のコスト感覚が高くなることが要因のひ

とつであると考えられる(蒲島 1988 p.104)。この要因により、教

育程度の高いグループは政治参加、投票に積極的でなくなるため、

「投票義務感」が生まれなくなるのではないか、と考える。

表 2「投票義務感」と「有職者であるか否か」のクロス表分析で

は、有職者の 52.7%、学生の 37.0%、主婦の 61.4%、無職の 66.1%

が、“義務である”という分布を示した。これは、無職の人々が、

最も”義務である”と考えていることを表す。また、有職の 24.2%、

学生の 33.4%、主婦の 16.0%、無職の 13.7%が“自由である”と

いう分布を示した。これは、主婦・無職の人々は比較的、投票を

“自由である”と考えていないということを表す。これらより、

有職者のほうが「投票義務感」が高くなる、という仮説も支持さ

れなかった。

学生の「投票義務感」が低いのは、政治的関わりが尐なく、政

治への関心も薄いことが要因の一つであろう。では、なぜ有職者

の「投票義務感」が低いのか。このことは、「教育程度の高さ」と

同様に政治参加のコスト感覚が高くなることが要因ではないかと

15

考える。有職者は、政治への関心はあるが、主婦や無職の人々よ

り、時間に拘束されているので、投票へのコストが高くなる。そ

れにより、有職者の「投票義務感」が低くなるのだ。

6. まとめ 今回の分析で、自らの仮説は支持されなかったが、「投票義務感」

は、教育程度が低いほど高くなり、また、主婦・無職が最も高い

ということが分かった。この分析結果より、「投票義務感」は、投

票へのコスト感覚と関係があるという考えに至った。今回の分析

では、「投票義務感をもたらすものはなにか」という問いに対し、

社会的属性を用いて分析したが、その他にも要因は存在すると思

う。冒頭でも述べたように、「投票義務感」は、投票と正の相関を

示す重要な態度である。そのため、これからも「投票義務感をも

たらすものはなにか」という問いを研究していくべきだと考える。

(1)ここで利用したデータは、「第 44 回衆議院議員総選挙につい

ての意識調査」(平成 17 年 10 月社団法人中央調査社)である。

それをレヴァイアサン・データ・バンク(LDB、木鐸社)よ

り同志社大学が購入したものを使用した。このデータは、同

志社大学・法学部の西澤由隆先生のご指導と便宜により利用

ができた。このデータを公開・寄託され、利用できるように

してくださった先生方に感謝いたします。

<補遺>

・分析に用いた質問項目 *()内は全て%

・Q3.〔回答票 3〕あなたはふだん、選挙の投票について、こ

の中のどれに近い考えをもっていますか。

(ア)投票することは国民の義務である(56.1)

(イ)投票することは、国民の権利であるが、棄権すべきでは

(22.2)

(ウ)投票する、しないは個人の自由である(20.2)

わからない(1.5)

・F3.(学 歴)あなたは学校はどこまでいらっしゃいました

か。

(在学中・中退は卒業とみなす)

小・高小・新中卒(19.5)

旧中・新高卒 (43.9)

新高専・短大・専修学校卒(18.2)

旧高専大・新大卒(16.2)

大学院(修・博)卒(1.5)

わからない(0.7)

・F4.(本人職業)あなたのご職業は何ですか(どんな仕事を

なさっているのですか。ご自分で経営していらっしゃるので

すか。勤めていらっしゃるのですか)。

【調査員注】いわゆる主婦であっても、家の職業を手伝ってい

る場合は家族従業の方に入れる。

16

農林漁業(4.6) 自 由業・商 工サービス業(11.1) 管 理職(2.6)

専 門・技術職・事務職(16.3) 販 売・保安・サービス従事者

(12.5)

運 輸・通信・生産工程従事者 (9.3)

学 生(1.4) 主 婦(23.7) 無 職(18.3) わ からない(0.3)

<参考文献>

・蒲島郁夫 1988. 『政治参加(現代政治学叢書-6))』東京大学出

版会.

・三宅一郎 1990. 『政治参加と投票行動 大都市住民の政治生

活』 ミネルヴァ書房.

17

4 政党支持 なぜ、無党派層は生まれるのか

塩谷 侑己

1. はじめに 現代の日本の有権者には、政党支持を持たない、いわゆる「無

党派層」が非常に多い。表 1 でわかるように、有権者のうち 30%

が無党派層である。そのため、この無党派層が地方選挙から国政

選挙に至るまで、今日の選挙を左右させると言っても過言ではな

い。また、各政党は無党派層の支持を得ることを目的にした政策

を提言しており、無党派層は日本政治に多大な影響を与えている

と言える。

しかし、本来ならば有権者は、それぞれ支持する政党を持って

いるはずである。三宅一郎も「政治意識の中で最も重要なもので

あり、その中核をなす。」「多くの政治的態度や政治行動は多かれ

尐なかれ政党支持と関連をもつ。」と述べている(三宅 1985,3)。こ

のように、政党支持は投票行動に最も影響を与える尺度の一つで

ある。それにもかかわらず、現代の日本人の多くは、なぜ政党支

持を持たなくなったのであろうか。

以上のことから、本稿では無党派層が生まれる要因について検

証する。

出典:JESⅢ

2. 仮説 本稿では、「有権者が政党の政策を理解していないから、無党派

層が生まれる」という仮説を検証する。

本来、イデオロギーや思想をもとに様々な政治問題に対して独

自の政策を、政党は持っている。そして政党は、その政策を様々

な活動を通じて有権者に主張する。これをもとに有権者は自身の

考えに一致、または近しい政党を支持すると考えられる。そのた

め逆を言えば、有権者が政党の政策を理解できなければ、自身の

考えに一致する政党を見いだせなくなり、無党派層になるのでは

ないかと考えられる。つまり、有権者が「政党の政策を理解して

いるか、していないか」は、「政党支持を持つか、持たないか」を

表1 ふだんの政党支持

自民党 民主党 他の政党 無党派 計 N

政党支持 42.1% 18.1% 9.8% 30.0% 100% 2054

18

決定づける要因になると本稿では考える。

以上のことから、本稿の仮説は「有権者が政党の政策を理解し

ていないから、無党派層が生まれる」とした。

図 1 自仮説のモデル図

3. 分析 3-1. 分析方法

先の仮説を証明するために「21 世紀初頭の投票行動の全国的・

時系列的調査研究 2001-2005」(文中では、JESⅢと略す)から、

2004 年 7 月に行われた参議院選挙における事前・事後調査のデー

タを利用する。1 なお、今回の分析における「有権者が政党の政

策を理解していない」という部分に関しては、有権者の重要度が

最も高かった公的年金制度に対する政策を理解しているかを問う。2 また、近年の政党システムを鑑みて、自民党と民主党の 2 大政

党における先の政策を理解しているかに注目する。

よって、独立変数:政党の政策、従属変数:政党支持のクロス

表を用いて仮説を証明する。詳しい作業定義は、補遺に掲載する。

3-2. 分析結果

「政党の政策」と「政党支持」をクロス表分析したものが表 1

である。なお、危険率は小さく有意である。

表 2 で見られるように、政策を理解している有権者のうち、

75.8%が政党支持を持ち、24.2%が無党派層であった。一方で、政

策を理解していない有権者のうち、59.1 %が政党支持を持ち、

40.9%が無党派層であった。そのため、政党支持を持っている有権

者において、政策を理解している有権者としていない有権者では、

20%程の減尐が見られる。一方、無党派層においては、20%程の増

加が見られる。

これにより、「政党の政策」と「政党支持」には相関関係がある

と確認できる。

出典:JESⅢ カイ二乗値 55.736 危険率 .000

表 2 政党の政策と政党支持

政党支持

計 N 政党支持あり 政党支持なし

政党の政策 理解 75.8% 24.2% 100% 1105

わからない 59.1% 40.9% 100% 682

政党の

政策 政党支持あり

無党派層 わからない

理解

19

3-3. 統制変数

表 2 で見られたように、「政党の政策」と「政党支持」には一定

の相関関係があった。

しかし、因果関係をより確かなものとして検証するためには、

統制変数を用いて「偽の関係」を排除しなければ(高根 1979, 112)

その統制変数として、本稿では「後援会への加入」を使用する。3

今回、「後援会への加入」を統制変数として使用する理由は、後援

会に加入する有権者は、その政党に支持を持っているから加入し、

支持を持たなければ後援会には加入しないと推測するからである。

3-4. 分析結果

表 2 のクロス表に、統制変数を加えたものを表 3・4 で示した。

なお、危険率は小さく有意である。

結果を見ると、表 3 の後援会に加入している人において、政策

を理解している有権者のうち 93.5%が政党支持を持ち、6.5%が無

党派層であった。一方で、政策を理解していない有権者のうち、

65.8%が政党支持を持ち、34.2%が無党派層であった。

同様に、表 4 の後援会に加入していない人において、政策を理

解している有権者のうち 74.8%が政党支持を持ち、25.2%が無党派

層であった。一方、政策を理解していない有権者のうち 62.1%が

政党支持を持ち、37.9%が無党派層であった。

これにより、「後援会への加入」にかかわらず、政党支持を持つ

人と無党派層とでは、政策の理解度に増減が見られるため「政党

の政策」と「政党支持」には相関関係が確認された。

さらに、注目すべき点がこの分析結果で見られる。それは、表 3

の後援会に加入をしている有権者においては、統制変数を導入す

る以前より、両変数間にはより強い相関関係が見られるようにな

った点である。つまり、統制変数である「後援会への加入」は、

両変数間の関係を強調させる作用があるということがわかる。

この要因には、政党支持の強弱が関係すると考察する。一概に

政党支持といっても、人によって強さは違う。例えば、後援会に

加入している人は、比較的強い政党支持を持ち、後援会に加入し

ていない人は、比較的弱い政党支持を持つと考えられる。強い政

党支持を持っているため、政策を理解する有権者の 93.5%が政党

支持を持つという高い数値が、表 3 に現れたと考察する。また、

政策を理解していない人のうち 34.2%が無党派層であったのは、

政党に属さない候補者の後援会に加入しているために、政党の政

策をわからないと答えたのではないかと考える。

以上のことによって、表 3 においてだけに強い相関関係が現れ

たと考察する。

20

出典:JESⅢ カイ二乗値 14.805 危険率 .000

出典:JESⅢ カイ二乗値 25.306 危険率 .000

4. おわりに 本稿で立てた「有権者が政党の政策を理解していないから、無

党派層が生まれる」という仮説は、分析の結果から支持されたと

言える。

このことから、政党が「自分たちの政党は何を目指すのか」「ど

のような政策をとっているのか」などを明確に有権者に主張する

ことができれば、有権者は政策を理解する機会が増え、政党支持

を持つようになる。その結果、無党派層は減尐すると思われる。

また、今回の結果により、選挙前に無党派層が減るという現象

も論理的な説明ができるだろう。なぜなら、選挙前になれば、多

くのマスメディアが政治に関連するニュースを大々的に報じるこ

とになる。すると、必然的に有権者は政党の政策を理解する機会

が増え、無党派層が減尐するというメカニズムが成立するからだ。

最後に、分析を行っていくなかで、いくつかの課題が浮き彫り

になった。本稿では、公的年金制度だけを「政党の政策」として

取り入れた。しかし、一つだけの政策で政党支持を持つ有権者は、

多くはないであろう。例えば、福祉とその負担・財政問題に関す

る政策と合わせて理解し、政党支持を持つかどうかを決める有権

者が多いかもしれない。さらに、無党派層が生まれる要因は、「政

党の政策」だけではないはずだ。これらを踏まえ、次の課題にし

たい。

表 3 政党の政策と政党支持と後援会

後援会に加入している 政党支持 計 N

政党支持あり 政党支持なし

政党の政策

理解 93.5% 6.5% 100% 77

わからない 65.8% 34.2% 100% 38

表 4 政党の政策と政党支持と後援会

後援会に加入していない 政党支持 計 N

政党支持あり 政党支持なし

政党の政策

理解 74.8% 25.2% 100% 902

わからない 62.1% 37.9% 100% 509

21

(1)21 世紀初頭の投票行動の全国的・時系列的調査研究(JES

SSJDA 版),2001-2005」は、JESⅢ研究会が実施された世

論調査である。その個票データについて、東京大学社会科学

研究所附属日本社会研究情報センター・データアーカイブ

(Social Science Japan Data Archive)より教材としての利

用許可(申請者:西澤由隆教授)を得たものを使用した。こ

のデータは、同志社大学・法学部の西澤由隆先生のご指導と

便宜により利用ができた。データを公開・寄託され、利用で

きるようにしてくださった先生方に感謝いたします。

(2)「公的年金制度問題は、あなたにとってどのくらい重要です

か」という質問に対して、「かなり重要である」「やや重要で

ある」を合わせ 86.7%を占めた。次に高かったのは、「福祉

と負担に関する問題」であり、82.6%を占めた。

(3)「偽の関係」とは、両変数間に相関関係が見られても、それ

は第三の変数によって相関関係が見られるようになっている

関係である。

<補遺>

分析に使用した作業定義

・ふだんの政党支持

Q8

「話はかわりますが、今回何党に投票するかは別にして、ふだん

あなたは何党を支持していますか。」

1:自民党

2:民主党

3:公明党

4:社民党

5:共産党

8:その他の政党

9:支持政党なし

10:わからない

11:答えない

→1:自民党 2:民主党 3:他の政党 4:無党派層 9:欠損値

・政党の政策

Q18 SQ3

「それでは、次にあげる政党は、公的年金制度の問題について、

どのような主張をしていると思いますか。自民党についてはいか

がですか。」

A:将来的に安定した財源を確保するために、保険料を値上げすべ

きである。

B:全ての世代が同じように負担するために、消費税を上げるべき

である。

1:A に近い

2:どちらかといえば A

3:どちらかといえば B

22

4:B に近い

5:わからない

6:答えない

→0:理解 1:わからない 9:欠損値

※民主党について尋ねる質問も同じ作業定義でコード化し、if 関数

を用いて

自民党・民主党の政策を理解=0

自民党・民主党の政策がわからない=1

とした。

・政党支持

Q8

「話はかわりますが、今回何党に投票するかは別にして、ふだん

あなたは何党を支持していますか。」

1:自民党

2:民主党

3:公明党

4:社民党

5:共産党

8:その他の政党

9:支持政党なし

10:わからない

11:答えない

→0:政党支持あり 1:政党支持なし 9:欠損値

・後援会

Q12

「今年の 5 月以降、あなたは、選挙運動に関することに参加した

り、働きかけを受けたりしましたか。それはどの政党のものです

か。それぞれあてはまるものをいくつでもお答えください。まず、

「自分が候補者の後援会に入っている」」

1:そのようなことはない

2:入っている

→0:加入 1:加入していない

〈参考文献〉

・久米郁男 川出良枝 古城佳子 田中愛治 真淵勝 2003.『政治学』

有斐閣.

・高根正昭 1979. 『創造の方法学』講談社.

・三宅一郎 1985. 『政党支持の分析』創文社.

・三宅一郎 1989. 『投票行動』 東京大学出版会.

23

5 政治的有効性感覚と

投票 林 香織

1. はじめに 民主主義国に暮らす人にとって、自分の意見を政治に反映させ

る方法はたくさんある。政治家に手紙を書くこともできるし、デ

モによって大々的に主張することも可能だ。インターネットが発

達したおかげで、Twitter や facebook など、政治家と簡単に会話

ができるツールも増えた。このように、民主主義社会には大小様々

な利益表出方法があるが、この中で最も簡単で、かつ公式的なも

のが「投票」である。特定の期間に投票所に行き、国政を託せる

と思った候補者・政党の名前を書く。これだけで自分の主張が「選

挙結果」という形で政治に反映される。

しかし、このような便利な制度があるのにも関わらず、日本国

では投票に行かない人は多い。投票率が年々下がっていることに

加え、若者の政治離れもメディアでよく取り上げられているのが

日本の現状だ。

この「投票という便利なシステムがあるにも関わらず、それに

行く人が減っている」という日本の現状は、何が原因で生じてい

るのだろうか。利益表出という意味で、選挙は有権者にとって有

意なシステムであるはずなのに、どうして利用されないのだろう

か。

以上のことを考えるために、本稿では「有権者はそもそも、ど

ういう心理がはたらけば投票に行くのか」という根本的な問いに

迫りたい。

2. 仮説 上記の仮説のキータームとして、本稿では「政治的有効性感覚」

を挙げる。安野によると、政治的有効性感覚とは「『自分を含め、

有権者ひとりひとりが政治を変えることができる』という意識で

あり、政治参加の基盤となるものの 1 つ」だとしている(安野 2005,

155)。有権者がもしこの意識を持っているなら(つまりもし自分

が政治を変えられると思っているなら)、投票をすることにより、

日本の政治をよりよくしようと思うのではないか。逆にこの意識

を持っていないのなら、日本の未来に票を投じる価値を見出しに

くいのではないか。本稿ではこのように予想し、「政治的有効性感

覚があると、投票に行く」という仮説を立てた。

ここで問題になるのは、有権者はどのような経緯から政治的有

効性感覚を得るのかである。日本国民の中で、物心もつかない時

から「政治を変えられる」と確信する人はなかなかいないだろう。

何かしらの経験・考えなどが、政治的有効性感覚の形成にかかわ

24

っているはずだ。

政治的有効性感覚を得る経験は様々あると思うが、今回はそれ

を「国政への信頼」とし、仮説の統制変数にした。(図 1 参照)有

権者が国政を信頼しているということは、程度の差はあっても、

「国政は自分たちの利益を代表してくれる」と考えているという

ことだ。こういった思想を持っている有権者なら、自分の政治参

加に意味はある、つまり「政治を変えることができる」と思うの

ではないだろうか。そしてそれが、選挙に行くという最終的な政

治参加につながるのである。

以上のことから、本稿の最終的な仮説は「国政を信頼していれ

ば、人は政治的有効性感覚を持ち、投票に行く」とする。

3. 分析方法 2.に示した仮説を検証するために、本稿では「衆議院選挙に関す

る世論調査(1996 年総選挙前後調査)」の選挙後調査を使用した。

(以下 JEDS96 と略す)。1

本稿の仮説検証は二部構成で行った。本稿で証明したいことは

「政治的有効性感覚があると、投票に行く」なので、まずは「政

治的有効性感覚」と「投票」の二つでクロス表分析を行った(表 1)。

次に、上記のメカニズムの前提に「国政への信頼」があるかどう

かを確かめるために、「国政への信頼」・「政治的有効性感覚」・「投

票」の三重クロス表分析を行った(表 2)。

これらの具体的な作業定義は補遺を参照して頂きたい。

4. 検証結果

4-1. 政治的有効性感覚と投票

表 1:政治的有効性感覚と投票のクロス表(%)

投票

合計 N

していない した

政治的

有効性感覚

ない 16.4 83.6 100 578

どちらかといえばない 15.9 84.1 100 227

どちらともいえない 15.8 84.2 100 221

どちらかといえばある 12.0 88.0 100 83

ある 8.1 91.9 100 86

合計

N

15.3

183

84.7

1012

100

1195

カイ二乗値=4.756 危険率=.313

国政を信頼

している

政治的有効性感覚

を持つ 投票する

図 1:本稿の仮説 <統制変数>

25

政治的有効性感覚と投票をクロス表分析すると、表 1 のように

なった。数値だけを見れば、政治的有効性感覚を強く持つほど、

投票に行く人は増えている。しかし危険率が 0.313 と高い値にな

ってしまった。したがって、「政治的有効性感覚」と「投票」の相

関関係は確認できなかった。

4-2. 国政への信頼度・政治的有効性感覚・投票

表 2:信頼・政治的有効性感覚・投票のクロス表(%)

国政への

信頼度

政治的

有効性感覚

投票 合計 N

していない した

なし

ない 17.0 83.0 100 (400)

どちらかといえばない 15.6 84.4 100 (167)

どちらともいえない 19.0 81.0 100 (163)

どちらかといえばある 10.2 89.8 100 (59)

ある 7.7 92.3 100 (52)

あり

ない 10.3 89.7 100 (156)

どちらかといえばない 16.7 83.3 100 (54)

どちらともいえない 6.1 93.9 100 (49)

どちらかといえばある 13.6 86.4 100 (22)

ある 6.5 93.5 100 (31)

合計

(N)

14.6

(168)

85.4

(985)

100

(1153)

国政に対する信頼度:なし カイ二乗値=5.572 危険率=.233

国政に対する信頼度:あり カイ二乗値=3.938 危険率=.415

表 2 は、表 1 の関係性に、統制変数として「国政への信頼度」

を導入した分析結果である。

表 2 から分かる通り、国政への信頼度の有無に関わらず、危険

率がまたもや高くなってしまった。このことから「政治的有効性

感覚が高いと、投票に行く」という仮説に、「国政への信頼度」と

いう前提があることが確認できなかった。

しかし、表 1 の危険率が 0.313 であるのに比べ、表 2 の「国政

を信頼していない」有権者に対する危険率は 0.233 と尐し下がっ

た。つまり、「政治的有効性感覚と投票」に「国政への信頼度」と

いう統制変数を導入すると、尐しだけ相関関係が強くなるという

ことが分かる。したがって、国政を信頼していない人にとっては、

政治的有効性感覚を持っているかどうかが、ほんのわずかでも投

票行動に関係しているのではないかと考えた。

「国政を信頼している」有権者の場合、危険率は 0.415 と大き

い値なので、このような人たちには、本稿の仮説のメカニズムが

全くはたらいていないことが分かる。JEDS96で分析を行うと、「国

政への信頼度」と「投票」の相関関係はみられる。2よって、国政

を信頼している人は、すでに信頼しているということだけで政治

参加がうながされる(信頼と投票の間に、他の要因が入らない)、

26

あるいは政治的有効性感覚以外の要因が働いていることが分かる。

5. まとめ 本稿では、「有権者はそもそも、どういう心理がはたらけば投票

に行くのか」という疑問から、「国政を信頼していれば、人は政治

的有効性感覚を持ち、投票に行く」という仮説を立てた。それを

確認すべく JEDS96 を使って分析を行った。分析は二段階に分け、

まずは「政治的有効性感覚と投票」、次に「国政への信頼度」を導

入した場合の相関関係をみた。その結果、どちらも危険率が高く、

有意な分析結果は得られなかった。

二つの分析の危険率が高い理由には、仮説が間違っていること

も含めて、様々なものがあるだろう。しかし JEDS96 の回答者の

ほとんどが投票をした人だということも、要因の一つだと考えら

れる。3このことにより、表 1・表 2 両方で、投票した人の割合が

約 85%と高い値になってしまった。この数値は尐し現実離れして

いるため、もう尐し投票していない人の標本も利用できていれば、

結果はまた違ったと思われる。

また、政治的有効性感覚を持っている有権者が尐ないことも一

つの要因だ。表 1 を見ると、有効回答者数 1195 人のうち、86 人、

つまり約 7%しかこの意識を持っていない。本稿の仮説は、政治的

有効性感覚を「持つ」メカニズムをさぐったため、相関が出なか

ったのだろう。政治的有効性感覚を「持つ」仮説のメカニズムは、

最初から無理があったようだ。

では、どうして日本国民は政治的有効性感覚を持たないのだろ

うか。今回の統制変数となった「国政への信頼度」・「今まで受け

てきた政治教育の内容」・「政治に対するイメージ」など、様々な

原因が考えられる。また、この約 7%という値は、他国と比べても

低いのだろうか。民主主義が発達した先進国の中で比較すると、

どのような結果が出るだろうか。

本稿の仮説は、残念ながらデータをもって確認することはでき

なかった。しかし、以上のような有権者の投票行動に関する新た

な課題が見つかったという意味では、意味のある分析ではないだ

ろうか。

本稿は、どうして選挙という便利な制度があまり使用されない

のか、という疑問から始まった。(「1.はじめに」参照)これにもた

くさんの答えがあるはずだが、一つには、上記に述べた「政治的

有効性感覚を持つ有権者が尐ないこと」があるだろう。日本は民

主主義体制を取っているのだから、政治を動かすことは可能なは

ずだ。よって日本の政治は、より有権者が政治的有効性感覚を持

てるような社会づくりを目指さなければならない。

日本という国は民主主義を基礎に成り立っている。この民主主

義は、政治家や官僚だけでなく、普段政治的営みを自ら行ってい

ると感じにくい、一般の有権者の意識や行動で成り立っている。

また、東日本大震災が起こり、日本国内の「力」が試されている

今、国民一人一人の政治に対する意識はとても重要だ。この意識

を高めるためにも、新たに出てきた課題と一緒に、有権者の政治

参加について、今後も研究していきたい。

27

(1)ここで利用したデータ「衆議院選挙に関する世論調査(1996

年総選挙前後調査)-1996」は、選挙とデモクラシー研究会

(JEDS)(西澤由隆教授も参加)が実施された世論調査であ

る。同志社大学・法学部の西澤由隆先生のご指導と便宜によ

り利用ができました。このデータを公開・寄託され、利用で

きるようにしてくださった先生方に感謝いたします。

(2)表 3 が、国政への信頼度と投票をクロス表分析にかけたもの

である。国政を信頼していなくて、投票に行った人は約

83%いるのに対し、国政を信頼して投票に行った人は約 89%

と 6%多い。したがって、国政を信頼している有権者の方が

投票に行くことが分かる。また危険率も 0.008%と低いので、

この相関関係が偶然である確率は低い。

(3)表 4 が、JEDS96 における、投票をしていない人・した人の

割合である。分析には欠損値を入れないため、ますます相対

的に投票に行った人が増え、行っていない人が尐なくなって

しまう。

<補遺>

表 3:「国政への信頼度」と「投票」(%)

投票

合計 (N)

していない した

国政への信頼度 なし 16.7 83.3 100 (909)

あり 10.7 89.3 100 (338)

カイ二乗値=7.092 危険率=.008

表 4:衆議院選挙の投票をしたかどうか(%)

投票を

していない 14.0

した 72.3

その他(欠損値) 13.7

合計(N) 100(1535)

分析に使用した作業定義

・国政への信頼度

Q24 あなたは国の政治をどれくらい信頼できるとお考えでし

ょうか。いつも信頼できる、大体信頼できる、時々は信頼で

きる、全く信頼できないのうちどれでしょうか。

1. いつも信頼できる

2. 大体信頼できる

3. 時々は信頼できる

4. 全く信頼できない

5. DK

⇒1,2 を「1. 信頼している」、3,4 を「0. 信頼していない」、そ

の他のコードを「99. 欠損値」に再コードした。「3. 時々は

28

信頼している」は、一応信頼があるように見受けられるが、

それを「1. 信頼している」に入れてしまうと、あまりにもそ

の度数が大きくなってしまう。また、「時々は」となれば、

信頼度はかなり低いものと予想できるので、今回の分析では

「0. 信頼していない」とコード化した。

・政治的有効性感覚

Q20〔回答票 23〕次に国民と選挙や政治とのかかわり合いにつ

いての意見を読み上げます。それぞれについてのあなたのお

気持ちをこの中からお答えください。1) 自分には政府のす

ることに対して、それを左右する力はない。

1. 賛成

2. どちらかといえば賛成

3. どちらともいえない

4. どちらかといえば反対

5. 反対

6. DK

7. NA

⇒順番に、政治的有効性感覚が「-2.ない」・「-1.どちらかとい

えばない」・「0.どちらともいえない」・「1.どちらかとい

えばある」・「2.ある」、そして最後の二つを「99. 欠損値」

に再コードした。

・投票

Q2〔回答票 2〕いろいろな事情で選挙のときに投票に行けない

人がありますが、あなたは、10 月 20 日(日)の衆議院選挙

の投票に行きましたか。この中からあげてください。

1. 行くつもりだったが、行けなかった

2. 行かなかった

3. 行った

4. わからない

5. DA

⇒1,2 を「0. 投票していない」、3 を「1. 投票した」、その他

のコードを「99. 欠損値」に再コードした。

<参考文献>

・安野智子 2005.『JGSS−2003 にみるパーソナル・ネットワー

クと政治意識』JGSS 研究論文集[4] .

・原田唯司 1994.『政治的有効性感覚,政治に対するイメージと

政治的態度の関連』静岡大学教育学部研究報告. 人文・社会

科学篇. 44, P.217-233.

29

6 制度認知と関係する

要因とは 学歴・政治関心との相関関係

水田 紗希子

1. はじめに

期日前投票とは、2003 年に制定された制度である。この制度は従来実

施されてきた不在者投票制度よりも手続きが簡単で、忙しい有権者にとっ

て投票しやすい環境を、ということを目標として実施されている。そして、

小泉純一郎率いる自民党が大勝した第 44 回衆議院議員総選挙は、衆議院

選挙において、期日前投票が初めて実施された選挙であった。この時、導

入されてまだ日の浅いこの期日前投票制度を、一体どのような有権者が認

知していたのか、今回は学歴と政治関心に注目し、この二つが制度認知に

おいて必要なものであるのかを本稿で検証していきたい。

2. 仮説

本稿の命題は三つある。まず一つ目は、「学歴の高い人ほど政治関心も

高い」、そして二つ目は、「政治関心の高い人は、期日前投票制度について

も知っている」である。最後に三つ目は、「学歴の高い人ほど政治関心は

高まり、期日前投票制度についても認知している」である。

本稿の最終目的としては、学歴と政治関心の関係、政治関心と制度認知

の関係を順に検証することで、「学歴の程度によって政治関心も異なり、

制度認知の有無も変わってくるのではないか」という仮説を確かめること

である。

この仮説を図化したものが、図1である。

図1:本稿の仮説

学歴 政治関心 期日前投票

の認知

30

3. 分析方法

分析には、明るい選挙推進委員会によって行われたアンケートである

「第44回衆議院議員選挙に対する意識調査」を用いる。1

まず、学歴(独立変数)と政治関心度(従属変数)とのクロス表分析(表

1)、次に政治関心度(従属変数)と期日前投票の認知(独立変数)とのク

ロス表分析(表2)を行う。この二つの関係を前提としたうえで、学歴・

政治関心・期日前投票の認知での多重クロス表分析(表3)を行う。

具体的な作業定義については、補遺を参照して頂きたい。

4. 実証

4-1. 分析

ここでは、政治関心が学歴の高さによって身につく度合いが異なるのか

を検証し、学歴の高い人ほど政治関心も高いのかどうかを確かめる。表1

は、政治関心と学歴についてのクロス表分析を行ったものである。

出所:第44 回衆議院議員選挙に対する意識調査

表1を見ると、高学歴になるにつれて政治関心は高くなるといえる。そ

のため、この結果は本稿の命題 1 を支持していると言える。しかし、「多

尐関心あり」の項目を選ぶ人が全学歴を通して多いことから、政治関心度

についての選択肢に尐し問題があるように感じた。この項目は、「多尐」

という言葉の曖昧さ、または「関心あり」という文末であることによって、

「関心なし」を選ぶよりかは印象が良さそう、という思いから、実際はあ

まり政治に興味はない人でも選びやすいものなっているのではないだろ

うか。つまり、質問の書き方により、回答者に答えやすいもの、そうでな

いものがあるのではないか、ということが分かった。

次に、表2は、政治関心と期日前投票制度を認知しているかについての

クロス表分析である。これにより、政治関心が高い人ほど制度を認知して

いるかどうかを確かめる。

表1 学歴と政治関心についてのクロス表(%)

政治関心

合計 (N)

非常に

関心あり

多尐

関心あり

あまり

関心なし

全く

関心なし

学歴

低学歴 21.5 52.9 19.2 6.4 100.0 (312)

中学歴 21.8 60.3 14.8 3.1 100.0 (1004)

高学歴 38.0 51.9 9.4 0.7 100.0 (287)

合計

(N)

24.6

(395)

57.3

(919)

14.7

(236)

3.3

(53)

100.0 (1603)

カイ二乗値=54.265 危険率=0.00

31

表2より、政治関心の高い人(項目でいうと、「非常に関心あり」と「多

尐関心あり」の人)は、政治関心の低い人(「あまり関心なし」と「全く

関心なし」の人)よりも期日前投票制度の認知度が高く、95%を超えてい

た。これにより、この結果は本稿の命題2を支持していると言えるだろう。

そして最後、表3は、期日前投票の認知具合と政治関心、そして学歴と

の多重クロス表分析の結果である。

表 3 を見ると、確かに学歴が高くなることにより、関心度は高くなり、

そしてそれに比例して期日前投票制度の認知も上がっている。だが、この

ことは低学歴・中学歴の人にしか言えない。なぜなら、高学歴の人の多重

クロス表分析の結果が有意ではないからである。これはつまり、「高学歴

表2 政治関心と制度認知についての投票のクロス表(%)

期日前投票制度の認知

合計 (N)

知らない 知っている

政治

関心

非常に関心あり 1.5 98.5 100.0 (396)

多尐関心あり 4.2 95.8 100.0 (921)

あまり関心なし 11.5 88.5 100.0 (207)

全く関心なし 29.4 70.6 100.0 (51)

合計

(N)

5.4

(87)

94.6

(1515)

100.0 (1602)

カイ二乗値=88.943 危険率=0.00

出所:第44 回衆議院議員選挙に対する意識調査

表3 学歴・政治関心・制度認知についての多重クロス表(%)

政治関心度 合計 (N)

制度の

認知

非常に

関心あり

多尐

関心あり

あまり

関心なし

全く

関心な

低学歴 ○ 22.8 53.7 19.2 4.3 100.0 (281)

× 11.1 44.4 22.2 22.2 100.0 (27)

中学歴 ○ 22.7 61.5 13.5 2.3 100.0 (947)

× 5.9 39.2 37.3 17.6 100.0 (51)

高学歴 ○ 39.1 51.4 8.7 0.7 100.0 (276)

× 0.0 77.8 22.2 0.0 100.0 (9)

低学歴:カイ二乗値=15.460 危険率=0.001

中学歴:カイニ乗値=65.336 危険率=0.000

高学歴:カイニ乗値=6.491 危険率=0.090

出所:第44 回衆議院議員選挙に対する意識調査

32

の人は政治関心云々に関係なく、期日前投票制度を知っている」というこ

とではないだろうか、と私は考えた。そのため、新たにもう一つの検証を

行うこととする。

表4は、学歴(高学歴)と期日前投票制度の認知について新たに行った

クロス表分析の結果である。

表4 学歴と制度認知についてのクロス表(%)

期日前投票制度の認知

学歴 知らない 知っている 計 (N)

高学歴 3.2 96.8 100.0 (285)

カイ二乗値=11.912 危険率=0.003

出所:第44 回衆議院議員選挙に対する意識調査

表 3、4 より、高学歴の人は関心度云々ではなく、最早知識の一つとし

て期日前投票を当然知っているのではないのだろうか、ということが言え

るのではないだろうか。そして、表3において高学歴に関して有意な結果

が出なかったのは、このことから高学歴の内で期日前投票を知らない人が

非常に尐なかったこと(アンケート回答者1591人中9人しかいなかった)

が大きな理由であろう。

4-2. 分析結果

表 1~4 の分析結果を踏まえると、低学歴・中学歴の人については、学

歴の程度によって政治関心、そして制度認知度も決まってくる、というこ

とが出来る。勿論、低学歴の人より中学歴の方が政治関心度・認知度は上

であることもわかった。図2は、この結果を図化したものである。

図2 低学歴・中学歴の制度認知過程

高学歴の人については、政治関心によって制度認知度が異なるのではな

く、元々政治関心が低くても期日前投票制度を知識として知っている、と

いうことが出来る。この結果を図化したものが図3である。

図3 高学歴の制度認知過程

学歴 政治関心 期日前投票

の認知

学歴 政治関心 期日前投票

の認知

33

5. まとめ

本稿では、新制度に対する認知が学歴や政治関心によっていかに変化す

るかの分析を行った。今回は2005年の衆議院総選挙という選挙一回分の

データのみを用いたが、今後の目標としては、期日前投票が導入されてし

ばらくたった最近のデータの比較や、認知に対してほかにどんな要因があ

るかを分析していけたらと思う。

また、この分析を行ったことで、私自身期日前投票制度に対して新たな

考えを持つことができた。私は以前まで、期日前投票制度は、レジャーな

どの簡単な理由で利用出来る、どちらかといえば投票意欲の低い人が利用

する制度なのではないかと思っていた。だが調べていくと、期日前投票制

度は手続きが簡単で、投票率向上に確かに役立っていることを知った。そ

してむしろ、この制度を利用する人こそ、政治・選挙に意欲的な人なのか

もしれないのである。

期日前投票制度の有効性を今後研究していく上で、上記のような近年と

の期日前投票制度の認知度・使用度の比較や、期日前投票制度や不在者投

票がない国の投票率を検証していくことなどが新たな課題となるだろう。

(1) ここで利用したデータは、「第44 回衆議院総選挙についての意識調

査」である。「第44 回衆議院議員総選挙についての意識調査」は明る

い選挙推進協会からの依頼を受けて2005 年(平成17 年)10 月に社団法

人中央調査社が実施した世論調査である。同志社大学・法学部の西澤由

隆先生のご指導と便宜により利用がした。データを公開・寄託され、利

用できるようにしてくださった先生方に感謝いたします。

<補遺>

分析に使用した作業定義

・期日前投票の認知

Q4. 仕事や旅行、冠婚葬祭などで投票日当日に投票できない方

が、公示日の翌日から投票日の前日までの間、投票することがで

きる期日前投票制度をあなたはご存知でしたか。

知っていた/知らない

・政治関心

Q4. あなたはふだん国や地方の政治についてどの程度関心をも

っていますか。

非常に&多尐関心を持っている/ほとんど&まったく関心を持っ

ていない

・学歴

F3. (学 歴)あなたは学校はどこまでいらっしゃいましたか。

(在学中・中退は卒業とみなす)

・低学歴…小・高小・新中卒

・中学歴…旧中・新高卒&新高専・短大・専修学校卒

・高学歴…旧高専大・新大卒&大学院(修・博)卒

34

<参考文献>

総務省 HP

http://www.soumu.go.jp/senkyo/senkyo_s/news/touhyou/kijits

umae/index.html 2012 年 1 月 20 日参照.

35

7 なぜ世代間で投票率

に差が生じるのか 尐子高齢化に着目して

山内 逸平

1. はじめに 「若者の投票率が低い」ことや「若者の選挙離れ」は昨今よく

耳にすることである。明るい選挙推進協会によると、平成 21 年に

行われた第 45 回衆議院議員選挙では 60 歳代の人の投票率が約

84%であるのに対して、20 歳代の人の投票率は約 49%と非常に低

い。1 すなわち高齢者層は総じて投票率が高く、若年層はその半分

程度の投票率しかないということが、年齢という観点から見た最

近の有権者の投票行動における特徴だと言える。

では、この世代間での投票率の差は一体何に起因しているのだ

ろうか。若者側に働きかけるものや高齢者側に働きかけるものな

ど様々な要因が想定されるだろう。今回、私は日本が直面してい

る「尐子高齢化」に着目することで、なぜ世代間で投票率に差が

生じるのかを分析したいと思う。

2. 仮説 尐子高齢化が進行している日本においては、人口に占める高齢

者の割合が非常に高い。この状況下では政党や候補者は高齢者を

重視した政策や主張を行うことにより高齢者へ働きかけ、自分た

ちへの支持を期待するのではないか。また、それゆえ若者への働

きかけは軽視されてしまう傾向にあるのではないかということを

推測した。そして、このような動きにより世代間で政治への満足

度の差が生まれ、これが世代間での投票率の差の原因になってい

るのではないかと考えた。下図がそのモデル図である。

以上により私は仮説を「世代間で政治への満足度が異なるので、

投票率にも差が出る」と設定した。次節以降でこれを検証し考察

していく。

【図】

高齢化の進展 政党・候補者の

高齢者重視

若年層の選挙離

36

3. 検証

3-1. 使用するデータ

本稿の検証で使用するデータはどれも明るい選挙推進委員会

「第 44 回衆議院議員総選挙についての意識調査」によるものであ

る。2 なお作業定義は補遺に示している。

3-2. 年齢と政治参加の有無

まず前提として、「年齢」と「政治参加の有無」に相関関係があ

るのかを確認する。以下の表 1 が回答者の年齢と投票の有無をク

ロス表にしたものである。検証での年齢層の定義だが、20 歳から

39 歳を「若年層」、40 歳から 59 歳を「中年層」、60 歳以上を「高

齢層」とした。

表 1:年齢層と政治参加のクロス表

投票か棄権か 合計(%) N

投票 棄権

年齢層

若年層 69.3 30.7 100 381

中年層 88.3 11.7 100 564

高齢層 86.8 13.2 100 674

カイ二乗値 69.448 危険率 .000

若年層で投票した人は 69.3%で棄権した人は 30.7%である。一

方、中年層・高年層では投票した人がどちらも 80%を超えており、

棄権した人の割合はかなり低くなっていることが表 1 から分かる。

また、カイ 2 乗値・危険率が示しているように「年齢」と「投票

の有無」は相関関係にあると言える。つまり「高齢になるにつれ

選挙で投票する人の割合は多くなる」ということが言える。

3-3. 年齢と政治満足度

次に仮説の検証を行う。「年齢」と「今の政治への満足度」をク

ロス表にしたものが以下の表 2 である。

表 2:年齢層と政治満足度のクロス表

今の政治の満足度

合計(%) N

満足 不満足

年齢層

若年層 25.1 74.9 100 355

中年層 21.9 78.1 100 552

高齢層 34.1 65.9 100 625

カイ二乗値 23.1 危険率 .000

37

表 2 から分かることは、年齢層が上がるにつれて政治に満足し

ている人の割合が上昇し、不満足だと感じている人の割合が低下

しているということである。また、こちらもカイ 2 乗値と危険率

が示しているように、「年齢」と「政治への満足度」には相関関係

があると言える。つまり本稿での「世代間で政治への満足度が異

なるので、世代間で投票率が異なる」という仮説は現時点では支

持されたということが言える。なお、例外的に中年層では満足し

ている人の割合が最低であり、不満足だと感じている人の割合が

最高となっているが、若者の政治満足度は低く、高齢者の政治満

足度が高いということが言えることに変わりはないので中年層の

結果は無視することにする。

3-4. 年齢と政治満足度と政治信頼度

これまでの過程を経て仮説は支持されているように見えるが、

それが本当なのかを確認する必要がある。そのために今回は「政

治への信頼」を第 3 の変数として導入し統制を図ることにする。

表 2 のクロス表にさらに第 3 変数を掛け合わせた 3 重クロス表が

以下の表 3 である。

まず、上段の「政治を信頼している」人の間での「年齢」と「政

治満足度」の関係を表したクロス表に注目する。高齢層で政治に

満足している人が 62.4%と他の年齢層と比べて最高の値を示した。

一方で、若年層・中年層の政治に不満足である人は、それぞれ

48.2%・50.3%と高齢層の 37.6%という値に比べてかなり高い値

になった。またカイ 2 乗値・危険率が示しているように、政治を

信頼している人の間では第 3 変数は仮説にほとんど影響を与えず、

仮説は支持されたと言える。

次に下段の「政治を信頼していない」人の間での「年齢」と「政

治満足度」の関係を表したクロス表に注目する。カイ 2 乗値・危

険率の値からも分かるように、有意な検定にはならなかった。表

からも読み取れるように、どの年齢層も 90%に近い割合の人が不

満足となっている。つまり、政治を信頼していない人の間では仮

表 3:年齢層と政治満足度と政治への信頼のクロス表

今の政治の満足度 合計(%) N

満足 不満足

政治を信頼

している 年齢層

若年層 51.8 48.2 100 110

中年層 49.7 50.3 100 175

高齢層 62.4 37.6 100 266

カイ二乗値 8.012 危険率 .018

今の政治の満足度 合計(%) N

満足 不満足

政治を信頼

していない 年齢層

若年層 12.8 87.2 100 243

中年層 9.3 90.7 100 365

高齢層 13.1 86.9 100 336

カイ二乗値 2.917 危険率 .233

38

説は全く支持されず、年齢に関係なく政治が不満足であることに

偏るということである。

よって「年齢」と「政治満足度」の関係は相関関係があるよう

に見えたが、政治を信頼している人に限って相関関係が見られる

ということが分かった。

4. 考察 3 節ではクロス表分析による検証を行ってきたが、検証結果につ

いて考察する。

仮説における「年齢」と「政治への満足度」の関係は、政治を

信頼している人の場合では殆ど影響を与えず、信頼していない人

の場合では「満足度」を不満足に偏らせる傾向が見られた。これ

は「世代間で政治への満足度が異なるので、投票率にも差が出る」

という仮説が成立するための前提として「政治への信頼」が不可

欠であるということである。しかし危険率からも分かるように、

政治を信頼している人の場合でも完全に仮説が支持されていると

いう訳ではない。ゆえに政治を信頼していることは仮説が成立す

るために必要な前提の 1 つであり、他にも仮説に影響を与えうる

前提が存在しているということが言えるだろう。

5. おわりに 考察からも分かるように本稿での仮説は完全には支持されなか

った。すなわち「年齢」と「政治への満足度」は完全に相関関係

があるというわけではなく、それが成立するためには前提の条件

の 1 つとして政治への信頼が必要であるということだ。よって年

齢を問わず投票率を上昇させるためには有権者からの政治への信

頼を得ることが優先されるべきことだと言える。

一方で、仮説が成立するためのその他の前提条件の存在は指摘

できたものの、それが何であるのかということにまでは踏み込む

ことができなかった。ゆえに、さらなる仮説の検証を今後の課題

としたい。また、調査が行われる年によって状況が変化する高齢

化を扱ったものであるだけに、今回の検証・考察に留まるだけで

なく、過去の選挙やこれからの選挙のデータを用いた検証・考察

を試み、比較するということも重要であろう。

また、検証において中年層の動きが非常に興味深いものである

ことも判明した。それは表 1 において中年層で投票した人の割合

が最も高いということ、表 2・表 3 において政治に満足している人

の割合が他の世代よりも低く、不満足である人の割合が高いとい

うことだ。これは「中年層が若年層よりも政治に不満を持ってい

るにも関わらず、政治に参加している割合は高齢層よりも高い」

ということである。しかし本稿では、なぜ中年層がこのような値

を示したのかは考察することができなかった。よって、中年層の

動きに関する考察も今後の課題である。

39

(1)財団法人明るい選挙推進協会のホームページ「衆議院議員選

挙年齢別投票率の推移」のグラフより

http://www.akaruisenkyo.or.jp/070various/071various/3

77

(2)ここで利用したデータは、明推協データである。明推協選挙

後調査(2005 年)」は、全国明るい選挙推進協会が実施した

世論調査である。それをレヴァイアサン・データ・バンク

(LDB、木鐸社)より同志社大学が購入したものを使用した。

いずれも、同志社大学・法学部の西澤由隆先生のご指導と便

宜により利用ができた。それぞれのデータを公開・寄託され、

利用でるようにしてくださった先生方に感謝いたします。

<補遺>

分析で使用した質問と回答をどのように再コードしたかを以下

に示す。

・投票の有無

Q5:「あなたは今回の衆議院選挙では、投票しましたか、しませ

んでしたか。この中から1つだけ選んでください。」

投票:小選挙区・比例代表とも投票した・小選挙区だけ投票し

た・比例代表だけ投票した/棄権:どちらも投票しなかった

/欠損値:わからない

・年齢

フェースシート:「あなたのお年は満でおいくつですか。」

若年層:20~39 歳/中年層:40~59 歳/高齢層:60 歳以上

・今の政治の満足度

Q23:「あなたは、現在の政治に対してどの程度満足しています

か。」

満足:いつも信頼している・たいてい信頼している/不満足:あ

まり信頼していない・まったく信頼していない/欠損値:わ

からない

・政治への信頼

Q24:「あなたは、日本の政府が「正しいことをしてくれる」と、

どれくらい信頼していますか。」

信頼している:いつも信頼している・たいてい信頼している

/信頼していない:あまり信頼していない・まったく信頼してい

ない/欠損値:わからない

<参考文献>

・高根正昭 1979. 『創造の方法学』 講談社.

40

8 生活満足度が政治満

足度に及ぼす影響に

ついて 所得は生活満足度に影響するか

渡邊 智弘

1. はじめに 選挙において、有権者の主な投票理由は、景気や年金問題など

の日々の生活に直結するものであると考えられる。たとえば、明

るい選挙推進協会編の「第 22 回参議院議員通常選挙の実態」をみ

ると、有権者が第 22 回参議院議員通常選挙の投票の際に考慮した

問題として、最も多かった回答が景気・雇用問題、次に多かった

回答が医療・介護問題と年金問題、三番目に多かった回答が税金

問題であった。1 このことから、有権者にとって生活に直接関係す

る問題が、選挙の主な関心事項になっているといえるだろう。つ

まり、有権者は日々の生活に直結する問題の解決を政治に求めて

いるものと思われる。生活の問題が解決されなければ、それは政

治への不満に結びつくであろう。ゆえに、生活に満足感を抱けば

政治に満足感を抱くのではないだろうか。

本稿では、生活満足度と政治満足度の関係について検証してい

く。

2. 仮説 設定する仮説は、「生活満足度が高ければ、政治満足度も高くな

る」というものである。一般に、「生活満足度が低ければ、政治満

足度も低くなるという傾向」にある(明るい選挙推進協会 2011)。

生活に不満を抱いている有権者が、政治に不満を抱くという論理

は理解しやすい。では、生活に満足感を抱いている有権者は政治

に満足感を抱くのであろうか。両者を説明するモデルとして、「業

績投票」モデルがあげられる。このモデルによると、有権者は「過

去の政府業績の判断と未来への期待」(三宅 1989)により投票を

行うとされる。業績投票では、有権者は、過去の政府の業績が良

ければ政権党に投票し、悪ければ野党に期待して投票する。前述

したように、有権者の選挙での主要な関心事項は、自身の生活に

直接関係する問題である。そのため、政府の業績も日々の生活に

かかわる問題であるといえる。

ゆえに、「生活満足度が高ければ政治満足度も高くなる」という

仮説を立てた。本稿の仮説をモデル図にしたものが下記の図 1 で

ある。

41

図 1 本稿の仮説

3. 検証方法 本稿では、仮説を検証するために用いるデータとして、明るい

選挙推進協会のデータの中から「明推協選挙後調査(2003)」を用

いる。2 使用する変数は、独立変数が生活満足度、従属変数が政治

満足度、そして、統制変数として所得を用いる。

まず、生活満足度と政治満足度のクロス表分析を行う。3

次に、生活満足度と政治満足度と所得のクロス表分析を行う。4

4. 分析結果

4-1. 政治満足度と生活満足度のクロス表

生活満足度と政治満足度のクロス表分析の結果は、以下の結果

となった。

表 1 政治満足度と生活満足度のクロス表分析

生活満足

政治満足度

計(%) とても満足 やや満足 やや不満 とても不満

とても満

6.8 26.2 52.4 14.6 103

100.0

やや満足 0.3 27.3 58.3 14.1 1191

100.0

やや不満 0.5 5.9 59.7 33.9 573

100.0

とても不

0.7 2.8 21.5 75.0 144

100.0

計 15

0.7

390

19.4

1121

55.7

485

24.1

2011

100.0

カイ二乗値:432.318 危険率:0.000

出所:第 43 回衆議院議員総選挙についての意識調査

表 1 を見ると、生活満足度でとても満足と答えた人の 67.0%が

政治満足度においてとても不満・やや不満と回答している。次に、

生活満足度をやや満足と回答した人の 72.4%が政治満足度におい

て、とても不満・やや不満と回答している。よって、生活満足度

を満足と回答しても、政治満足度で不満と回答している割合が七

割近くにも上る結果となった。政治満足度で満足の度合いが高か

ったのは、生活満足度でとても満足と回答した人々である。

一方、生活満足度で、やや不満と回答した人が政治満足度で満

足と回答した割合は、とても満足・やや満足を合計しても 6.4%に

すぎない。反対に、政治満足度を不満であると回答した割合はや

所得

(統制変数)

高い

生活満足度

(独立変数)

高い

政治満足度

(従属変数)

42

や不満が 59.7%、とても不満が 33.9%となっており計 93.4%とな

っている。さらに、生活満足度をとても不満と回答した人が、政

治満足度で満足と回答した割合はとても満足が 0.7%、やや満足が

2.8%となっている。政治満足度でやや不満と回答した率が 21.5%、

とても不満と回答した率が 75.0%となった。

生活満足度を満足と回答した人が、政治満足度も満足であると

回答するというようなきれいな結果にはならず、どの生活満足度

でも、政治満足度においてやや不満・とても不満の占める割合が

六割を超えた。生活満足度が低くなるほど、政治満足度でとても

不満と回答する割合は高くなる傾向にある。また、表を見ると生

活満足度が高くなればなるほど、政治満足度においてとても満

足・やや満足と回答する割合も高い傾向にある。生活満足度をと

ても満足・やや満足と回答した人が、政治満足度をとても満足・

やや満足と回答する割合は生活満足度をとても満足と回答した人

が 6.8%、26.2%、やや満足と回答した人が 0.3%、27.3%という割

合となった。

4-2. 生活満足度と政治満足度と所得とのクロス分析

次に、先ほどの二変数に所得を加えクロス表分析を行う。クロ

ス表分析の結果は以下の結果となった。

表 2 月の所得が 10 万円未満の層のクロス表分析

生活満足

政治満足度

(%)

とても満

やや満足 やや不満 とても不満

とても満

0.0 33.3 55.6 11.1 9

100.0

やや満足 1.1 21.3 68.1 9.6 94

100.0

やや不満 0.0 4.2 58.3 37.5 48

100.0

とても不

0.0 .0 25.0 75.0 12

100.0

計 1

0.6

25

15.3

100

61.3

37

22.7

163

100.0

カイ二乗値:40.701 危険率:0.000

出所:第 43 回衆議院議員総選挙についての意識調査

表 2 では、生活満足度をとても満足と回答した人の中で、政治

満足度をやや満足と回答した率が 33.3%という割合になり、やや

不満・とても不満と回答した率が、それぞれ 55.6%、11.1%という

割合となった。同様に、生活満足度をやや満足と回答した人の中

で、政治満足度をとても満足・やや満足と回答した率は、それぞ

れ 1.1%、21.3%という結果となり、政治満足度をやや不満・とて

も不満と回答した率は、それぞれ 68.1%、9.6%という結果となっ

た。生活満足度をやや不満と回答した人の中で、政治満足度を満

足と回答した人は、やや満足の 4.2%のみであった。政治満足度を

43

やや不満・とても不満と回答した人の割合はそれぞれ 58.3%、

37.5%という結果となった。生活満足度をとても不満と回答した人

は、政治満足度を満足と回答した人はいなく、やや不満が 25%、

とても不満が 75.0%という結果となった。この表からも生活満足

度が高いと、政治満足度が高くなるという傾向がみられる。

表 3 月の所得が 10 万円以上 20 万円未満の層のクロス表分析

生活満足度

政治満足度

計(%) とても満足 やや満足 やや不満 とても不

とても満足 0.0 25.0 50.0 25.0 8

100.0

やや満足 0.0 21.6 63.1 15.3 111

100.0

やや不満 0.0 5.6 52.8 41.7 72

100.0

とても不満 4.8 4.8 19.0 71.4 21

100.0

計(%) 1

0.5

31

14.6

116

54.7

64

30.2

212

100.0

カイ二乗値:48.403 危険率:0.000

出所:第 43 回衆議院議員総選挙についての意識調査

表 3 において、生活満足度をとても満足と回答し、政治満足度

をとても満足・やや満足と回答した割合は、それぞれ 0%・25.0%

という結果であった。政治満足度をやや不満・とても不満と回答

した率はそれぞれ、50.0%・25.0%であった。次に、生活満足度を

やや満足と回答し、政治満足度をやや満足と回答した率は、21.6%

であった。やや不満・とても不満と回答した率はそれぞれ 63.1%・

15.3%という結果であった。生活満足度をやや不満と回答し、政治

満足度をやや満足と回答した率は、5.6%という結果であった。政

治満足度をやや不満・とても不満と回答した率は、それぞれ

52.8%・41.7%であった。最後に、生活満足度をとても不満と回答

し、政治満足度をやや不満・とても不満と回答した率は、それぞ

れ 19.0%・71.4%となっている。一方で、政治満足度をとても満足・

やや満足と回答した率はともに 4.8%という数値が出た。しかし、

全体としては、生活満足度が高い方が政治満足度も高い傾向とな

っている。

44

表 4 月の所得が 20 万円以上 30 万円未満の層のクロス表分析

生活満足度

政治満足度

計(%) とても満足 やや満足 やや不満 とても不

とても満足 0.0 33.3 33.3

33.3

3

100.0

やや満足 0.7 24.6 62.3 12.3 138

100.0

やや不満 0.0 7.9 66.7 25.4 63

100.0

とても不満 0.0 5.6 16.7 77.8 18

100.0

計(%) 1

0.5

41

18.5

132

59.5

48

21.6

222

100.0

カイ二乗値:48.231 危険率:0.000

出所:第 43 回衆議院議員総選挙についての意識調査

表 4 において、生活満足度をとても満足と回答した人の政治満

足度は、やや満足・やや不満・とても不満がどれも 33.3%という

値を出した。生活満足度をやや満足と回答した人の中で、政治満

足度をとても満足・やや満足と回答した率は、それぞれ 0.7%、

24.6%であった。同様に政治満足度をやや不満・とても不満と回答

した率は、それぞれ 62.3%・12.3%であった。生活満足度をやや不

満と回答した人が、政治満足度をとても満足・やや満足・やや不

満・とても不満と回答した率はそれぞれ 0%・7.9%・66.7%・25.4%

という結果であった。生活満足度をとても不満と回答し、政治満

足度をとても満足・やや満足と回答した率はそれぞれ 0%・5.6%、

やや不満・とても不満と回答した率は、それぞれ 16.7%・77.8%と

なった。この表においても、生活満足度の低さが政治満足度の低

さにつながる傾向がみられ、生活満足度の高低により政治満足度

も左右される傾向がこの表でも示された。

45

表 5 月の所得が 30 万円以上 40 万円未満の層のクロス表分析

生活満足度

政治満足度

計(%) とても満足 やや満足 やや不満 とても不

とても満足 16.7 0.0 50.0 33.3 6

100.0

やや満足 0.0 26.0 63.0 11.0 100

100.0

やや不満 0.0 1.8 61.8 36.4 55

100.0

とても不満 0.0 0.0 11.8 88.2 17

100.0

計(%) 1

0.6

27

15.2

102

57.3

48

27.0

178

100.0

カイ二乗値:88.332 危険率:0.000

出所:第 43 回衆議院議員総選挙についての意識調査

表 5 において、生活満足度をとても満足と回答し、政治満足度

を満足と回答した率は、とても満足の 16.7%のみであった。生活

満足度をやや満足と回答し、政治満足度をとても満足・やや満足・

やや不満・とても不満と回答した率は、それぞれ 0%・26.0%・

63.0%・11.0%という結果となった。次に、生活満足度をやや不満

と回答した人が、政治満足度をとても満足・やや満足と回答した

率は、それぞれ 0%・1.8%となり、政治満足度をやや不満・とても

不満と回答した率は、それぞれ 61.8%・36.4%という結果となった。

生活満足度をとても不満と回答し、政治満足度をやや不満・とて

も不満と回答した率は、それぞれ 11.8%・88.2%という結果となっ

た。この表では、生活満足度をやや満足と回答し、政治満足度を

やや満足と回答した人が 26.0%となっている。しかし、全体とし

てみれば生活満足度が高いほど、政治満足度も低くなる傾向にあ

る。

46

表 6 月の所得が 40 万円以上 50 万円未満の層のクロス表分析

生活満足度

政治満足度

計(%) とても満足 やや満足 やや不満 とても不

とても満足 0.0 0.0 100.0 0 .0 4

100.0

やや満足 0.0 19.6 58.7 21.7 46

100.0

やや不満 0.0 4.8 52.4 42.9 21

100.0

とても不満 0.0 14.3 28.6 57.1 7

100.0

計(%) 0

0.0

11

14.1

44

56.4

23

29.5

78

100.0

カイ二乗値:10.454 危険率:0.107

出所:第 43 回衆議院議員総選挙についての意識調査

表 6 は、カイ二乗検定において有意でない結果が出たため分析

結果の解釈は省略する。

表 7 月の所得が 50 万円以上の層のクロス表分析

生活満足度

政治満足度

計(%) とても満足 やや満足 やや不満 とても不

とても満足 0.0 50.0 50.0 0 .0 2

100.0

やや満足 2.1 25.5 55.3 17.0 47

100.0

やや不満 0.0 0.0 50.0 50.0 14

100.0

とても不満 0.0 0.0 0.0 100.0 3

100.0

計(%) 1

1.5

13

19.7

34

51.5

18

27.3

66

100.0

カイ二乗値:18.008 危険率:0.035

出所:第 43 回衆議院議員総選挙についての意識調査

表 7 において、生活満足度をとても満足と回答し、政治満足度

をとても満足・やや満足と回答した率は、それぞれ 0%・50%とい

う結果となった。政治満足度をやや不満と回答した率は、50%であ

った。次に、生活満足度をやや満足と回答し、政治満足度をとて

も満足・やや満足と回答した率は、それぞれ 2.1%・25.5%という

結果であった。一方、政治満足度をやや不満・とても不満と回答

した率は、それぞれ 55.3%・17.0%という結果になった。続いて、

生活満足度をやや不満と回答し、政治満足度をやや満足・やや不

満・とても不満と回答した率は、それぞれ 0%、50.0%・50.0%と

いう結果であった。生活満足度をとても不満と回答し、政治満足

47

度をとても不満と回答した率はなんと 100%という結果であった。

どの所得階層でも生活満足度が低いと、政治満足度も低下する

という傾向にあった。この傾向は生活満足度をとても不満と回答

した人に顕著である。逆に、生活満足度が高ければ、政治満足度

も高くなるという仮説は傾向として認められるものの、低い生活

満足度が低い政治満足度を導くという仮説に比べるとその傾向は

小さい。唯一、月の所得が 50 万円以上の層では生活満足度をとて

も満足と回答した人の亓割が政治満足度をやや満足と回答してい

るものの、回答者の実数が 1 という結果のため、論証としては弱

い。生活満足度をとても満足・やや満足と回答し、政治満足度を

とても満足と回答した人は、表 5 をのぞいて、どれも 10%を上回

ることはなかった。生活満足度にかかわらず、政治満足度をやや

満足と回答した率は、表 7 を除き、回答者の二割から三割程度に

とどまった。

このことから、所得と生活満足度と政治満足度との間には関連

はないと推定できる。また、全体として、生活満足度の高低がそ

のまま政治満足度の高低に影響する傾向にある。しかし、生活満

足度をとても満足・やや満足と回答しても、政治満足度をやや不

満・とても不満と回答する割合が全体として高い。

5. まとめ 本稿では、生活満足度が高ければ、政治満足度も高くなるとい

う仮説を検証してきた。確かに、高い生活満足度が高い政治満足

度につながるという傾向は現れた。しかし、低い生活満足度が低

い政治満足度をもたらすという傾向に比べ、その傾向の度合は顕

著ではない。

最も所得の低い層と、最も所得の高い層の双方において、生活

満足度をとても満足と回答し、政治満足度をとても満足・やや満

足と回答した割合は、前者が 33.3%、後者は 50.0%であった。し

かし、後者の調査人数は計 66 人であり、必ずしも適当とはいえな

い。また、生活満足度をやや満足と回答し、政治満足度をとても

満足・やや満足と回答した率は、前者が 22.4%、後者が 27.6%で

あった。この数値から所得の多寡が生活満足度の高さに影響を与

えるとはいえないことがわかる。どの所得層でもグラフの傾向は

おおむね一致しており、生活満足度の高低は所得以外の要因に影

響されると推測できる。例として、治安の良さや将来に対する不

安などが挙げられるだろう。

生活満足度の高さが、高い政治満足度につながらないという結

果は、有権者が政治に満足感を抱くには日々の生活が満たされる

ことのみでは不十分であることを示していると言えるだろう。し

かし、低い生活満足度では低い政治満足度を示す傾向は明らかで

あり、高い生活満足度が政治満足度を満たす一要因になっている

ものと考えられる。

今後の課題として、上記の生活満足度を規定する他の要因の検

討は何であるか、生活満足度が高い有権者が政治に満足感を抱く

要因は何であるかを検討していきたい。

48

(1) 回答は多い順に 54.6%、48.7%、37.2%であった(回答は複数

回答)。

(2) ここで利用したデータは、明推協データである。「明推協選挙

後調査(2003)」は、全国明るい選挙推進協会が実施した世

論調査である。それをレヴァイアサン・データ・バンク(LDB、

木鐸社)より同志社大学が購入したものを使用した。同志社

大学・法学部の西澤由隆先生のご指導と便宜により利用がで

きた。それぞれのデータを公開・寄託され、利用でるように

してくださった先生方に感謝いたします。

(3) 両変数ともに「わからない」を欠損値として扱った。

(4) 所得は、安定した月の収入と定義する。そのため、「収入不定」

と「わからない」を同変数の欠損値として扱った。

<補遺>

◇分析に利用した作業定義

・生活満足度

Q23

生活にどの程度満足しているか。

1.大いに満足している 2.だいたい満足している 3.やや不満足で

ある 4.大いに不満足である 5.わからない

(5.わからないを欠損値として定義)

・政治満足度

Q24

政治にどの程度満足しているか。

1.大いに満足している 2.だいたい満足している 3.やや不満足で

ある 4.大いに不満足である 5.わからない

(5.わからないを欠損値として定義)

・所得

F4-2

月の収入はいくらか。

1.10 万円未満 2.10 万円以上 20 万円未満 3.20 万円以上 30 万円

未満 4.30 万円以上 40 万円未満 5.40 万円以上 50 万円未満

6.50 万円以上 7.収入不定 8.わからない

(7.収入不定 8.わからないを欠損値として定義)

「明推協選挙後調査(2003)」より作成

<参考文献>

・明るい選挙推進協会編 2011 http://www.akaruisenkyo.or.jp/

wp/wp-content/uploads/2011/04/22saninnsennchousagaiyo

u.pdf. 2011 年 12 月 3 日参照.

・蒲島郁夫 1988.『政治参加』東京大学出版会.

・三宅一郎 1989.『投票行動』東京大学出版会.

3 年生論文

50

1. はじめに 近年のフィリピンでは、市民によるデモンストレーションが政

治に影響を与えることが多くなってきている。1970 年代から続い

ていたマルコス大統領による権威主義体制を崩壊させたきっかけ

は、市民によるデモンストレーションであった。そして、1986 年

に民主化された後は、市民によるデモンストレーションが頻繁に

起こっているが、特に 2000 年以後はその傾向が顕著になってきて

いる。

その例の1つには、2001 年の選挙によらない政権交代のきっか

けとなった、市民によるデモンストレーションがある。就任以来、

多くのスキャンダルに囲まれたエストラーダに対し、議会では弾

劾裁判が開かれた。しかしながら上院での対立により裁判は進展

しなくなった。これをうけて大統領辞任を要求する市民の大規模

なデモンストレーションが始まり、最終的にエストラーダは大統

領職を追放された(粕谷 2007)。この市民のデモンストレーション

がきっかけとなった、選挙によらない政権交代の一連の流れは「ピ

ープルパワー2」と呼ばれる。

では、モンストレーションによって民主主義を取り戻したフィ

リピンの市民が、その獲得した民主主義を否定するかのように再

びデモンストレーションに参加するのはなぜだろうか。「ピープル

パワー2」では選挙によらない政権交代までおこっているのである。

近年のフィリピンの政治において、これだけの影響を与えている

デモンストレーション。このデモンストレーションへの市民の参

加が、何によって規定されているかを確認することは、フィリピ

ンの政治を理解する上で重要なことだと私は考える。

そこで、本稿では「フィリピンにおいて市民のデモンストレー

ションへの参加を規定する要因はなにか」というリサーチクエス

チョンを設定する。

9 フィリピンの市民が

デモンストレーショ

ンに参加するのはな

ぜか 池田 洋

51

2. 仮説 上記のリサーチクエスチョンに対し、フィリピンにおける「民

主主義の定着の遅れ」が1つの答えとして考えられる。すでに触

れたように、デモンストレーションは民主主義を不安定にしてい

る要因である。民主化後 15 年を経ての「ピープルパワー2」や、

その後の街頭集会など、政治的に不安定な状況が続いた。これは

市民の間で民主主義が定着していないからではないだろうか。

また、フィリピンでは「汚職の蔓延、政治犯や一般犯罪者に対

する警官の拷問、治安維持軍や一部のイスラム勢力による人権侵

害が残っているため」(亓十嵐 2004 p107)マルコス大統領による

権威主義体制が崩壊したあとでも市民的自由が阻害されている現

実がある。このような状況から、汚職の認識や警察不信、人権侵

害等の要因による政治的信頼の低下がデモンストレーションへの

参加を規定しているのではないかと考えられる。

これらの要因を以下のように個別に確認する。

(1) 民主主義に対する信頼

フィリピンにおける民主主義の定着の遅れに関しての先行研究

がある。粕谷(2007)はフィリピンにおいて「超憲法的な政権交

代を容認する態度」が深刻な問題の一つであるとした。そしてそ

の態度が実際に見られるかどうかとその要因を、慶應義塾大学 21

世紀 COE プログラムが 2005 年に行ったフィリピンでのサーベイ

調査のデータを用いて分析した。その結果、「弾劾裁判や選挙など

の憲法に規定された手続きではなく、街頭示威行動によって大統

領を交代させることは、その大統領が国を効果的に統治できない

状態であれば正当化できる」という意見に対し 57%の回答者が「強

く同意」「同意」と答えた。半数以上の人々が、非民主的な政権交

代を容認するという結果を得られたが、この調査はフィリピンの

みであったので他国との比較ができなかった。そのためこの結果

が特徴的であるかどうかは判断できなかった。

そこで、他国との比較をするために world values survey(世界価

値観調査)のデータを利用して分析をおこなった。「民主主義は秩序

を維持するのに有効ではない」という文言に対しての回答結果が

表1である。フィリピンは「強く同意」「同意」の割合が 49.9%と、

パキスタンを除いたどのアジア諸国よりも高かった。世界的に比

較しても、有効な結果を得られた 68 か国中 11 番目に高く、旧共

産圏であるロシアやポーランドといった東欧諸国以外でフィリピ

ンより「強く同意」「同意」との回答が多かった国はパキスタンと

ブラジルのみであった。この結果からもやはり、フィリピンでは

民主主義が定着していないと解釈することができる。

52

(2) 汚職の認識

私がフィリピンで生活していた際に、現地の人々と政治に関す

る話をすると、「汚職」という単語を彼らから聞くことが多かった

ように思う。なぜ彼らは政治の話になれば「汚職」を連想するの

であろうか。

戦後、経済的に急成長を遂げた東南アジアの中で、フィリピン

での経済成長は他国に比べて遅れをとっていた。この経済成長の

停滞の原因の1つとして、オリガ―キーやクローニーズムが悪い

レントの温床となったことが考えられる。1 この悪いレントを汚

職と認識するのであれば、市民は汚職に対して強い嫌悪感を抱く

のではないだろうか。

また、アロヨ前大統領の就任のきっかけは、上記でも触れたよ

うにエストラーダ元大統領の不正献金疑惑による退陣であった。

ところが再選を目指した2004年の大統領選挙での得票率操作疑惑

をはじめ、一族の収賄疑惑など様々な嫌疑がかかり、1986 年の民

主化後では最低水準となるまでに支持率は低下した。そして、2010

年の大統領選挙では、ベニグノ・アキノ候補が「汚職撲滅」を主

とした選挙公約として選挙戦を戦い当選した。これらのことから、

市民の間では汚職に対する不満が強いのではないかと考えられる。

(3) 公権力への信頼

他国に比べてフィリピンでは政党が政治的に重要な役割を担っ

ていない。大統領選挙前になると、有力な大統領候補の数だけ政

党が組織され、選挙が終わると利益誘導のために政権党と連立を

組むといった状況が続いている。このようなことから自らの利益

を反映し、一貫した政策を持つ政党が存在しないことが、市民の

不満になると考えられる。

表1.民主主義は秩序を維持するのに有効ではないかどうかの認識

強く同意 同意 同意しない 強く同意しな

フィリピン 9.5 40.4 42.7 7.4 100.0

日本 1.2 19.9 65.7 13.1 100.0

韓国 1.7 21.3 62.5 14.4 100.0

台湾 1.8 31.8 58.8 7.5 100.0

インド 10.5 30.4 48.7 10.3 100.0

インドネシア 3.6 19.1 64.2 13.1 100.0

イラン 7.1 24.4 47.2 21.3 100.0

ブラジル 25.1 31.5 20.2 23.3 100.0

パキスタン 12.3 40.7 40.7 6.3 100.0

出所:world values survey

53

また、実際に半年間フィリピンで生活をした私にとって警察の

存在が非常に印象的であった。現地の私の友人は、フィリピンの

警察はお金さえ払えば罪を見逃してくれるといった、小遣い稼ぎ

に職権を乱用することもあると言う。公権力に対して汚職のイメ

ージが非常に強いフィリピンでも、とくに自分に身近で経験的な

警察が具体的な汚職の場として市民に認識されるのではないか。

その結果として警察に対する信頼の程度は低いのではないだろう

か。

(1)(2)(3)より、フィリピンで市民がデモに参加する理由として

「民主主義に対して否定的な意見を持つフィリピンの市民が、汚

職や政党、警察に対する蓄積された不信・不満をデモへの参加と

いうかたちで表わす」という仮説がたてられる。

図1.デモ参加への因果メカニズム

3. 分析手法 前節での仮説を実証するため、次のような手法で分析をおこな

う。

3-1. 使用するデータ

分析に利用するデータは world values survey(世界価値観調査)

が 1981 年 か ら 2008 年 に 行 っ た 「 WVS FIVE WAVE

AGGREGATED FILE 1981-2005」の調査結果である。2

3-2. 作業定義

ここで定義する変数は、①汚職の認識②公権力への信頼③民主

主義への信頼④汚職の認識と民主主義への信頼の交互作用項⑤公

権力への信頼と民主主義への信頼の交互作用項⑥デモ参加経験で

ある。ここで特に重要な点は④汚職の認識と民主主義への信頼の

交互作用項と、⑤公権力への信頼と民主主義への信頼の交互作用

項である。交互作用項とは、複数の変数を掛け合わせた変数であ

る。その変数を用いることにより複数変数間でそれらの変数が同

時にはたらいている際の効果を確認できるようになるのである。 3

① 汚職の認識

公務員の汚職を市民がどれだけ認識しているかを「0」「0.25」

「0.75」「1」の4段階で表わす。

② 公権力への信頼

公権力の主要なアクターである政府・政党・警察への信頼

をそれぞれ「0」「0.25」「0.75」「1」と表す。これらの不信・

汚職の認識

公権力の不信

民主主義への不信 デモ参加

54

不満は因果関係として想定するのは難しく、おそらく互いに

強い関係があるように思われる。そのために分析の段階で互

いに効果を打ち消すことを避けるために3つの変数を足し合

わせて新たな変数を作成する。

③ 民主主義への信頼

民主主義は秩序を維持するのに有効であるかどう認識して

いるかを表す。有効でないを「0」、有効であるを「1」と表す。

④ 汚職の認識と民主主義への信頼の交互作用項

汚職の認識と民主主義が同時にはたらいているかどうかを

表す。上記のように定義した①と③を掛け合わせる。

⑤ 公権力への信頼と民主主義への信頼の交互作用項

公権力への信頼と民主主義への信頼が同時にはたらいてい

るかどうかを表す。④と同様に②と③を掛け合わせる。

⑥ デモへの参加経験

過去のデモへの参加経験を、経験無しを「0」、経験したか

もしれないを「0.5」、経験ありを「1」として表す。

3-3. 分析手順

以上のように作業定義したデータを利用して重回帰分析をおこ

なう。まずは①汚職の認識と②公権力への信頼が③民主主義への

信頼に対して与える影響を確認する。そのために③民主主義への

信頼を従属変数、①汚職の認識、②公権力への信頼、そしてコン

トロール変数である学歴、年齢を独立変数とした重回帰分析をお

こなう。

次に、①~⑤の各変数が⑥デモ参加経験に与える影響を確認す

る。ここでは⑥デモ参加経験を従属変数、①~③、④汚職の認識

と民主主義への信頼の交互作用項と、⑤公権力への信頼と民主主

義への信頼の交互作用項、コントロール変数である学歴、年齢を

独立変数とした重回帰分析をおこなう。4

4. 分析 以上のような分析手法で得られた結果が表 2と表 3である。5 ま

ず各変数が「民主主義の信頼」に与える影響である表 2 に関して

だが、予想に反してあまり有意な結果が得られなかった。まず各

変数の影響だが、コントロール変数として投入した「教育程度」

以外に有意な結果は得られなかった。さらには調整済み R 二乗値

が 0.010 であることから、このモデルの説明力が極めて低いこと

がわかる。

55

表2 各変数が民主主義への信頼に与える影響

ベータ 危険率

公権力への信頼 -.040 .188

汚職の認識 .006 .849

教育程度 .107 .001

年齢 .012 .698

調整済みR二乗値:0.010

次に各変数がデモ参加経験に与える影響を表 3 で確認してみる。

表 3 で「汚職の認識」・「公権力への信頼」が「民主主義の信頼」

に対して影響を与えてなかったことがわかったために予想できた

ことであるが、「汚職の認識」・「公権力への信頼」と「民主主義へ

の信頼」の交互作用項は有意な結果を得られなかった。むしろ全

く影響がないと言えるだろう。また、「民主主義への信頼」それ自

体としてもデモ参加経験に対しては全く影響がないことがわかっ

た。唯一効果を確認できたのは「公権力への信頼」であった。ベ

ータがマイナスの値をとっていることから、公権力への信頼が低

いほどデモ参加経験が高くなるこということがわかった。

表3 各変数がデモ参加経験に与える影響

ベータ 危険率

公権力への信頼×民主主義への信頼 .007 .919

公権力への信頼 -.083 .044

民主主義への信頼 -.003 .966

汚職の認識×民主主義への信頼 .001 .983

汚職の認識 -.021 .608

教育程度 .185 .000

年齢 .024 .436

調整済みR二乗値:0.038

56

5. おわりに 本稿では「フィリピンにおいて市民のデモンストレーションへ

の参加を規定する要因はなにか」というリサーチクエスチョンを

設定し、「民主主義に対して否定的な意見を持つフィリピンの市民

は、汚職や政党、警察に対する蓄積された不信・不満をデモへの

参加というかたちで表わす」という仮説を立て、分析をおこなっ

た。その結果、仮説とは異なり民主主義への信頼がデモ参加経験

に与える影響は確認できなかった。唯一確認できたのは、公権力

の信頼がデモ参加に与える負の影響であった。

また、汚職の認識や公権力への信頼が民主主義への信頼に与え

る影響も確認することはできなかったが、これらの要因が影響を

与えないのであれば、民主主義は有効でないと回答した約 50%の

市民は何によってその意思を決定しているのであろうか。本稿で

の分析ではより政治的な変数を独立変数としたが、コントロール

変数として投入した教育程度が非常に強い影響を示していたこと

からもわかるように、人としてより根本的な社会的属性が影響し

ているのかもしれない。この点はより、フィリピンの市民・社会

を理解しなければならない問題であろう。

そして驚いたことに本稿の執筆中に、アロヨ前大統領が選挙妨

害容疑で逮捕された。不正蓄財で 2007 年に有罪判決を受けたエス

トラーダ元大統領に続き、大統領経験者が2代続けて逮捕された

のである。このニュースを知ったときに、本稿の1つのポイント

として「汚職」を選んだことは間違いではなかったのだと思うこ

とができた。本稿において「汚職」についても注目したが、有意

な結果を得ることができなかったことからもわかるように、「汚

職」の実態をデータだけで確認するというのは難しいことである

と実感できた。大雑把に「汚職」といっても収賄・贈賄もあれば

アロヨ全大統領の容疑である選挙妨害や2004年の大統領選の得票

水増しや、政府の基金の不正使用など様々である。「汚職」という

意味でさえ曖昧であるのに、さらにはその違法性を考慮するとな

かなか実態をつかむのは困難であると思われる。確かにその困難

性はあるにしても、大統領経験者が 2 代続けて逮捕されたという

まさに今、その困難性を乗り越えてそのパズルを解く必要がある

のではないだろうか。

(1) 尐数の経済エリートによる支配体制をオリガ―キー、権威

主義体制下での政治関係者の縁故主義をクローニーズムとい

う。

(2) world Values Survey の公開データ「WVS FIVE WAVE

AGGREGATED FILE 1981-2005」を利用した。同志社大

学・法学部の西澤由隆先生のご指導と便宜により利用ができ

た。それぞれのデータを公開・寄託され利用できるようにし

てくださったすべての方に感謝いたします。

57

(3) 例えば性別と学歴の交互作用項を定義してみる。男性を「0」

女性を「1」とし、高卒以下を「0」大卒以上を「1」と定義

する。これらの変数を掛け合わせると、男性・高卒以下、男

性・大卒以上、女性・高卒以下、は「0」となり、女性・大

卒以上のみが「1」となる。このことにより、女性・大卒以

上かそれ以外かどうかのダミー変数を作成することができる

のである。

(4) 教育程度は程度が低い順に「0」「0.5」「1」と表した。

(5) 危険率とは、説明変数が従属変数に与える効果が偶然であ

る確率を示し、その値がより 0 に近いほどより有意なモデル

であることを意味する。ベータは標準偏回帰係数とも呼ばれ、

各変数が従属変数に及ぼす影響の大きさと向きを示している。

そして調整済み R2 乗値はそのモデルの説明力を表す値で、

1に近づくほど説明力が上がる。

〈補遺〉

質問票

・デモへの参加経験

Now I'd like you to look at this card. I'm going to read out

some different forms of political action that people can

take, and I'd like you to tell me, for each one, whether you

have actually done any of these things, whether you might do

it or would never, under any circumstances, do it.

V120 Attending lawful demonstrations 1 「Have Done」2

「Might Do」 3「Would Never Do」9「DK」

・公権力への信頼

I am going to name a number of organizations. For each one,

could you tell me how much confidence you have in them:

is it a great deal of confidence, quite a lot of confidence, not

very much confidence or none at all?

V141 The police

V142 The government in[WASHINGTON/ YOUR CAPITAL]

V143 Political parties

1 「A Great Deal」2「Quite A Lot」 3「Not Very Much」 4

「None At All」 9「DK」

・民主主義は有効であるかどうか

I'm going to read off some things that people sometimes say

about a democratic political system. Could you please tell

me if you agree strongly, agree, disagree or disagree

strongly, after I read each one of them?

V162. Democracies aren't good at maintaining order 1

「Agree Strongly」 2「Agree」 3「Disagree」4「disagree

Strongly」9「DK」

・汚職の認識

V213. How widespread do you think bribe taking and

corruption is in this country?

1. Almost no public officials are engaged in it

2. A few public officials are engaged in it

58

3. Most public officials are engaged in it

4. Almost all public officials are engaged in it

9. DK

・年齢

V216. This means you are __ __ years old.

〈参考文献〉

・亓十嵐誠一 2004. 『フィリピンの民主化と市民社会―移行・

定着・発展の政治力学―』 成文堂.

・粕谷祐子編 2010. 『アジアにおける大統領制の比較政治学―

憲法構造と政党政治からのアプローチ―』ミネルヴァ書房.

・川中豪編 2005. 『ポスト・エドサ期のフィリピン』 アジア経

済研究所.

・小林良彰・富田広士・粕谷祐子編 2007. 『市民社会の比較政

治学』慶應義塾大学出版会/

・清水一史・田村慶子・横山豪志編 2011. 『東南アジア現代政

治入門』第三章 ミネルヴァ書房.

59

10 2005 年衆議院選挙

の投票率上昇は何

によってもたらさ

れたのか 誰がマスメディアの影響を受けたのか

岡本 怜子

1. はじめに 近年、選挙が行われるたびに投票率の低下が問題となって取り

あげられる。それは、国政選挙、地方選挙問わずである。投票率

低下の原因は、若者の投票不参加や社会全体の政治に対する無関

心化が決まり文句となって挙げられる。そして投票率低下の傾向

は、年々悪化しているようにも報道される。しかしながらこの流

れとは反対に、投票率が大きく上昇した選挙がある。それが 2005

年第 44 回衆議院議員総選挙である。

2005 年に実施された第 44 回衆議院議員総選挙は、多くの人の

記憶に残っている選挙の 1 つではないだろうか。私がそう考える

理由は、この選挙の投票率の高さにある。2005 年総選挙の投票率

は 67.5%で、2003 年総選挙の投票率 59.8%を約 8 ポイント上回っ

た。近年投票率の低下が問題となっている中で、投票率が 8 ポイ

ント上昇したというのは特異である。新憲法下最初の選挙である

1947 年第 23 回衆議院総選挙以降、投票率が前回選挙より 8 ポイ

ントも上昇した選挙は一度もない。自民党から民主党へ政権交代

が起こった 2009 年第 45 回衆議院議員総選挙は記憶に新しいが、

この選挙でさえ投票率は 69.2%と前回の選挙からわずか 2 ポイン

トの上昇であった。これらのことから、2005 年総選挙の投票率 8

ポイント上昇がいかに異例だったかが推測できる (図 1) 。

では、2005 年総選挙ではなぜ投票率が上昇したのだろうか。「投

票率が上昇する」ということは、言い換えれば「前回の選挙より

も多くの人が投票に行った」ことを意味する。つまり 2005 年総選

挙では、2003 年総選挙よりも多くの有権者が選挙に対して何らか

の関心をもち、投票参加したと言える。政治に無関心だった有権

者が関心を持つようになるには、政治に関する情報を手に入れる

ことが必要である。政治に関する情報を有権者が手にする最も身

近な手段として、マスメディアが考えられる。つまり私は、2005

年総選挙の投票率上昇はマスメディアと何か関わりがあるのでは

ないかと考える。

そこで、本稿のリサーチクエスチョンを「なぜ 2005 年総選挙の

投票率は上昇したのか」と設定し、投票率上昇 8 ポイントの正体

60

の一つを探ることを試みる。

2. 2005 年第 44 回衆議院議員総選挙

本稿で扱う、2005 年衆議院議員総選挙の背景とその特徴につい

て紹介する。

2005 年総選挙は「郵政民営化の是非」が論点に据えられ、「郵政

選挙」と言われた。結果的には、衆議院 480 議席の内 296 議席を

自民党が獲得し、単独過半数を得る圧勝となった。国民は、選挙

以前は郵政民営化に特別な関心があったわけではなく、どちらか

というと年金や税制の方に関心を向けていた。読売新聞が行った

世論調査によると、政府が取り組むべき重要課題(複数回答可)に対

して、「郵政民営化」と答えたのは 25.7%で 10 項目中 8 位。1 位

は「年金・福祉制度改革」で 76.1%、「景気・雇用対策」、「治安・

防災」、「財政改革」の順に続いた。1

このように郵政民営化は、選挙以前は国民にとって関心の低い

トピックであった。しかし、2005 年総選挙といえば「郵政選挙」

と呼ばれるほど郵政民営化の印象が強い。私は、国民の関心が低

い郵政民営化を掲げながら圧勝した小泉の選挙戦略が、投票率の

上昇と何か関係があるのではないかと考える。

では、小泉の選挙戦略とはどのようなものだったのだろうか。

2005 年総選挙が多くの国民の記憶に残っている選挙であることも

示すように、小泉はメディアをとても巧みに利用して選挙活動を

行った。小泉のメディア戦略の巧妙さは、有権者の関心を年金問

題や税制から郵政民営化へと上手にシフトさせたことに表れてい

る。

小泉はどのようにして、有権者の関心を郵政民営化に向けたの

か。そのキーパーソンは竹中郵政民営化担当大臣である。竹中は

有権者に郵政民営化について関心をもたせるために、広告会社「ス

リード」と契約を結んだ。スリード社は、「B 層」と呼ばれるター

ゲットに向けて宣伝を行う事が最も効果的だと考え、小泉の選挙

活動を計画した。B 層とは、「小泉内閣の支持基盤である主婦や若

年層、シルバー世代で、具体的なことはよくわからないが小泉総

理や閣僚のキャラクターを支持する IQ の比較的低い層」とシルバ

ー社が定義づけている。それに対して A 層も存在し、その定義は

「財界勝ち組企業や大学教授、マスコミ、都市部のホワイトカラ

ーなど比較的 IQ の高い層」となっている。この IQ による区分は

あくまでもスリード社によるものであるが、両者の間に職業階層

的な差があることは事実である。つまりスリード社は、B 層という

特定の有権者に向けてメディアを発信することで、効率よく有権

者に郵政民営化に関心をもたせることができると考えたのである。

よって、小泉は印象に残る短いフレーズを使う「ワンフレーズポ

リティクス」を行うことで、B 層に受けるようなメディア戦略を実

施した。

61

3. 先行研究 本稿は、「有権者が選挙に関心をもったから投票参加した」とい

うことを前提にしている。そこで、政治関心と投票参加に関する

先行研究を紹介する。

有権者が政治参加するか否かの原因はいくつか挙げられる。蒲

島(1988)の研究に、政治的関与がどの程度政治参加のレベルと関

連しているか、その相対的影響力を推計しているものがある。(表

1) この研究は、投票参加を目的変数に、政治的関与を説明変数

にしたモデルを重回帰分析したものである。政治的関与には因子

分析により得られた、政治的関心・政治的義務感・政治的有力感・

政治的信頼感・政治参加のコスト感覚・地域愛着度・政党支持の

強度が用いられている。この分析の結果、政治的関心が高ければ

高いほど有権者はより投票参加するという結果が得られた。つま

り、政治に関心のある有権者は投票参加するのだから、何が有権

者に政治関心をもたらしたかが判明すれば、投票率上昇の原因も

わかるということである。

4. 仮説 2005 年総選挙の特徴、また、先行研究を踏まえて、本稿の仮説

を「2005 年総選挙は、有権者がマスメディアに影響を受けて選挙

に関心をもったことで投票率が上昇した」と設定する。

この仮説を設定する理由は、2005 年総選挙における小泉の

メディア戦略が、それまでの選挙と大きく違うと考えるからで

ある。マスメディアはおそらく 2005 年以前の選挙でも選挙報

道を行い、有権者はマスメディアから情報を得ていただろう。

しかし私は、マスメディアが有権者に与える影響が 2005 年総

選挙では顕著に大きかったのではないかと推測する。「テレビ

の特性を熟知した劇場型の政治手法を通じて、高い内閣支持率

を誇ったのが小泉政治の真骨頂だった」ともあるように、2005

年総選挙はマスメディアが郵政民営化についてしきりに取り上

げ、また「刺客」・「小泉劇場」のような有権者の印象に残る

用語を用いることで、国民の関心を集めていたと言える。(草

野厚、2006、p.117)

マスメディアと一口に言っても、新聞・雑誌・テレビ・ラジオ・

インターネットなど様々な媒体がある。この中でも、本稿では「テ

レビ」に注目する。なぜなら、小泉のメディア戦略である「劇場

型ワンフレーズポリティクス」はテレビに最も効果的だと考える

からである。

小泉のメディア戦略はどのようにテレビの特性と一致していた

のだろうか。テレビの特性として次の 5 つの点が挙げられる。(蒲

島・竹下・芹川、2007、p.211 l.6~)

①1 つの事柄が視聴率を取れるとなれば、各局ともそれに話題が

集中し洪水報道化する。

②時間的制約があるため、善玉悪玉の二項対立で番組を作る傾

向がある。

③視聴者に提供される情報はカメラがとらえた映像に限られる

62

ため、制作者の意図に誘導しやすい。

④テレビは映像が命であるために、映像のない事柄はニュース

になりにくい。

⑤放送は一定の時間内に終わらさなければならない。

小泉は自らを「善」、郵政民営化に難色を示す者を「悪」と位置

付けて、党内対立を演出した。また、インパクトのある短いフレ

ーズを用いることで、番組の限りある放送時間を有効活用した。

「テレビキャスターは当時、『マニフェスト(政権公約)と言った瞬

間に視聴率が下がり、刺客が登場すると、とたんに視聴率が上が

る。視聴率がテレビのものさしで、視聴率競争をしている以上は、

どうしても刺客作戦を報じることになる』と語っていた」ともあ

るように、小泉のメディア戦略はきっちりとテレビの特性を捉え

ていたと言えるのではないだろうか。(蒲島・竹下・芹川、2007、

p.211 l.14 ~) よって私は、テレビが最も小泉の選挙活動を反映し、

有権者に影響を与えたのではないかと考える。そこで本稿では、

マスメディアの中でもテレビが有権者に与えた影響に着目して分

析を行う。

私の考える仮説は以下の通りである。

(1)マスメディア影響仮説;有権者は、マスメディアの選挙報道

に影響を受けて選挙に関心をもち、投票参加した。

(2)主婦層仮説;マスメディアの選挙報道は、有権者の中でも特

に「B 層」に分類される主婦層に影響を及ぼした。

ここで私が B 層を主婦層に限定したのは、B 層の中でも主婦

層が最もマスメディアの影響を受けやすいのではないかと考える

からである。小泉のワンフレーズポリティクスが取り上げられた

のはワイドショー的な要素を含むテレビ番組に多く、それらは主

に平日の昼間に放送されることが多い。このことから、日中に学

校に通う学生を対象から省いた。また、シルバー世代は日中にテ

レビを見る機会は多くても、実際に投票に行くというコスト感覚

を考えると、主婦の方が投票に行きやすいのではないかと考えた。

よって、本稿では B 層の中でも主婦層に着目し分析を行う。「B 層

が2005年総選挙においてメディア戦略に影響されたことによって

投票参加が促され投票率上昇が実現した」と推測できる。

そこで以上の事を図で表すと、次の様なモデル図を示すことが

できる。

職業(年齢) → マスメディアへの接触 →選挙活動への関心

5. 分析枞組み 5-1. 使用するデータ

前節で述べた仮説を検証するために、2005 年の衆議院議員総選

挙とそれ以前の選挙について調査を行っているサーベイ・データ

が必要である。また、そのサーベイ・データには有権者のメディ

ア接触量を計ることができる質問項目が含まれている必要がある。

これらの条件を満たすのは JESⅢである。2 2003 年と 2005 年の

63

事後調査を用いる。

5-2. 作業定義

(1)有権者のメディア接触量

有権者のメディア接触量は、「この中で今度の選挙について、選

挙期間中によく見聞きしたものはどれですか」という質問に対す

る回答によって定義する。この質問に対する回答の中でも、テレ

ビ番組への接触のみを対象とした。テレビ番組が放送される時間

帯に応じて、メディア接触量の変数を 4 つ作成した。具体的には、

平日の昼・平日の夜・休日の昼・休日の夜の 4 つである。

平日の昼は 5 番組(2005 年は「ワイドショー」が加わり 6 番組)、

平日夜は 5 番組を対象とする。

(2)職業

職業は、「あなたは通常どのような仕事をしていますか」という

質問への回答によって定義する。なお、回答されたそれぞれの職

業を、有職者、無職者・学生、専業主婦の 3 つに分類した。

6. 分析(表 2-表 5) メディア接触量と職業についてクロス表分析を行った。

まず平日の昼間の分析について職業別に比較する。(表 2、表 3)

すると平日の昼間にテレビを全く見ていないのは、2003 年 2005

年共に有職者が最も多く、無職者・学生が最も尐ないことがわか

る。一方で、平日の昼間に 2 番組以上見ているのは 2003 年 2005

年共に専業主婦が最も多い。このことから平日に昼間のメディア

接触量は、有職者や無職者・学生に比べて主婦の方が多いことが

分かる。

次に平日の昼間の分析について、2003年と 2005年を比較する。

(表 2、表 3) すると、有職者、無職者・学生については、2003

年より 2005 年の方がメディア接触量は増えているが、その量は 2、

3%の幅である。一方、主婦について見てみると平日の昼間に 1 番

組しかテレビを見なかった主婦が 2003年は 56.9%だったのに対し

て、2005 年では 46.3%と 10%減尐している。その分、2 番組視聴

した主婦が 2003年では 19.6%だったのに対し、2005年では 26.3%

と約 10%増えている。2番組視聴した人でも 2005年では 7%増え、

4 番組視聴した人でも 3%増えている。このことから平日の昼間に

関して、主婦が最もメディアの影響を受けたと推察する。

平日の夜の分析についても職業別に比較する。(表 4、表 5) す

ると平日の夜のテレビ接触量は平日の昼間とは対照的に、2003 年

2005 年ともに無職者・学生が最も接触量が尐なく、有職者が最も

多いという結果が見られた。また、平日の夜に 3 番組以上テレビ

を視聴しているのは、2003 年 2005 年ともに有職者が最も多い。

このことから平日の夜は、有職者が最もテレビ視聴していたこと

がわかる。

次に、平日の夜について 2003 年と 2005 年を比較する。(表 4、

表 5) すると全ての職業において、平日の夜に 2 番組以上テレビ

視聴した人は 2003 年よりも 2005 年の方が減尐している。

以上の点から、やはり昼間は専業主婦がメディア接触しやすく、

64

夜は仕事から帰ってきた有職者のメディア接触が多いということ

がわかる。そして、2003 年と 2005 年を比較してみても、小泉の

劇場型ワンフレーズポリティクスが取りあげられやすいワイドシ

ョーが放送される平日の昼間について、2005 年の主婦層が最もメ

ディア接触していることがわかった。

7. おわりに これらの結果から、まず、平日の昼間のメディア接触量は主婦

層が最も多いということが分かった。また、2003 年よりも 2005

年のほうが主婦の平日昼間のメディア接触が増えたということも

分かった。

つまり、小泉の行ったワンフレーズポリティクスがしきりに取

り上げられたワイドショーを、主婦層はよく視聴していたという

ことである。このことから、2005 年総選挙の投票率上昇の 1 つの

原因として主婦層がマスメディアに影響を受けたことが考えられ

る。

今日、投票率の低下が問題となっているが、有権者に最も身近

であるマスメディアの発信の方法によっては有権者が選挙に関心

をもち、投票率の低下を改善できるのではないかと考える。

(1) 読売新聞 2005 年 4 月 27 日 掲載

(2) ここで使用したデータは JESⅢである。JESⅢは、平成 14

~18 年度文部科学省研究費特別推進研究「21 世紀初頭の投票行

動の全国的・時系列的調査研究」に基づく「JESⅢ研究プロジェ

クト」(参加者・池田謙一:東京大学教授、小林良彰:慶應義塾

大学教授、平野浩:学習院大学教授)が行った研究結果である。

それをレヴァイアサン・データバンクより同志社大学が代表し

て利用申請したもの、西澤由隆先生のご指導と便宜の下、利用

させていただいた。この場を借りて、それぞれのデータを公

開・寄託され、利用できるようにしてくださった先生方に感謝

いたします。

65

<補遺>

0

10

20

30

40

50

60

70

80

36 37 38 39 40 41 42 43 44 45

投票

率(%

)

衆議院総選挙(回)

図1 投票率の推移

表 1 政治参加に対する政治的関与の影響

政治的関与 投票 選挙運動 地域・住民運動

政治的関心 6 19 24

政治的義務感 17 ― ―

政治的有力感 ― 9 9

政治的信頼 4 ― ―

政治参加のコスト感覚 45 ― ―

地域愛着度 9 ― 9

政党支持の強度 8 28 16

66

表 2 メディア接触量と職業のクロス表

(2003)

接触量 平日昼

0 1 2 3 4 5 合計(%) N

職 有職者 16.0 57.6 18.1 5.9 1.5 0.9 100.0 1208

業 無職者・学生 9.4 66.1 15.8 5.0 2.1 1.6 100.0 576

(%) 主婦 12.0 56.9 19.6 8.6 1.0 1.8 100.0 383

表 3 メディア接触量と職業のクロス表(2005)

接触量 平日昼

0 1 2 3 4 5 6 合計(%) N

職 有職者 18.4 50.7 18.4 6.9 3.1 1.4 1.0 100.0 858

業 無職者・学

生 7.5 61.2 18.8 6.0 3.0 1.5 2.1 100.0 335

(%) 主婦 11.0 46.3 26.3 8.9 4.3 0.7 2.5 100.0 281

カイ二乗値:44.631 危険率:.000

表 4 メディア接触量と職業のクロス表

(2003)

接触量 平日夜

0 1 2 3 4 5 合計(%) N

職 有職者 30.5 37.1 24.9 5.8 1.5 0.2 100.0 1208

業 無職者・学

生 48.6 26.6 19.8 3.5 1.2 0.3 100.0 576

(%) 主婦 34.2 33.4 26.4 4.4 1.0 0.5 100.0 383

カイ二乗値:60.259 危険率:.000

表 5 メディア接触量と職業のクロス表(2005)

接触量 平日夜

0 1 2 3 4 5 合計(%) N

職 有職者 33.3 34.8 24.2 4.9 1.7 0.9 100.0 858

業 無職者・学生 56.1 23.3 15.5 2.7 0.9 1.5 100.0 335

(%) 主婦 37.4 35.2 21.7 3.2 1.1 1.4 100.0 281

カイ二乗値:57.434

危険率:.000

分析に使用した質問文(2005 年事後調査のみ掲載)

・Q15 この中で今度の選挙について、選挙期間中によく見聞き

したものはどれですか。(M.A.)

1. NHK のニュース番組 2.小栗泉の「きょうの出来事」(日本テ

レビ系)3.笛吹雅子、近野宏明の「News プラス 1」(日本テレ

ビ系) 4.筑紫哲也、草野満代の「ニュース 23」(TBS 系) 5.

67

三雲孝江、池田裕行の「イブニング・ファイブ」(TBS 系) 6.

松本方哉、滝川クリステルの「ニュース JAPAN」(フジテレ

ビ系) 7.安藤裕子、木村太郎の「スーパーニュース」(フジテ

レビ系) 8.古館伊知郎の「報道ステーション」(テレビ朝日系)

9.小宮悦子の「スーパーJ チャンネル」(テレビ朝日系) 10.

小谷真生子の「ワールドビジネスサテライト」(テレビ東京系)

11.辛坊次郎の「ウェークアップ!ぷらす」(日本テレビ系)

12.徳光和夫の「ザ・サンデー」(日本テレビ系) 13.関口宏の

「サンデーモーニング」(TBS 系) 14.田丸美寿々の「報道特

集」(TBS 系) 15.黒丸祐治の「報道 2001」(フジテレビ系)16.

田原総一郎の「サンデープロジェクト」(テレビ朝日系) 17.

福留功男、久保純子の「ブロードキャスター」(TBS 系) 18.

午前や午後のワイドショー 19.その他のテレビ番組 20.ラ

ジオ番組 21.週刊誌 22.インターネットの政治ニュース

23.インターネットの候補者ページ 24.インターネットの政

治関係のホームページ(候補者以外のもの) 25.インターネ

ット上の政治関係の電子掲示板・電子会議・メーリングリス

ト 26.わからない 27.答えない

⇒この中からテレビ番組のみに着目し、テレビ番組を平日昼・平

日夜・休日昼・休日夜の 4 つに分類。

平日昼:1、3、5、7、9、18(2003 年の質問票には 18 がない

ため 5 項目)

平日昼:2、4、6、8、10

休日昼:11、12、13、15、16

休日夜:14、17

F5(2) 重ねての質問で申し訳ございませんが、あなたは通常、

どのような仕事をしていますか。仕事の内容を具体的にお聞

かせください。

1.農林漁業 2.商工サービス 3.自由業 4.管理職 5.専門技術

職 6.事務職

7.販売・サービス・労務職 8.その他の有職者 9.学生 10.専業

主婦 11.その他の無職

12.分類不能 13.わからない 14.答えない

⇒これらを、有職者、無職者・学生、専業主婦の 3 つに分類。

1~8 を有職者「1」、9・11 を無職者・学生「2」、10 を専業主

婦「3」として再コード

12、13、14 については欠損値として処理。

<参考文献>

・池田謙一 2007.『政治のリアリティと社会心理』木鐸社.

・蒲島郁夫 1988.『政治参加』東京大学出版会.

・蒲島郁夫、竹下俊郎、芹川洋一 2007.『メディアと政治』有斐

閣アルマ.

・草野厚 2002.『テレビ報道の正しい見方』PHP 新書.

・草野厚 2006.『テレビは政治を動かすか』NTT 出版.

・平野浩 2007.『変容する日本の社会と投票行動』木鐸社.

・三宅一郎 1989.『投票行動』東京大学出版会.

68

11 争点投票を可能とす

る要因は何か

ソーシャル・キャピタルの可能性 岡山 義信

1. はじめに 選挙において、有権者は合理的な判断をしているのか? 田中

(1998, 2000)によると、これに関連して問題となるのが「争点投

票」の有無であり、有権者が争点投票を行っているかどうかにつ

いては多くの論争があり、様々な実証研究においても決着がつい

ていない。争点投票がなかなかうまく実証的に確認されない理由

として争点投票そのものの困難性を指摘することが出来る。具体

的には、争点投票を行うには次の 3 条件が必要とされている。

(1)有権者が選挙時における政策争点を認知していること

(2)有権者にとってその政策争点が重要な意味をもっていること

(3)有権者にとってどの政党の争点上の立場が最も自分の立場に

近いかを理解していること

それらに加えて、争点になっている案件に対して政党が実際に

実現してくれるという有権者の信頼がなければそもそも争点投票

はおこらないだろう。

このように多くのハードルが設定されている争点投票をすべて

の有権者が行っているとは考えづらい。しかし、平野(2007, 7 章)

は 2005 年の衆議院総選挙の結果を用いて、実際に争点投票を行っ

ている有権者が存在することを確認した。平野(2007, 7 章)は、こ

の選挙で最大の争点であった「郵政民営化」に対する有権者の認

知と「郵政民営化」に対する政党立場の認知は、政党支持の投票

行動への影響を統制した上でも、有権者の投票行動に影響を与え

ていたことを見いだした。例えば、郵政民営化に反対する有権者

が、民主党を郵政民営化に反対している政党と認知した上で民主

党に投票していることなどである。

ではこのような多くのハードルを乗り越えて争点投票を行って

いる有権者とは、一体どのような有権者なのか? これが本研究

の問いである。争点投票を行うに至るまでのハードルを乗り越え

ることが出来る有権者と乗り越えることが出来ない有権者を分か

つ要因は何か。

上記の問いに対する一つの答えとして、本研究では「ソーシャ

ル・キャピタル(社会関係資本)に富んだネットワーク内の有権

者が争点投票を行う」という仮説を提示し、計量分析を用いて実

証的に示したい。パットナム(1993)は「地方政府のパフォーマンス

は何によって規定されるのか」という問いに対して、その地域の

「水平的ネットワーク、一般的信頼、互酬性の規範」としてのソ

ーシャル・キャピタル(社会関係資本)がその地域のパフォーマ

ンスを規定するとした。このようなソーシャル・キャピタルは争

69

点投票のハードルを緩和する効果があるのではないだろうか。

ここで想定しているメカニズムは「水平的なネットワーク」が

選挙の際に周りの有権者との率直な意見交換を可能とし、またそ

のような意見交換の中で政治的知識は増大してゆくというもので

ある。さらに「一般的な信頼」は争点投票の前提を整える。なぜ

なら、有権者が争点投票を行う上では、争点となっている政策案

を政党が本当に実行してくれるという信頼が必要条件となってい

ると考えられるからである。

以上のことを論じるために、本稿では以下の構成をとることと

する。第 2 節では、争点投票についての主な先行研究を紹介した

上で本研究のリサーチ・クエスチョンを提示する。第 3 節では、

ソーシャル・キャピタルについての主な先行研究を紹介して、本

研究の仮説を提示する。第 4 節では、本研究における分析枞組み

を提示し、変数の操作化を行う。ここでは、本研究の仮説を実証

するために、具体的に確認することになる 4 つの作業仮説の提示

も行う。第 5 節では、第 4 節に基づく分析結果を提示し、その解

釈を行う。そして第 6 節では、本研究全体を通じての考察を述べ

る。

2. 争点投票とリサーチ・クエスチョン 本節では、争点投票(issue voting)とは何かについて主要な先行

研究を紹介しながら述べたい。またその際に争点投票研究におけ

る主要な論点を紹介した上で本研究のリサーチ・クエスチョンの

提示を行う。また、争点投票の研究史についてレビューしたもの

として既にいくつかの詳しい文献が存在する(田中 1998・谷口

2005・小林 2000)。ここでは、主にそれらに依拠しながら紹介し

ていきたい。

「争点投票をめぐる論争(issue voting controversy)は、アメ

リカの投票行動研究史上最大の論争であった」と田中は述べてい

るが、その論点は有権者の合理性の有無についてであった。1 争

点投票を定義すれば、それは選挙において「有権者が候補者(や

政党)の政策争点上の立場を認知し、自己の争点態度に近い方の

候補者に投票する」ことである。2 この定義からもわかるように

争点投票を行うにはかなりハードルの高い有権者の合理性が求め

られる。実際、いわゆる「ミシガン・モデル」においても当初は、

有権者の投票行動に影響を与える要因として争点態度よりも政党

帰属意識と候補者評価の影響力の比較的な大きさを示していた。

しかし、このようなミシガン・モデルはいくつかの反論を受け、

一定の争点態度の効果を認めるように修正されていくこととなっ

た。具体的な反論内容をあげれば、時代状況の問題についての指

摘である。争点投票とはそもそも争点が顕在化していないと起こ

りにくく、ミシガン学派が調査した 1950 年代というのは 60 年代

以降のような大きな争点はなかった。60 年代になるとベトナム戦

争など争点が顕在化されたことにより、有権者の争点態度が投票

行動に影響を与えていることが実証された。

一方、日本の投票行動研究では、有権者の争点態度の投票行動

への影響は、政党支持や候補者評価の影響に比べて低いという結

70

果を示すものが多かった。谷口(2005)は、日本での争点態度に

よる投票行動への影響が相対的に低かった理由として、選挙制度

の影響と自民党長期政権の影響を挙げている。戦後長らく続いた

中選挙区制では同政党(多くは自民党)の候補者同士が同じ選挙

区で争うため、個人投票の影響がどうしても強くなってしまうと

いうことである。また、自民党が長期政権を続けてきたため、自

民党以外の政党の政策能力はどうしても低く評価されてしまって

いた。そのために、特定の争点が顕在化する機会は尐なかったの

である。

しかし、このような争点投票を阻害する要因を緩和する選挙制

度改革が 1994 年に行われた。中選挙区制から小選挙区比例代表並

立制への移行である。この選挙制度改革後初めて 1996 年に衆議院

議員総選挙が行われ、徐々に有権者の中に争点投票を行うことを

可能にする条件が整う方向に動いていった。そのような制度面の

変化の中、2005 年の衆議院議員総選挙は、小泉首相のもとで「郵

政選挙」が謳われ、ほとんどの有権者が「郵政民営化」という争

点を認知したもとで行われた選挙となった。

このような格好の条件のもとで争点投票を確認しようとしたも

のが、「はじめに」でも紹介した平野(2007, 7 章)である。平野は

以下のようにして争点投票有権者の存在を実証した。まず、この

選挙で最大の争点であった「郵政民営化」についての有権者の態

度(賛成・反対)と「郵政民営化」についてのその有権者が評価

する政党の立場(賛成・反対・わからない)とを組み合わせた変

数を独立変数にとり、自民党投票と民主党投票を従属変数にして

回帰分析を行った。その結果は政党支持の投票行動への影響を統

制した上でも、独立変数が従属変数に統計的に有意な効果を持っ

ていた。また、「憲法・安全保障問題」争点についても同様の分析

を行い一定の争点態度の効果を実証した。

しかし、当然のことながら全ての有権者が争点投票を行ったの

ではなく、この選挙において争点投票を行っていない有権者もい

る。私の研究ではこの違いは何であるのかということに着目し、

本研究のリサーチ・クエスチョンを「どのような有権者が争点投

票を行っているのか」と設定する。その問いに対して本研究では

ソーシャル・キャピタルという一つの答えを用意した。

そこで以下の節ではソーシャル・キャピタルという概念はどの

ようなものであるのかについて先行研究を紹介しながら説明し、

本研究における仮説の提示を行う。

3. ソーシャル・キャピタルと仮説 本節ではソーシャル・キャピタルについての主な先行研究につ

いて述べていきたい。そこでソーシャル・キャピタルに関する多

くの議論を呼ぶきっかけとなったロバート・D・パットナムの

Making Democracy work: Civic Tradition in Modern Italy

(Princeton, N.J.: Princeton University Press, 1993)[河田潤一訳

『哲学する民主主義-伝統と改革の市民的構造』NTT 出版, 2001 年]

について紹介する。そして、それに関連してソーシャル・キャピ

タルと政治参加(投票行動)に関する先行研究を紹介した上でソ

ーシャル・キャピタルと争点投票に関する本研究の仮説を提示す

71

る。

パットナム(1993)は、1970 年代にイタリアで、全国に等しく新

しい州制度が導入されたにもかかわらず、なぜ北部の州と南部の

州とで制度パフォーマンスが違うのかという疑問に対し、「水平的

ネットワーク、一般的信頼、互酬性の規範」としてのソーシャル・

キャピタル(社会関係資本)が地方政府のパフォーマンスを規定

すると結論付けた。これは新制度論への反証であり、独立変数と

して社会文化的な変数の比較優位を強調したものであった。

しかし、ソーシャル・キャピタルがどのように政治制度パフォ

ーマンスを高めるのかという因果メカニズムについては必ずしも

明確ではなかったように思われる。パットナム(1993)は、ソーシャ

ル・キャピタルが集合行為のジレンマを弱めることで制度パフォ

ーマンスを高めるとしたが、集合行為のジレンマが弱まるとどの

ように地方政府の統治パフォーマンスが高くなるのかまでは示し

ていない。この点に関して、ソーシャル・キャピタルが統治パフ

ォーマンスを高めるまでのメカニズムには様々な経路があるので

はないかと私は考える。ソーシャル・キャピタルが有権者の政治

参加を向上させて良い統治を生むというのも一つのメカニズムで

ある。

そこでソーシャル・キャピタルと比較的操作化の容易な政治参

加(投票行動)との関係についての研究が多くなされるようにな

った。池田(2002)は、スノーボール・パネル調査データを用い

て、2000 年衆議院選挙を対象とした分析を行った。そこではソー

シャル・キャピタルが政治参加にプラスの影響を与えていること

を明らかにした。また、自身が属しているネットワーク内の他者

の投票行動が自身の投票行動にもプラスの効果を与え、その時、

他者の党派的なバイアスも影響することが確認された。

平野(2002)も同じく、スノーボール・パネル調査データを用

いて、2000 年衆議院選挙を対象とした分析を行った。そこでは「積

極的な」団体への参加が一般的信頼や互酬性の規範に形成にプラ

スの効果を与えていることが明らかになり、そのような「積極的

な」団体への参加者は「消極的な」団体への参加者と比べて政治

参加する傾向も見られた。

西澤(2004)は、ソーシャル・キャピタルの効果を測定するこ

とを主目的としたものではないが、Jeds96 のデータを用いてソー

シャル・キャピタル(政治的会話量)が投票行動以外の政治参加

に間接的影響を与えていることを示した。

池田(2007, 第 5 章)は JESⅢのパネルデータを用いて、4 回

の国政選挙を対象にした分析を行った。ここでも池田(2002)の

ように自身が属しているネットワーク内の他者の党派的なバイア

スが自身の投票行動に影響することが確認された。また、ネット

ワーク内他者の政治的知識の量が、自民党においてはその党派的

バイアスを増進させることも部分的に示された。

以上のようにソーシャル・キャピタルと政治参加(投票行動)

における実証的研究はソーシャル・キャピタルの効果を支持する

ものが多く蓄積されてきている。しかし、ソーシャル・キャピタ

ルと争点投票の関係を扱った研究は私の知る限りない。先行研究

が示すように、例えソーシャル・キャピタルが政治参加の「量」

72

を増加させる効果があっても、その参加の「質」を保証したもの

であるとは限らない。そこで争点投票が有権者の高い合理性を必

要とすることから、争点投票という形の政治参加は質の高い政治

参加ということが出来るのではないだろうか。もしソーシャル・

キャピタルに有権者の合理性を向上させ、有権者に質の高い政治

参加を促進させる効果があるとすれば、そのことを明らかにする

ことは、民主主義を考える上でも重要な意味を持つだろうと私は

考える。

選挙の際に、有権者一人一人がその争点についてよく吟味して

投票に行くことによって良い統治者を生み、それが良い統治につ

ながるというのもソーシャル・キャピタル仮説のメカニズムの一

つとして考えられるのではないかと私は思う。ソーシャル・キャ

ピタルに富んだネットワーク内の有権者は、選挙の際に周りの有

権者との率直な意見交換を可能とし、またそのような意見交換の

中で政治的知識は増大されてゆく。このことにより、有権者一人

一人が選挙の争点についてよく理解し、政党の立場も理解した上

で投票に行くこと可能にするのではないだろうか。そこで本研究

ではその関係を扱い、「ソーシャル・キャピタル(社会関係資本)

に富んだネットワーク内の有権者が争点投票を行う」という仮説

を提示して分析を行う。

以下の節では本研究の分析枞組みを提示する。

4. 分析枞組み 本節では本研究の分析枞組みを提示する。3 具体的には扱う選

挙や争点・使用するデータについて述べる。さらに本研究ではど

のような有権者を争点投票している有権者とみなすのかについて、

また、ソーシャル・キャピタルに富んだ有権者がより争点投票を

行っているのかをどのように検証するのかについて述べたい。そ

して分析で使用する変数の作業定義を行い、作業仮説とモデル図

を提示する。

4-1. 分析対象の選挙

本研究では 2005 年の衆議院議員総選挙を分析対象として、「郵

政民営化」の是非を争点として扱って分析を行いたい。この選挙

は、当時の小泉首相のもとで「郵政選挙」と言われたことから、「郵

政民営化」が最大の争点であったと一般的に考えられている。争

点がはっきりとしていたという意味で、有権者が比較的に争点投

票を行いやすい選挙であったと考えられる。また、争点投票有権

者の存在を前提とする本研究において、その存在を確認した平野

(2007, 7 章)が同様にこの選挙を扱っていたという点でもこの選

挙を扱う意味がある。

分析に使用するデータとしては JESⅢを用いる。4 その理由と

して、本研究が争点投票有権者を確認する方法として参考とした

平野(2007, 7 章)が JESⅢを用いて分析したこと、また本研究の

独立変数であるソーシャル・キャピタルに関する質問項目が JES

Ⅲでは豊富に用意されていることが挙げられる。

73

4-2. 争点投票有権者の分析方法

ここではどのような有権者を争点投票している有権者とみなす

のかについて述べる。本研どのような有権者が争点投票を行った

とみなすのかについては平野(2007, 7 章)に倣い、以下の手順を

踏んで確認することとする。

(1)有権者の郵政民営化に対する賛否

(2)その有権者が自民党・民主党の郵政民営化に対する賛否の立

場をどのように認知しているか

(3)その有権者の比例区での投票政党(自民党・民主党のみ)

以上の 3 つの確認を行うことによって、有権者自身の政策立場

の認知、政党の政策立場の有権者による認知、それらによる有権

者の投票という争点投票が想定するメカニズム通りの投票行動を

とっている有権者を浮かび上がらせることが出来る。具体例を挙

げると、ある有権者が郵政民営化に賛成であり、かつ自民党を郵

政民営化賛成政党とみなし、かつ実際に比例区で自民党に投票し

たとする。そのような有権者を、本研究では「争点投票を行った

有権者」であると考える。ただし、上にあげた具体例の有権者が

自民党支持者の場合は必ずしも争点投票とは言えないので、その

ことを回避するために分析上は政党支持をコントロール変数とし

て投入する。

実際の分析上は比例区投票政党を従属変数としたダミー変数を

2 つ用いる。自民党に投票したかそうでないかの変数と民主党に投

票したかそうでないかの変数である。そして有権者の郵政民営化

の立場(賛成・反対)とその有権者が判断した自民党と民主党の

郵政民営化の立場(賛成・反対)がマッチしているかいないかの

ダミー変数(争点態度一致ダミー変数)を独立変数として争点投

票有権者を判定することとなる。より具体的には後述の「作業定

義」の節で詳述する。

4-3. ソーシャル・キャピタル仮説分析方法

上に述べた争点投票の定義のもとで、本研究の仮説である「ソ

ーシャル・キャピタルに富んだ有権者がそうでない有権者より争

点投票を行っている」傾向があることを確認するために、独立変

数にソーシャル・キャピタル変数と上記の争点態度一致ダミー変

数との交互作用項の変数を追加する。この交互作用項を用いるこ

とによって、争点投票を行っている有権者がどのようにソーシャ

ル・キャピタル変数の大小に影響与えているかと、ソーシャル・

キャピタル変数の大小が争点投票を行っている有権者にどのよう

な影響を与えているかを確認することができる。ここでは、後者

の確認をすること、つまり本研究の仮説を確認するために相互作

用項を用いる。

4-4. 作業定義

a)従属変数

・投票政党ダミー変数

比例区で自民党に投票したかしていないかのダミー変数(自民

党に投票した場合を「1」、そうでない場合を「0」)、を作成した。

また民主党についても同じ作業を行い計 2 つの従属変数を作成し

74

た。

上記のように従属変数がダミー変数であるため、本研究ではロ

ジスティック回帰分析を用いて分析を行う。

b)独立変数

・争点態度一致ダミー変数

この変数は郵政民営化について、有権者と有権者が評価する政

党の態度が一致しているかどうかを判定するためのものである。

具体的な作成手順は、まず有権者の郵政民営化に対する賛否のダ

ミー変数を作成して、さらにその有権者が自民党・民主党の郵政

民営化に対する賛否の立場をどのように認知しているかのダミー

変数を作成した(細かな作業定義は補遺を参照されたい)。そして、

これら 2 つ変数を用いて争点態度一致のパターンを表すために独

立変数となる 4 つのダミー変数を作成した。具体的には、

(1)郵政民営化に反対かつ、自民党を郵政民営化賛成政党とみな

した有権者「1」であるかそれ以外の有権者「0」

(2)郵政民営化に賛成かつ、自民党を郵政民営化賛成政党とみな

した有権者「1」であるかそれ以外の有権者「0」

(3)郵政民営化に反対かつ、民主党を郵政民営化反対政党とみな

した有権者「1」であるかそれ以外の有権者「0」

(4)郵政民営化に賛成かつ、民主党を郵政民営化反対政党とみな

した有権者「1」であるかそれ以外の有権者「0」

となる。5

・情報リッチなネットワーク変数

この変数はソーシャル・キャピタルを操作化したものである。

「実証分析において社会関係資本という概念に何を包含させるの

かは、分析者がこの概念を用いてどのような因果関係を分析しよ

うとしているのかを十分意識し、かつそれを明示している限りに

おいて、基本的には分析者の自由である」という坂本(2004)の指摘

がある。6 本研究ではこれにならい、ソーシャル・キャピタルの

ネットワーク属性として、有権者の政治的会話をするネットワー

クに加えて、そのネットワーク内の情報量も加味した形で扱った。

それは情報量が多いネットワークの方が争点投票をするにあたっ

ての必要な情報を獲得しやすくなるのではという推論による。7

具体的には質問「日本の首相や政治家や選挙のことが話題にな

る人で 20 歳以上の方」の話し相手(最大 4 人)の中で「その方は

政治についてはどのくらい詳しいですか」の回答項目として「あ

まり詳しくない」を 0 点、「ある程度詳しい」を 1 点、「かなり詳

しい」を 2 点として点数化した。8 したがってこの変数はそれを

足し合わせるので理論上 0~8 点の範囲をとることになる。

・交互作用項変数(争点態度一致ダミー変数 × 情報リッチな

ネットワーク変数)

この変数はソーシャル・キャピタルに富んだ有権者がそうでな

い有権者より争点投票している傾向があることを確認するための

ものである。上記の「争点態度一致ダミー」計 4 つの変数と情報

リッチなネットワーク変数をそれぞれ掛け合わせた変数を作成し

た。

c)コントロール変数

75

コントロール変数として性別、年齢、教育程度、年収、政党支

持(自民党・民主党)を作成した(細かな作業定義は補遺を参照

されたい)。これらの条件を同じとした上で上記の独立変数の効果

を確認するためである。これらを用いる理由として、例えば本研

究の従属変数である比例区投票政党は政党支持に大きく規定され

ることがわかっている。そこで政党支持の条件を同じにした上で

も争点投票が起きるのか、あるいはソーシャル・キャピタルの効

果はあるのかを確認する上で政党支持は重要なコントロール変数

となる。

4-5. 作業仮説とモデル図

上記のような作業定義のもとで、本研究では以下のような 4 つ

の作業仮説を設定する。またその 4 つの作業仮説をモデル図で表

すと図 1 のようになる。

作業仮説:

(1)郵政民営化に反対で自民党を郵政民営化賛成政党とみなした有

権者の中で、情報リッチなネットワークを持つ有権者は、そう

でない有権者よりも、自民党以外の政党に投票を行う傾向があ

る。

図 1 作業仮説 1 に基づくモデル図

(2)郵政民営化に賛成で自民党を郵政民営化賛成政党とみなした有

権者の中で、情報リッチなネットワークを持つ有権者は、そう

でない有権者よりも、自民党に投票を行う傾向がある。

図 2 作業仮説 2 に基づくモデル図

76

(3)郵政民営化に反対で民主党を郵政民営化反対政党とみなした有

権者の中で、情報リッチなネットワークを持つ有権者は、そう

でない有権者よりも、民主党に投票を行う傾向がある。

図 3 作業仮説 3 に基づくモデル図

(4)郵政民営化に賛成で民主党を郵政民営化反対政党とみなした

有権者の中で、情報リッチなネットワークを持つ有権者は、そ

うでない有権者よりも、民主党以外の政党に投票を行う傾向が

ある。

図 4 作業仮説 4 に基づくモデル図

*モデル図のプラスとマイナスの符号は係数の符号を指す。具体的には(1)

の有権者は自民党よりも自民党以外の政党に投票する傾向があり、(2)の有

権者は他の政党より自民党に投票する傾向があるということである。また、

破線と実線の違いは破線よりも実線の方がより因果関係の効果が強いこ

とを表す。具体的には、(1)では「郵政反・自民郵政賛-有権者」の中でも

情報リッチなネットワーク内の有権者の方がより自民党投票を行う傾向

があることを示す。

*本稿では以下より、表記が煩雑になることを避けるために下記の表記を

用いる。

・「郵政民営化に反対で自民党を郵政民営化賛成政党とみなした有権者」

→「郵政反・自民郵政賛-有権者」

・「郵政民営化に賛成で自民党を郵政民営化賛成政党とみなした有権者」

→「郵政賛・自民郵政賛-有権者」

・「郵政民営化に反対で民主党を郵政民営化反対政党とみなした有権者」

→「郵政反・民主郵政反-有権者」

・「郵政民営化に賛成で民主党を郵政民営化反対政党とみなした有権者」

→「郵政賛・民主郵政反-有権者」

77

5. 分析 本研究では合計 4 つのロジスティック回帰分析を行った。分析

に使用した 4 つのモデルを紹介したい。まずは従属変数が自民党

投票のものを「自民党投票モデル」とし、民主党投票のものを「民

主党投票モデル」とする。そしてそれぞれのモデルの中にさらに 2

つのモデルを用意した。それは今回の主たる独立変数である情報

リッチなネットワーク変数とその交互作用項変数を用いない「通

常モデル」とそれらの変数を含めた「Social Capital(SC)モデル」

である。通常モデルはソーシャル・キャピタルの影響を考慮せず、

単純に争点投票が行われているかどうかを確認するものである。

このような分析をしたのはソーシャル・キャピタルがどのように

争点投票に影響を与えているか確認するためである。

5-1. 自民党投票モデル では、まずは自民党投票モデルから確認したい。通常モデルと

SC モデル、2 つの分析を 1 つの表にまとめたものが以下の表 1 で

ある。

表 1 郵政民営化に対する争点投票の SC の影響(自民党投票モデル)

通常モデル SC モデル

比例区自民党投票

政党支持 2.14 *** 2.203 ***

郵政反・自民郵政賛-有権者 -1.349 *** -1.246 ***

郵政賛・自民郵政賛-有権者 0.367 * 0.129

情報リッチなネットワーク

-0.332

郵政反・自民郵政賛-有権者×

情報リッチなネットワーク

0.101

郵政賛・自民郵政賛-有権者×

情報リッチなネットワーク

0.379 *

男性 0.076 0.038

年齢 0.001 0.001

教育程度 -0.088 -0.088

年収 0.000 0.000

定数 -1.149 *** -0.989 ***

Nagelkerke-R2 乗 0.401 0.413

Model 危険率 p < 0.01 p < 0.01

*p<0.1 **p<0.05 ***p<0.01

まずは通常モデルからみてみよう。「郵政賛・自民郵政賛-有権

者」をみると 10%水準ではあるが、統計的に有意なプラスの係数

となっている。これは、「郵政賛・自民郵政賛-有権者」は自民党

に投票する傾向があるということである。すなわち、争点投票で

ある。次に「郵政反・自民郵政賛-有権者」をみると統計的に有意

なマイナスの係数となっている。これは、「郵政反・自民郵政賛-

有権者」は自民党以外の政党に投票する傾向があるということで

ある。これも同じく争点投票のメカニズムに合致した形である。

これらは、政党支持をコントロールした上での結果である。つま

78

り、自民党への投票に大きな影響を持つ自民党への政党支持を一

定とした上でも、有権者が郵政民営化への是非を判断した上で自

民党へ投票するか否かを決めていたのである。以上の結果は当然

ではあるが平野(2007, 7 章)の結果とほぼ合致する。

次に本研究の主たる目的であるソーシャル・キャピタルの効果

を確認するために SC モデルの結果をみていただきたい。まずは

「郵政賛・自民郵政賛-有権者 × 情報リッチなネットワーク」の

交互作用項をみてみると、10%水準ではあるが統計的に有意なプ

ラスの係数になっている。これは、「郵政賛・自民郵政賛-有権者」

の中で、情報リッチなネットワーク内の有権者は自民党に投票す

る傾向があるということである。また、SC モデルの「郵政賛・自

民郵政賛-有権者」をみると、通常モデルでは統計的に有意であっ

たものがそうでない結果になったことがわかる。「郵政賛・自民

郵政賛-有権者」が自民党に投票する傾向は、情報リッチなネット

ワーク交互作用項でコントロールすると自民党投票の傾向はなく

なった。つまり、「郵政賛・自民郵政賛-有権者」の中では情報リ

ッチなネットワークにいる有権者の方がより争点投票を行うとい

うことであり、これは本研究の作業仮説 2 と合致する。

しかし、「郵政反・自民郵政賛-有権者 × 情報リッチなネット

ワーク」の相互作用項をみてみると、統計的に有意な結果にはな

らなかった。これは、「郵政反・自民郵政賛-有権者」は自民党以

外の政党に投票する傾向があるが、特にそれは情報リッチなネッ

トワーク内の有権者において増す傾向ではなかったということで

ある。これは、本研究の作業仮説 1 が実証されたとはいえない結

果になった。

ここまでの議論をまとめると、確かに情報リッチなネットワー

ク内の有権者は争点投票を他の有権者よりも行う傾向があるが、

それは「郵政賛・自民郵政賛-有権者」の中においてだけであると

いうことである。

79

5-2. 民主党投票モデル

次に、今度は民主党投票モデルの通常モデルと SC モデル、2 つ

の分析結果を比較してみる。その分析結果を 1 つの表にまとめた

ものが以下の表 2 である。

表 2 郵政民営化に対する争点投票の SC の影響(民主党投票モデル)

通常モデル SC モデル

比例区民主党投票

政党支持 2.482 *** 2.54 ***

郵政反・民主郵政反-有権者 0.633 *** 0.303

郵政賛・民主郵政反-有権者 -0.637 *** -0.6 **

情報リッチなネットワーク

-0.12

郵政反・民主郵政反-有権×

情報リッチなネットワーク

0.267 **

郵政賛・民主郵政反-有権×

情報リッチなネットワーク

-0.063

男性 0.028 0.032

年齢 0.001 0.001

教育程度 0.052 0.052

年収 0.001 0.000

定数 -1.738 *** -1.537 ***

Nagelkerke-R2 乗 0.335 0.352

Model 危険率 p < 0.01 p < 0.01

*p<0.1 **p<0.05

***p<0.01

まずは自民党投票モデルと同じく通常モデルからみて頂きたい。

「郵政反・民主郵政反-有権者」をみると統計的に有意なプラスの

係数となっている。これは、「郵政反・民主郵政反-有権者」は民

主党に投票する傾向があるということである。同じく「郵政賛・

民主郵政反-有権者」をみても有意な結果であり、自民党投票モデ

ルと同じように争点投票のメカニズムに合致した有権者の存在が

確認できた。

そして次に SC モデルである。まずは「郵政反・民主郵政反-有

権者 × 情報リッチなネットワーク」の交互作用項をみてみると、

有意なプラスの係数となっている。これは、「郵政反・民主郵政反

-有権者」の中で、情報リッチなネットワーク内の有権者は民主党

に投票する傾向があるということである。また、SC モデルの「郵

政反・民主郵政反-有権者」をみると通常モデルでは有意な結果で

あったものがそうでなくなったことがわかる。つまり、「郵政反・

民主郵政反-有権者」の中では情報リッチなネットワーク内にいる

有権者の方がより争点投票を行うという本研究の作業仮説 3 の通

りである。

しかし、自民党投票モデルと同じように必ずしも仮説が支持さ

れたといえない結果もあった。それが「郵政賛・民主郵政反-有権

者 × 情報リッチなネットワーク」の交互作用項の結果である。

ここは統計的に有意な結果とならなかった。つまり、「郵政賛・民

80

主郵政反-有権者」の民主党以外の政党に投票する傾向は情報リッ

チなネットワーク内の有権者に増す傾向ではなかったということ

である。これは、本研究の作業仮説 4 が実証されたとはいえない

結果になった。

以上の分析結果をふりかえると部分的には本研究の仮説が支持

されたが、部分的には支持されたとはいえない結果もあった。支

持された結果の傾向は自民党投票モデルと民主党投票モデルで共

通する点があった。すなわち、それは自民党投票モデルでは自民

党投票の方向へ、民主党投票モデルでは民主党投票の方向へ、ソ

ーシャル・キャピタルが争点投票においては効果を持っていたと

いう点である。このことは何を意味するのだろうか? 今回の分

析をもって、それを明らかにすることはできないが、有権者が持

つネットワークには政党支持あるいは、政党支持とまではいえな

い党派的なバイアスがあるのかも知れない。もし、そうだとする

とこの結果は私が想定した形の争点投票メカニズムが働いた結果

であるとは必ずしも言うことが出来ない。

あるいは、自民党投票モデルと民主党投票モデルで支持されな

かった結果の傾向も共通していて、この側面から考察することも

可能だろう。自民党投票モデルと民主党投票モデルどちらにおい

ても、有権者の郵政民営化への態度とその有権者が評価する政党

の立場が違っている場合、情報リッチなネットワークが効果を持

たなかった。つまり有権者は自分の立場と違う政党に対してはそ

もそも正当に評価しようとしないので情報リッチなネットワーク

を必要としなかったのかも知れない。このようなメカニズムが働

いていたとすれば、本研究の結果は妥当なものであろう。いずれ

にせよ、これらのことは推論の域を出ないので新たにそのことを

明らかにするためには別の分析が必要であろう。

6. おわりに 本研究では争点投票をする有権者としない有権者を分かつ原因

は何かという問いを立て、それはソーシャル・キャピタルである

という答えを用意し、4 つのモデルを用意して実証的に分析した。

その結果は部分的には支持されたということが出来るだろう。つ

まり、ソーシャル・キャピタルに富んだ有権者はそうでない有権

者よりも争点投票を行う傾向があるということである。

しかし、いくつかの課題も残った。まず、それは本研究の仮説

を支持しない結果となったものもあり、さらにその傾向が自民党

投票モデルと民主党投票モデルとで一致した形であったことであ

る。つまり、自民党投票モデルでは民主党投票の方向へ、民主党

投票モデルでは自民党投票の方向へはソーシャル・キャピタルの

効果があったとは言えなかったことである。このことはソーシャ

ル・キャピタルのネットワークバイアスを想起させる。政党支持

とまではいえないような自民党のネットワークがあり、それが結

果的に自民党への投票を促す効果があり、同じように民主党のネ

ットワークが民主党への投票を促していたとすれば、このとこを

明らかにするために別の分析が必要である。

次に争点の特殊性についてである。今回は 2005 年衆議院議員総

81

選挙の結果を用いて、その争点を郵政民営化の是非と設定した。

しかし、争点には比較的、賛成・反対を判断しやすいものもあれ

ばそうでないものもある。このことを考慮するなら、同じ選挙で

のいくつかの争点による比較分析、また他の選挙との比較分析が

必要である。

以上のような課題も残ったが、本研究は一定の成果もあった。

パットナムはソーシャル・キャピタルが良い統治を生むと言った

が、そのメカニズムの経路は無数にあるように思われる。有権者

一人一人が選挙の際にその争点についてしっかり吟味して投票に

行くことによって良い統治者を生み、それが良い統治につながる

というのもそのメカニズムの一つであろう。そうだとすると、争

点投票にソーシャル・キャピタルが影響を与えていることを部分

的に実証できたことはソーシャル・キャピタル研究における意義

があったといえる。また、有権者にとって困難とされる争点投票

が、ソーシャル・キャピタルに富んだ有権者の中で特に行われて

いるという本研究の分析結果は、争点投票をしている有権者の特

定という意味で争点投票研究にも一定の意義があったのではない

だろうか。残された疑問は今後の課題として本稿を終わりとした

い。

(1) 田中愛治 1998 「選挙研究における『争点態度』の現状と課

題」『選挙研究』13 号 17 頁

(2) 前掲書 17 頁

(3) 本研究は争点投票をしている有権者を確認するための分析

手法として平野(2007, 7 章)の分析枞組みを基本的には踏

襲している。コントロール変数の作業定義で、平野(2007, 7

章)と違っている部分は本稿の補遺を参照していただきた

い。

(4) 文部科学省科学研究費・特別推進研究「21 世紀初頭の投票

行動の全国的・時系列的調査研究」による調査。メンバーは

池田謙一(研究代表者)、小林良彰、平野浩。本データはレ

ヴァイアサン・データバンクより同志社大学・法学部が購入

したものを、教材として西澤教授より提供を受けた。それぞ

れのデータを公開・寄託され、利用でるようにしてくださっ

た先生方に感謝いたします。

(5) 自民党を郵政民営化反対、民主党を郵政民営化賛成と有権

者が評価した変数を作成しなかったのは、それぞれのデータ

が比較的尐なかったためである。自民党を郵政民営化反対と

認知した有権者は弱い反対も含め 13 人(弱い賛成を含めた

数が 1374 人であったのに対し)のみであった。また、民主

党を郵政民営化賛成と認知した有権者は弱い賛成も含めると

307 人(弱い反対を含めた数が 989 人に対し)であり、こち

らは必ずしも尐ない数とはいえないが、自民党投票モデルと

の整合性をとるために今回は便宜的に民主党を郵政民営化反

対政党とみなして分析をした。

82

(6) 坂本治也 2004 「社会関係資本の二つの『原型』とその含意」

『阪大法学』53 巻 6 号 190 頁

(7) 操作化に関して池田(2007, 6 章)を参考とした。

(8) 回答から DK・NA は除いた。

<補遺>

分析に使用した変数の作業定義

●「争点態度一致ダミー変数」作成に使用した変数に関するもの

有権者の争点態度ダミー変数

・質問文「郵政民営化について、次のような代表的な二つの意見

があります。あなたの意見はどちらに近いですか。この中ではど

れにあたりますか。」

・意見

A:郵政事業の効率を良くしてコストを下げるためには、郵政民

営化に賛成である。

B:郵政事業が撤退して困る地域が出てくるので、郵政民営化に

は反対である。

・回答コード「1:A に近い、2:どちらかといえば A、3:どち

らかといえば B、4:B に近い、5:わからない、6:答えない」

から「5:わからない、6:答えない」を欠損値として除き、「1:

A に近い、2:どちらかといえば A」を「1:賛成」、「3:どちら

かといえば B、4:B に近い」を「0:反対」に再コードしたダ

ミー変数を作成。

有権者による政党の争点立場認知ダミー変数

・質問文「それでは、次にあげる政党は、郵政民営化の問題につ

いて、どのような主張をしていると思いますか。」

・回答コード「同上」から上記と同じ作業で自民党と民主党に対

する評価のダミー変数を 2 つ作成した。

●性別

・質問票回答コード「1:男性、2:女性」を「0:女性、1:男

性」に再コード。

●年齢

・2005 から質問票回答生まれ年を引き年齢とした(連続変数)。

●教育程度

・質問文「あなたが最後に卒業された学校はどちらですか。」

・回答コード「1:新中学・旧小・旧高小、2:新高校・旧中学、

3:高専・短大・専修学校、4:大学・大学院、5:答えない」か

ら「5:答えない」を欠損値として除きそのまま順序変数として

利用した。

●年収

・質問文「去年(平成 16 年 1 月~12 月)1 年間のお宅の収入は

ご家族全部あわせると、およそどのくらいになりますか。ボーナ

スや臨時収入を含め、税込みでお答えください。」

・回答コード「1:200 万円未満、2:200 万円~400 万円未満、

3:400 万円~600 万円未満、4:600 万円~800 万円未満、5:

800 万円~1000 万円未満、6:1000 万円~1200 万円未満、7:

1200 万円~1400 万円未満、8:1400 万円~2000 万円未満、9:

83

2000 万円以上、10:わからない、11:答えない」から「10:わ

からない、11:答えない」を欠損値として除きそのまま順序変

数として利用した。

●政党支持(自民党投票モデル)

・質問文「話はかわりますが、今回何党に投票するかは別にして、

ふだんあなたは何党を支持していますか。」

・回答コード「1:自民党、2:民主党、3:公明党、4:社民党、

5:共産党、6:国民新党、7:新党日本、8:その他の政党、9:

政党支持なし、10:わからない、11:答えない」

から「10:わからない、11:答えない」を欠損値として除き、「1:

自民党」以外を全て「0:その他」に再コードしたダミー変数と

して利用。

●政党支持(民主党投票モデル)

・上記の「政党支持(自民党投票モデル) ダミー変数」の自民党

と民主党を入れ替えたダミー変数として利用。

●一般的信頼

・質問文「ほとんどの人は信頼できる」

・回答コード「1:そう思う、2:どちらかといえばそう思う、3:

どちらともいえない、4:どちらかといえばそう思わない、5:

そう思わない、6:わからない、7:答えない」から「6:答えな

い」「7:答えない」を欠損値として除き、「1~5」の値を逆順に

して再コードした。

<参考文献>

・池田謙一 2007. 『政治のリアリティと社会心理 平成小泉政

治のダイナミックス』木鐸社.

・池田謙一 2002. 「2000 年衆議院選挙における社会関係資本

とコミュニケーション」『選挙研究』17 号 pp5-18.

・伊藤光利・田中愛治・真渕勝 2000 『政治過程論』有斐閣

・小林良彰 2006. 「マニフェスト選挙以降の争点態度投票」『選

挙研究』21 号 pp7-38.

・小林良彰 2000. 『選挙・投票行動』東京大学出版会.

・坂本治也 2004. 「社会関係資本の二つの『原型』とその含意」

『阪大法学』53 巻 6 号 pp181-210.

・坂本治也 2010. 『ソーシャル・キャピタルと活動する市民』 有

斐閣.

・田中愛治 1998 「選挙研究における『争点態度』の現状と課

題」『選挙研究』13 号 pp17-27.

・谷口尚子 2005. 『現代日本の投票行動』慶應義塾大学出版会.

・西澤由隆 2004. 「政治参加の二重構造と『関わりたくない』

意識-Who said I wanted to participate?」『同志社法学』55

巻 5 号 pp1-29.

・平野浩 2002.「社会関係資本と政治参加――団体・グループ加

入の効果を中心に」『選挙研究』17 号 pp19-30.

・平野浩 2005. 「日本における政策争点に関する有権者意識と

その変容」小林良彰(編)『日本における有権者意識の動態』

慶応義塾大学出版会 pp61-80.

・平野浩 2007. 『変容する日本社会と投票行動』木鐸社

84

・ロバート・D・パットナム/河田潤一訳 2001. 『哲学する民主

主義』NTT 出版.

・ロバート・D・パットナム/柴内康文訳 2006. 『孤独なボウリ

ング』柏書房.

85

12 「無風選挙」と有権者

の投票行動 日裏 瑠奈

1. はじめに 選挙の競争度と投票率には相関関係がある。(山田、1992・蒲島、

1998)そして、競争度が極めて低い選挙は「無風選挙」と呼ばれ

ている(蒲島、1988)。競争度が低い選挙が起きる要因には、直近の

選挙で無風選挙となった佐賀の知事選のように際立った争点がな

いことが挙げられる。1また、無風選挙は選挙に立候補した時点で

当選者が確定していることが特徴である。

無風選挙が特に問題であるのは知事選挙であると蒲島(1988)は

指摘している。蒲島によると、1956 年から 1987 年までの約 30 年

間の知事選挙分析から政党の相乗りで選挙結果の予想があきらか

になる選挙が増え、知事選の競争度が低下してきているとのこと

である。そして、当選者と次点者の得票率の差が 10%開けば、2.5%

投票率は下がることも示している。

また、国政選挙においても無風選挙区の存在が確認されている。

例えば、水崎・森(2007)は、1996・2000・2003 年の3回の総選挙

について、無風選挙の存在を指摘している。当選者と次点者の得

票差が選挙区有効得票に占める割合を算出した結果から、「小選挙

区部分の選挙戦の接戦の模様は(接戦の度合い)は、選挙区によ

って大きく異なっているのである。300 ある小選挙区のうち、次点

者に圧倒的な差をつけて当選者が決定した無風選挙区が、かなり

存在することは注目されよう」と論じている(『総選挙の得票分析

1958-2005』水崎・森 2007、147 項)。

これらのことから、無風選挙はあらゆる選挙戦で起こっている

と考える。W・ライカー&P・オードシュック(1968)らは有権者の

主観での選挙戦の状況が接戦であるかどうかが投票参加に関わっ

てくると指摘した。そうであるならば、無風選挙では有権者は投

票意欲が湧いてこないと考えられる。しかし、投票率は格段に減

るが、投票をする有権者も尐なからずいるのだ。無風選挙もしく

は無風選挙区において、投票所に行くまでにかかるコストを払っ

てまで投票参加する有権者とはどのような人々であるのか、この

点について考えていきたい。

86

2. 仮説 この問題提起に答える仮説として、「選挙(区)が例え接戦ではな

かったとしても投票に行かなければならないという、強い政治的

義務感を有権者が感じるからだ。」と設定する。

まず、仮説として挙げた「政治的義務感」について先行研究を

紹介する。「政治的義務感」が投票にもたらす影響を研究したもの

は多くある。W・ライカーと P・オードシュックら(1968)は、「政

治的義務感」についてのモデルを確立した。彼らによると、有権

者は投票の「結果」がもたらす長期的・短期的な効用だけでなく、

「投票すること自体」からもさまざまな効用を得ているというこ

とである。そして、ダウンズが指摘した長期的利益の概念(すべて

の有権者が合理的な行動を選択しない理由)を明確化し、「R=PB-

C+D」を主張した。この式の意味することは、「市民が投票に与え

る可能性(P),異なる政党や候補者を選ぶことによって期待される

利益(B)を掛け合わせた PB と、投票することによって義務感を果

たしたという満足感(D)の和が投票のコスト(C)よりも大きいと

きに人々は投票するという。」(蒲島、1988 、52 項) 蒲島(1988)

は、この投票義務感を政治的文化要因であり、長い間の社会生活

や教育によって身に付くものであり、経済的合理性から見れば非

合理的だと思われる無風選挙においても、投票義務感を有権者は

感じるために投票にでかけるのだと推論している。2

また、「政治的義務感」について、日本においても投票参加を促

す大きな要因だとして、その効果の検証がなされている。三宅は

1976 年衆議院選挙と 1986 年の同時選挙を分析し、有権者の投票

義務感が低いことが、投票率を低下させる要因になっていること

を示している(三宅,1990)。また、三宅・西澤は市民としての義務

感があることが投票率を高くする要因であるとしている(三宅・西

澤、1997)。さらに、蒲島は政治参加に対する政治的関与の影響で、

投票参加のコスト感覚の次に政治的義務感の影響が大きく、政治

的義務感が強ければ強いほど有権者の投票参加は高いと分析して

いる。

以上の先行研究から、仮説として立てた「政治的義務感」は、

一般的な(無風状態でない)選挙においても有意な値を示すとさ

れてきたことが分かる。それでは、なぜ無風選挙であっても投票

参加するのだろうか。通常の選挙において、政治的義務感を有権

者が持っていることが投票参加を促す要因だと指摘されている。

投票結果があらかじめ分かっているような選挙であっても有権者

が投票参加をするのは、無風状態でない選挙よりもさらに「強い

政治的義務感」を感じているからだと考える。義務を感じている

有権者は、投票に行くこと自体に満足感を感じており、投票に行

くことが合理的かどうかをあまり考慮に入れないのだろう。よっ

て、投票を義務だと感じている有権者は、選挙が例え激しい選挙

ではなかったとしても、投票参加すると考えられる。逆に、投票

義有漢を感じていない有権者は「無風状態」の選挙では、義務感

を感じている有権者より選挙状況やコスト感覚に敏感であるだろ

うから、投票参加する割合は低下するだろう。選挙区が「無風状

87

態」である場合においても値が有意であるならば、有権者の投票

参加における心理的メカニズムがどのような状況下であっても効

いていることを証明できるのではないだろうか。このことを確認

するために、有権者が考える競争度の高い選挙区と競争度の低い

選挙区で同じ変数を比較し、投票に対する両者の違いを見ていく。

「無風状態」である選挙には尐なくとも二つある。一つは、世

論調査から導き出される有権者の主観による選挙の状況認識であ

る。もう一つは集計データによって導きだされる客観的な選挙区

の状況である。主権的な選挙の状況認識のほうが、客観的な認識

よりも有権者の心理状態を大きく反映すると考えられる。このこ

と―有権者は実際の選挙状況で判断しているのか、それとも有権

者の心理的な側面が大きく作用しているのか―を確認するために

主観的な選挙区の状況認識の指標だけでなく、客観的な選挙区状

況の指標も仮説に入れて検討していく。詳しい説明は次節と補遺

で確認していただきたい。

3. 分析枞組み

3-1.使用するデータ

本稿で扱うデータは 2 種類ある。一つは、有権者の主観を確認

するためのサーベイ・データである。サーベイ・データは、1996

年に実施された「衆議院選挙に関する世論調査」(Japan Election

Studies group)を使用する。3 もう一つは、客観的なデータを確認

するめの集計データである。ここでは、「JED―M 衆議院議員

総選挙データ第 28 回~44 回」に収められている第 41 回衆議院議

員総選挙の分を使用する。4

3-2. 作業定義

本稿では、分析を行うために 3 つの変数を用いる。1 つは、被説

明変数である「投票参加」。2 つ目は説明変数である「政治的義務

感」。そして、3 つ目は無風状態かそうでないかで場合分けをした

変数である。これら 3 つの変数の関係をクロス表分析で確認する

ため、それぞれに関連する質問への回答をコード化しなおしてい

る。それぞれ、「分からない」「答えない」は欠損値として省いた。

詳しくは補遺を参照いただきたい。なお、客観的な選挙区の状況

は「接戦度」を計算した後、10%以上の選挙区を「接戦選挙区」、

10%~30%未満を「どちらでもない」、30%以上を「無風選挙区」と

便宜上定義した。

88

4. 分析結果 <分析 1>

まず、有権者は選挙戦が激しいほど投票に参加しているのかど

うかを確認する。そのために、表1で投票参加と選挙戦の程度の

クロス表で関連性を見る。

【分析 1 の結果:表 1】

表1 投票参加と選挙戦の程度のクロス表

選挙戦は激戦だったか

以前より激しい 変わらない 以前より無風状態

投票参加 投票へ行った 90.2 84.6 84.3

棄権した 9.8 15.4 15.7

合計(%) 100.0 100.0 100.0

N 325 506 343

カイ二乗値:6.376 危険率:.000

出所:衆議院に選挙に関する世論調査

表 1 を見ると、「以前より激しい」と選挙を感じている人投票へ

行った割合は 90.2%であり、選挙戦が激しいほうが投票参加をす

る割合が高いことが分かる。しかし、「以前より無風状態」だと感

じている人が、投票参加をした割合は 84.3%と高い数値を示して

いる。どうも、有権者が投票に行くのは選挙戦の激しさだけでは

ないようだ。「無風状態」であっても投票参加をするのはなぜなの

か。以下の分析で、この疑問について考えていく。

89

<分析 2>

先ほどの表1に、本稿の仮説である「政治的義務感」について

どう考えるかの質問文を加え、多重クロス表で関連性を確認する。

【分析 2 の結果:表 2】

表2 選挙戦の程度と投票参加と投票義務感(主観的)

選挙戦の程度

① ①以前より激

しい

投票は義務か

思う どちらとも言えない 思わない

投票参加 投票へ行った 92.0 81.0 86.7

棄権した 8.0 19.0 13.3

合計 100.0 100.0 100.0

N 274 21 15

カイ二乗値:3.237 危険率:.198

② 変わらな

投票は義務か

思う どちらとも言えない 思わない

投票参加 投票へ行った 90.2 66.7 80.0

棄権した 8.3 26.7 20.0

合計 100.0 100.0 100.0

N 396 48 25

カイ二乗値:22.394 危険率:.000

③以前より無

風状態

投票は義務か

思う どちらとも言えない 思わない

投票参加 投票へ行った 88.8 57.7 63.6

棄権した 11.2 23.4 36.4

合計 100.0 100.0 100.0

N 285 26 11

カカイ二乗値:46.400 危険率:.000

出所:衆議院に選挙に関する世論調査

②・③の分析結果では危険率がどちらも低く、信頼できる結果

が得られた。

まず、③「以前より無風状態」と感じている有権者が投票参加

した割合を見たい。「以前より無風状態」だと感じていて、「投票

は義務だと思う」人が投票参加した割合は 88.0%と高い数値を示

している。逆に、「投票を義務だと思わない」人が投票へ行った割

合は 63.6%である。この二つの値を比べると、「投票義務感」を持

っていれば選挙区が「無風状態」であったとしても投票参加する

と考えられる。

次に、「以前より無風状態」だと感じている場合と「以前より無

風状態」と感じている場合を比べる。そうすると、「以前より無風

状態」と感じている人で、投票は義務だと思っている人が投票に

行く割合が、「以前より激しい」で投票義は義務だと思っている割

90

合と比べて、あまり差がないことが分かる。しかし、「無風状態」

の場合で「投票を義務ではない」と思っている人が棄権した割合

が 36.4%なのに比べて、「以前より激しい」場合で棄権した割合は

13.3%であった。このことから、「以前より無風状態」であったと

しても、投票を義務だと思っていれば、投票参加をする確率が高

いと考えられる。つまり、投票義務感は選挙戦が激しくない場合

に関連性が見えてくると考察できる。

<分析 3>

ここまでの分析は、サーベイ・データを用いたものであり、有

権者の主観からの結果が得られていると言える。選挙が無風なの

か、それとも激しいのかは、有権者の主観的判断であるので、実

際に有権者が投じた選挙区がどのようであったかは分からない。

そこで、主観的な結果と客観的な結果と比較するために、1996 年

の衆議院議員総選挙の各選挙区の接戦度を求め、サーベイ・デー

タでの選挙区の有権者の主観的な認識と入れ替えて分析する。5

【分析 3 の結果:表 3】

表3 実際の選挙区の状況と投票参加と投票義務感(客観的)

選挙戦の程度

③ 接戦選挙区 投票は義務か

(10%未満) 思う どちらとも言えない 思わない

投票参加 投票へ行っ

88.4 67.9 73.0

棄権した 11.6 19.8 27.0

合計 100.0 100.0 100.0

N 510 53 57

カイ二乗値:21.617 危険率:.198

④ ど ち ら で

もない

投票は義務か

(10%~30%未満) 思う どちらとも言えない 思わない

投票参加 投票へ行っ

90.5 63.0 72.2

棄権した 9.5 37.0 27.8

合計 100.0 100.0 100.0

N 368 46 18

カイ二乗値:30.379 危険率:.000

⑤ 無風選挙

投票は義務か

(30%以上) 思う どちらとも言えない 思わない

投票参加 投票へ行っ

88.1 53.8 85.0

棄権した 11.9 46.2 15.0

合計 100.0 100.0 100.0

N 176 13 193

カイ二乗値:16.975 危険率:.000

91

表 3 から得られる結果としては、客観的な接戦度と投票義務感

は、影響がないと考えられる。まず、⑥「無風選挙区」を見てい

ただきたい。投票を義務だと思っている人が投票へ行った割合が

88.1%であるのに対し、義務だと思わない人が投票へ行った割合は

85.0%とほぼ同じ値を示している。主観的な選挙区の状況認識であ

る分析 2 の結果③「無風状態」の場合、両者にはっきりと違いが

見られた。客観的な選挙区のデータを使用したとしても、無風区

の場合には「投票義務感」が効いてくると予想していたため、「無

風選挙区」での異なった政治的義務感の考え方を持つ有権者の間

でそれほど差が見られない結果が出たことに驚いた。「無風選挙

区」で投票する有権者には「投票義務感」ではない別の要因が働

いているのだろうか。また、「無風選挙区」と「どちらでもない」

と定義した分析結果の危険率は低く、信頼できる結果だと言える。

5. 考察(分析 2 と分析 3 を比較して)

分析 2(主観的)と分析 3(客観的)を比較検討していく。

分析 2 と分析 3 からは異なる結果が得られた。分析 2 では、仮

説として立てた「投票義務感」が③「以前より無風状態」だと感

じている有権者の場合において関連性が確認できた。一方、実際

の選挙区の状況を考慮した分析 3 からは、思い描いていたような

結果を得ることができなかった。それは、「無風選挙区」の場合で

投票を義務だと思っていない人が投票に行った割合が、義務だと

思っている人の割合とさほど変わらなかったためである。

分析 2 と分析 3 に共通して見られることが一つある。それは、

投票は義務かという問いに対して「どちらとも言えない」と考え

ている有権者の投票参加のそれぞれの割合についてである。「どち

らとも言えない」という有権者が投票参加する割合は全ての結果

において「思う」「思わない」よりも低い数値を示している。投票

を義務でもなく権利でもないと感じている有権者は、選挙そのも

のに関心が低いため、投票参加する割合も低くなるのだろうか。

分析 2 と分析 3 の結果が異なったことから、何が言えるだろう

か。主観的な「無風状態」選挙区の状況認識からは、有権者は合

理的に行動しているように見える。客観的な選挙区の状況認識の

結果からは、有権者は合理的ではない行動をとっているように思

われる。しかし、投票を義務だと感じていなくても「無風選挙区」

では 85%もの割投票参加するとういう結果が示された。この数字

は「接戦選挙区」の場合よりも高い。投票参加には、投票義務感

が効いていることは確かかもしれないが、それだけの理由ではな

いことが、分析結果が示してくれている。

92

6.今後の課題

本稿の分析は、「なぜ有権者は無風選挙でも投票に行くのか?」

という関心によるものであった。この問題提起への仮説として、

様々な研究で取り上げられている「政治的義務感」が、無風選挙

においてさらに強い関連性が見えてくるのではないかと考えた。

分析結果としては、6.考察でも述べたが、投票義務感が効いてい

るというよりは、むしろ各選挙区の状況や他の要因が有権者の心

理に働きかけていると言えるようなものであった。特に、実際の

選挙区の状況における無風選挙区では、投票を義務感だと思って

いなくても 85%の人が投票参加をしたという結果になった。この

ことから、有権者が投票参加を決定する要因は別のところに存在

するのではないかと考える。

例えば、当選者が確実な選挙区では、その立候補者の固い支持

基盤があり、その候補者からの恩恵を期待して有権者が投票参加

を決定しているのかもしれない。また、根本的な問題として、有

権者はあまり選挙区の状況を認識していないのかもしれないこと

が挙げられる。また、今回の分析では、有権者を投票へと動員す

る指標として投票義務感のみを対象とし、他の要因を考慮できな

かった。また、今回データとして扱わせていただいている第 41 回

衆議院議員総選挙は小選挙区制が初めて導入された選挙である。

このことから、有権者の選挙への関心が高かったのかもしれない。

昨今の日本では投票率が低下していると言われているが、それ

でも棄権をせずに投票に行く有権者がいる。「なぜ有権者は投票に

行くのか。」今後も探求していきたいテーマである。

(1)佐賀全県 1 地方 朝刊(2011 年 04 月 12 日)

(2) W・ライカー&P・オーデシュックらの概念の説明は蒲島

(1988)を参照した。

(3) ここで使用したデータは「衆議院選挙に関する世論調査」

(1996,Japan Election Studies group)である。それをレヴ

ァイアサン・データ・バンク(LDB、木鐸社)より同志社大

学が購入したものを使用した。このデータは、同志社大学・

法学部の西澤由隆先生のご指導と便宜により利用ができた。

このデータを公開・寄託され、利用できるようにしてくださ

った先生方に感謝いたします。

(4) 本稿で使用させていただいた「JED―M(Japan Election

Data byMizusaki) 衆議院議員 総選挙データ第 28 回~44

回」とは、水崎節文・森裕城氏が集計したデータを編纂した

ものである。それをレヴァイアサン・データ・バンク(LDB、

木鐸社)より同志社大学が購入したものを使用した。このデ

ータは、同志社大学・法学部の西澤由隆先生のご指導と便宜

により利用ができた。このデータを公開・寄託され、利用で

きるようにしてくださった先生方に感謝いたします。

93

(5) 「接戦度=(当選者得票―次点者得票)÷選挙区有効票×100」

(2007,水崎・森)で求めた。なお、各選挙区のデータは「JED

―M 衆議院議員 総選挙データ第 28 回~44 回」に収めら

れている、第 41 回衆議院議員の集計データ分である。

<補遺>

クロス表分析に用いた質問文と回答肢のコード化

・被説明変数「投票参加」

Q2(事後)いろいろな事情で選挙のときに投票に行けない人が

ありますが、あなたは、10 月 20 日(日)の衆議院選挙の投

票に行きましたか。この中からあげてください。

(ア)行くつもりだったが、行けなかった (イ)行かなかった

(ウ)行った わからない 答えない

・回答コード

1.棄権した「行くつもりだったが、行けなかった」「行かなかっ

た」

0.投票へ行った「行った」

・独立変数「政治的義務感」

Q30(事前)〔回答票 30〕次に国民と選挙や政治とのかかわり合

いについての意見を読み上げます。それぞれについてのあな

たのお気持ちを「賛成」「どちらかといえば賛成」「どちらと

もいえない」「どちらかといえば反対」「反対」と分けた場合、

それぞれどれになりますか。

1)自分の支持している政党や候補者が 勝つ見込みがないとき

には、投票してもむだである

2)選挙では大勢の人々が投票するのだから、自分一人位投票して

も、しなくても、どちらでもかまわない

3)投票に行くことは有権者の義務である(本稿の分析に使用。)

・回答コード

1.思う「賛成」「どちらか言えば賛成」

2.どちらとも言えない「どちらとも言えない」

3.思わない「どちらかと言えば反対」「反対」

無風か非無風であるかどうかを判断する指標として、有権者が投

票する選挙区が接戦であるか無風であるかの質問とその回答

を用いる。(主観的)

Q3(事後)今回の選挙について、いろいろな感想や意見があっ

たと思いますが、これから読み上げるそれぞれについて、あ

なたはどう思いましたか。

1)〔回答票 6〕まず、この選挙区の選挙戦について、どう思いま

したか。

(ア) 以前より激しくなった (イ) 以前と変わらない (ウ) 以前よ

り無風状態となった 分からない 答えない

・回答コード

「選挙は激戦だったか」

1.以前より激しい「以前より激しくなった」

2.変わらない「以前と変わらない」

3.以前より無風状態「以前より無風状態となった」

94

<参考文献>

・山田真祐 1992「投票率の要因分析―1979 年~86 年総選挙

―」『選挙研究』所収.

・『政治参加』蒲島郁夫 1988. 『政治参加』東京大学出版会.

・佐賀全県 1地方 朝刊 (2011 年 04 月 12 日).

・水崎節文・森裕城 2007.『総選挙の得票分析 1958-2005』木鐸

社.

・蒲島郁夫 1998.『政権交代と有権者の態度変容』 木鐸社.

・三宅一郎 1995.『日本の政治と選挙』 東京大学出版会.

・三宅一郎・西澤由隆「日本の投票参加モデル」1997.綿貫譲治・

三宅一郎『環境変動と態度変容』 木鐸社.

・綿貫譲治・三宅一郎、猪口孝・蒲島郁夫 1986. 『日本人の選

挙行動』東京大学出版会.

・小林良有彰 2000.『選挙・投票行動(社会学科の理論とモデル

-1)』 東京大学出版会.

95

13 なぜ職業が投票政党

を規定するのか 山岸 未加

1. はじめに

一般に、「有権者の社会的属性の中でも、職業はその支持政党や投票政党と特に明確な関連を示している」(池田 2007,31)と言われており、それには「それぞれの職業に就いている者(あるいはそうした職業に就いている者によって家計を支えられている者)は、自分の職業の利益を推進してくれると考えられる政党に投票するという職業利益代表モデル」(三宅 1985)という説明がある。

私は、2005 年の衆議院総選挙における有権者の職業別関心事項を明確にし、今回の選挙にも職業利益代表モデルの説明が適用できるかどうかを検証したい。なぜなら、2005 年の選挙で「郵政民営化の単一争点に絞り、それを認めるか認めないかという議題設定を小泉は試みた。そしてその争点を改革を進める象徴だとシンボリックにフレーミングした」(池田 2007,28)といわれるほどに小泉は郵政民営化を主張し、マスメディアを利用して有権者にも広く知らしめた。「05 年総選挙において『郵政民営化』は有権者の投票行動に大きな影響を与えていた。しかもそれは争点投票のロジックによく一致するような形での影響の仕方であった。これは、この争点が有権者にとってセイリエントであり、各政党の立場も分かりやすく、過度にイデオロギー的ではなかったことによるものと思われる。…05年総選挙における郵政民営化問題は、具体的な政策争点というよりは小泉首相に対する信任・不信任のシンボルとして機能していた」と池田も述べている。(池田 2007,137)つまり、2005 年の総選挙では「自民党=小泉=郵政民営化」という認識が有権者の中に広まり、小泉自身もそれを狙っていたと考えられる。

そこで、自民党が大勝したという事実を踏まえると、今回の選挙にも職業利益代表モデルが適用されたとすれば、自民党に投票した有権者はどのような点で自分の職業の利益を推進してくれると考えたのか、そのメカニズムを検証したいと考えた。とりわけ自営業とサラリーマンでは、その投票心理にどのような違いがあるのだろうか。

2. 仮説

私が検討するのは、郵政民営化がもたらす「小さな政府」に賛成する自営業者は、自民党に投票するという「民営化賛成仮説」である。

なぜ自営業者と限定したかというと、農業・自営業・管理職に

96

従事する者は「戦後日本における自民党の長期政権を構造的に支えてきた(農業・自営業は再分配依存セクターとして、自民党との間で再分配的利益と票の交換を行い、管理職は市場における強者の一員として、現存する政治・経済的な秩序の維持と再分配用の資源および票の交換を行う)」(池田 2007,31)といわれており、自営業者の投票行動が職業利益代表モデルにもっとも当てはまると考えたからである。

では今回の選挙で、自営業者にとって自民党が与える再分配的利益とは何であろうか。

私は、郵政民営化による「小さな政府」が地方経済を活性化するという仕組みが、自営業者にとっての利益であると考える。よって郵政民営化に賛成する自営業者は自民党に投票するという「民営化賛成仮説」を立てる。

その関係を図示すると以下のようになる。

3. 実証

3-1. データベース JESⅢ「第 8250 号 衆議院選挙面接調査(事後)」(平成 17 年 9

月) のデータを用いて分析を行い、仮説の検証を行う。1

3-2. 分析手順 ① まず、「どの政党に投票しましたか」(投票政党)という質

問と、「あなたの職業についてお伺いします」(職業)というフェイスシートをクロス表に整理する。投票政党に関する質問については「自民党=1,その他=0」、職業に関する質問については「自営業=1,その他=0」と再コード化する。

② 次に、「この中にある具体的な争点について、最もふさわしい政策を持っているのはどの政党だと思いますか」(ふさわしい政党)という質問の中の郵政民営化の項目と、「あ

郵政民営化への賛否

自営業/その他

自民党に投票

97

なたの職業についてお伺いします」(職業)というフェイスシートをクロス表に整理する。ふさわしい政党に関する質問については「自民党=1,その他=0」、職業に関する質問については「自営業=1,その他=0」と再コード化する。

③ 投票政党、ふさわしい政党、職業に関するそれぞれの質問をクロス表に整理し、検証結果の総括として考察する。

3-3. 実証

表 1:投票政党と職業の関係

職業(%)

投票政党(%

)

自営業 その他 合計(%) N

自民党 51.1 44.1 54.5 724

その他 48.9 55.9 45.5 604

合計(%) 100 100 100 1328

カイ二乗値 4.303 自由度 1 危険率 0.038

この表は投票政党と職業の関係を表したものである。

この表から分かることは、自営業者のうち 51.1%が自民党に投票し、その他(サラリーマン、主婦、無職等)層のうち 44.1%が自民党に投票した。つまり、その他の層よりも自営業者のほうが 7%多く自民党に投票したということである。

ところで、従来から自民党票を支えてきた自営業者の票がその他の層とさほど変わりなく反映されたのは、今回の選挙の特徴とも言える“外からの支持層”の影響と考えられるだろう。従来から自民党を支えてきたのは自営業者の票であるが、それだけでは今回の大勝は成し得なかった。これまで他政党に流れていたその他(サラリーマン、主婦、無職等)の層を自民党に振り向かせたのは、やはり郵政民営化の影響だろうか。その要因は下記表 2 で検証する。

98

表 2:郵政民営化への賛否と職業の関係

職業(%)

郵政民営化の政策がふさ

わしい政党(%

)

自営業 その他 合計

(%) N

自民党 91.4 86.5 87.6 1008

その他 8.6 13.5 12.4 143

合計(%) 100 100 100 1151

カイ二乗値 4.147 自由度 1 危険率 0.042

この表は、郵政民営化について最もふさわしい政策を持っている政党に関する回答と、職業の関係を示す表である。

この表から分かることは、自営業者のうち 91.4%が、郵政民営化の政策について自民党がふさわしいと考えており、その他(サラリーマン、主婦、無職等)層のうち 86.5%も郵政民営化の政策について自民党がふさわしいと考えているということである。この結果は表1で推測した通り、その他(サラリーマン、主婦、無職等)層でも、その多くが郵政民営化について自民党がふさわしいと感じ、それがそのまま自民党への投票に繋がったと考えられる。

表 3:投票政党と郵政民営化への賛否、および職業の関係

職業(%)

郵政民営化の政策が

ふさわしい政党(%) 自営業 その他 合計(%) N

自民党 自民党に投票 55.7 52.4 53.2 489

他政党に投票 44.3 47.6 46.8 431

合計(%) 100 100 100 920

その他 自民党に投票 22.2 10.4 12.0 16

他政党に投票 77.8 89.6 88.0 117

合計(%) 100 100 100 133

カイ二乗値 78.731 自由度 1 危険率 0.000

99

この表は、投票政党と郵政民営化の政策についてふさわしいと考える政党、および職業の関係を表したものである。この表が示すものが私の仮説と一致していれば、つまり「郵政民営化に賛成する自営業者は、自民党に投票する」という結果が見られれば、この表は仮説の実証に大いに役立つものである。

この表から分かることは、郵政民営化について自民党がふさわしいと考える自営業者は 55.7%が自民党に投票し、他政党がふさわしいと考える自営業者は 77.8%が他政党に投票するということである。また、郵政民営化について自民党がふさわしいと考えるその他(サラリーマン,主婦,無職)層は 52.4%が自民党に投票し、他政党がふさわしいと考えるその他(サラリーマン,主婦,無職等)

層は 89.6%が他政党に投票するということである。つまり、郵政民営化に関して自民党に賛成する自営業者は自民党に投票し、郵政民営化に関して他政党に賛成するその他(サラリーマン,主婦,無職等)層は他政党に投票するという「民営化賛成仮説」(下図参照)

は実証されたということである。そして、それ以上に、郵政民営化に関して他政党がふさわしいと考えている人はかなりの割合で他政党に投票しているという結果に注目したい。自営業者でさえ、郵政民営化に関して他政党がふさわしいと考えとそのうち 77.8%

が他政党に投票している。郵政民営化に関して他政党がふさわしいと考えるその他(サラリーマン、主婦、無職等)層のうち 89.6%が他政党に投票している。郵政民営化に関して自民党がふさわしいと思っていない有権者は、かなりの割合で他政党に投票したということが分かる。

しかしこの結果を逆手にとってもう尐し詳しく見てみると、郵政民営化に関して自民党がふさわしくないと思っている自営業者でも、22.2%は自民党に投票している。従来からの自民党支持層は郵政民営化だけでは自民党支持を外したりしないということだろうか。一方で郵政民営化に関して他政党がふさわしいと思いながらも自民党に投票するその他(サラリーマン、主婦、無職等)層は10.4%である。自営業者のほうが 11.8%も多い。

このことは、「換言すれば、小泉に期待を感じる人が自民党に投票し、期待しない人は自民党には投票しない、という傾向がより明瞭になり、期待を焦点に票がより大きく割れた」と池田が述べていることの証明ともいえるだろう。(池田 2007,37)

しかしこれらの結果を踏まえても、自民党が大勝したという事実は変わらない。この事実に関しては、一つの文献を参考にしたい。

「2005 年には投票行動の形は再び変化した。自民党が圧勝した

郵政民営化に賛成する自営業者 自民党に投票

郵政民営化に反対するその他の層 他政党に投票

100

ことは現象面での変化であるが、業績評価・将来期待の効き方にも変化が生じた。…将来期待の効果が業績評価をはっきりと上回っていた。そしてその効果が過去の投票歴をコントロールしてもロバストに効いていた点で、2003,2004 年の選挙で観察されなかったことが生じている。つまりこの年、小泉は新しく外側から自民党への投票者を引きつけることに成功したのである。…そして期待を小泉に感じた人は、自民党支持を越えて、今までより確実に自民党に票をもたらしたのである」(池田 2007,37)

つまり、2005 年の総選挙に関しては業績評価よりも将来期待の様相が高まり、従来の自民党支持を越えて、小泉は”外からの投票者”を引きつけることに成功した。これが、自民大勝に繋がったもっともの要因である。

4. 考察

表 1~表 3 の分析結果により、「郵政民営化に賛成する自営業者は自民党に投票する」というモデルを実証することができた。

さて、さらに考えるべきはその要因についてであるが、なぜ自営業者は郵政民営化に賛成し、自民党に投票したのだろうか。郵政民営化が自営業者にとって何らかのメリットがあると彼らは本当に考えたのだろうか。2005 年当時、我が国の経済状況は決して良いものではなかった。国民の間では 05 年選挙の争点は「郵政民営化」に加え、年金問題、消費税増税等による財源確保の問題も懸念されていた。年金問題は半ば世代間の問題とも言えるが、消費税増税等による財源確保の問題は国民全体に均等に負担が掛かるとはいえ、実際に消費税増税となると自営業者にとっての負担が大きい。出来ればその他の手段で財源を確保し、再建してもらいたいと考えるのが普通だろう。そのような状況下で小泉首相が提示したのが「郵政民営化」である。”大きな政府から小さな政府へ”、”民間にできることは民間に”などのキャッチフレーズとともに郵便事業の民間委託が掲げられた。これによるメリットは”民間にできることは民間に”任せることで地方経済が活性化し、自営業者の市場が潤うということであった。何より、自営業者にとっての痛手である増税案が後回しになる。

これらはあくまで推測であり、自営業者の票が自民党支持につながる明確な根拠ははっきりしていない。三宅も「自分で自分の仕事をコントロールすることができるという一種の利害感覚であり、農業従事者,自営業者,管理職などが持つことができ、自民党への支持や投票に結びつくものである」(三宅 1985)という「自前意識仮説」を提唱しているが、「なぜそうした『自前意識』が自民党支持に結びつくのかについての政治心理学的 ロジックは必ずしも明確ではない」(池田 2007,31)と批判している。

しかし、2005 年の衆議院総選挙においていえることは「それぞれの職業に就いている者(あるいはそうした職業に就いている者によって家計を支えられている者)は、自分の職業の利益を推進してくれると考えられる政党に投票するという職業利益代表モデル」(三宅 1985)は適用されたということである。このモデルが適用されるには、何より有権者に各政党の争点が認知されていなければならない。その点について 2005 年衆議院総選挙は、マスメディアなどの媒体を活用し、小泉首相によってより広く人々に知らされていたのではないだろうか。

101

5. おわりに

「はじめに」でも述べたように、一般に「有権者の社会的属性の中でも、職業はその支持政党や投票政党と特に明確な関連を示している」(池田 2007,31)と言われているが、その相関には年齢・世代や年収、経営規模や労働規模等、さまざまな統制変数が複雑に関係しあっており、「自営業かサラリーマンか」といった単純な分類では見えてこないものがたくさんある。しかし、「自営業あるいはその他(サラリーマン、主婦、無職等)の層」という単純な分類を丁寧に分析し、その結果を推測も含めて論理的に考察できたことは、来年の研究にも大いに役立つことではないだろうか。

現代社会はパラダイムシフトと言われ、これまで想像もつかなかった革新や変動の時代であるといえる。従来から自民党票を支えてきたといわれる自営業者の票も、“2005 年総選挙、郵政民営化”という革命によって、“外からの支持層”をも含む支持基盤へと変化していった。もちろん、この後に民主党が政権与党となることも、誰も予知してなかっただろう。何が起こるかわからない時代であるからこそ、もっとも基礎的な、単純な分析を何度も練り直し、考察することが大切なのだと、このレポートを作成するにあたって気付かされた。

(1) ここで使用したデータは JESⅢである。JESⅢは、平成 14

~18 年度文部科学省研究費特別推進研究「21 世紀初頭の投

票行動の全国的・時系列的調査研究」に基づく「JESⅢ研究

プロジェクト」(参加者・池田謙一:東京大学教授、小林良

彰:慶應義塾大学教授、平野浩:学習院大学教授)が行った

研究結果である。それをレヴァイアサン・データバンクよ

り同志社大学が代表して利用申請したものを西澤由隆先生

のご指導と便宜の下、利用させていただいた。この場を借

りて、それぞれのデータを公開・寄託され、利用できるよ

うにしてくださった先生方に感謝いたします。

<補遺>

○分析に使用した質問文と回答コード

・SQ7.「どの政党に投票しましたか。」

1 自民党 2 民主党 3 公明党 4 社民党 5 共産党 6 国民新党 7 新党日本 8 × 9 その他 10 わからない 11 答えない

∇回答コード・・・自民党=1,その他=0

(8,9,10,11 は欠損値扱い)

・Q9. 「今回の選挙では、各党の公約として『マニフェスト(政権公約)』が発表され、そこで約束された政策が争点となりました。この中にある具体的な争点として、最もふさわしい政策を持っているのはどの政党だと思いますか。1つだけお答えください。」(郵政民営化の欄のみ使用)

102

1 自民党 2 民主党 3 公明党 4 社民党 5 共産党 6

国民新党 7 新党日本 8 その他 9 そういう政党はない・どれも同じ 10 わからない 11 答えない

∇回答コード・・・自民党=1,その他=0

(8,9,10,11 は欠損値扱い)

・F5. 「あなたのご職業についてお伺いします。お勤めですか、自分で事業をしていらっしゃるのですか、それとも家の職業(仕事)を手伝っているのですか。(仕事を持っていない場合)学生ですか、専業主婦ですか。

1 勤め 2 自営業(含自由業) 3 家族従業 4 学生 5 専業主婦 6 無職 7 その他 8 答えない

∇回答コード・・・自営業,家族従業=1,その他=0

(答えないは欠損値扱い)

<参考文献>

・池田謙一 2007 『政治のリアリティと社会心理 平成小泉政治のダイナミックス』木鐸社

4 年生論文

104

14 有権者の

メディア効果意識 メディアの影響を感じているのは誰か

西山 千尋

1. はじめに

メディアは物事のある側面だけを映し出しているのではないかと感

じることがある。あるニュース番組で図 1のグラフを見たのだが、回答

者の年齢分布や所得・家族構成などは明記されていないにもかかわらず、

あたかも全国民のほぼ半数が年金額引き下げを支持しているように報

道されていることに私は違和感を抱いた。このようにメディアの報道内

容に対して疑問を覚えることは、以前からもしばしばあった。

しかしメディアは人々に対して確実に影響力を持っていると考えら

れる。グルメ番組で取り上げられたレストランには翌日行列ができ、健

康番組で体に良いとされた食品は即売り切れるといった現象がよく見

られるからである。メディアが人々に与える影響について、内田氏は「メ

ディア・リテラシーの低い読者はいるが、メディアの下す人物評を一〇

〇%真に受ける『メディア・リテラシー・ゼロ』の読者なんていやしな

い。」(内田 2008年、121)と述べている。確かにこれは事実であろう。

しかし選挙や外交問題といった情報の多くをメディアに頼る事柄に関

しては、人々はメディアの影響を非常に強く受けるのではないだろうか。

本論文ではメディアの受け手である有権者に焦点を当て、「誰がメデ

ィアの影響を感じているのか」ということを実証したい。有権者のみに

焦点を当てたのは、メディア効果の実証に必要な「メディアの内容」「送

り手の意図」「受け手の感じ方や行動」の 3 つ全てを正確に測定するこ

とが大変困難であるからである。そこで「あなたの意見は、新聞やテレ

ビの影響を受けていると思うか」という問いを利用することによって、

尐なくともメディアが有権者に影響を感じさせる要因について確認し

たい。

有権者が自己へのメディア効果を感じること(以後「メディア効果意

識」とする)の要因として、メディアへの接触量、意見形成能力の有無、

メディアへの信頼度が考えられる。本論文では重回帰分析を行うことに

より、これらの要因がメディア効果意識に与える影響について実証した

い。

105

図1 年金額引き下げについて

出所:「日本テレビ調査社 定例世論調査」

2. 先行研究

本論文では「メディアの影響を受けていると感じているか」というメ

ディアの認知レベルの影響について確認したい。メディアの認知レベル

影響を肯定する研究報告の1つに竹下の 1982年、1986年の研究が挙げ

られる。

竹下は「メディアの議題設定力は受けて全員に一様に作用するもので

はなく、特定の条件を備えた人に対して、より顕著な効果をもたらす」

(竹下 2008)という仮説に基づき、メディアの議題設定能力の検証を

行った。1

1982 年の研究では、新聞の議題設定効果は政治記事への接触頻度が

多い人や、政治への関心度の高い人に顕著に現れること、テレビの議題

設定力は新聞と比べると低いが、政治関心のもっとも高い人には比較的

強い効果を持つことが実証された。新聞に比べテレビの議題設定力が务

っている理由として、分析対象番組の選定が適切とは言えなかった点、

メディアの内容分析に用いた争点の次元や性質が異なっていた点を竹

下は挙げている。

また 1986 年の研究においては、メディアの議題設定効果に対し

抵抗力を持つ比較的尐数のグループの存在が示される。この「ハードコ

ア」グループは、選挙報道への注目度が極めて高く、メディアの高度利

用者でありながらも自身の判断基準を持っているためその影響を受け

にくいと考えられる。

支持する

支持しない

わからない・答えない 40.4%

49.3%

10.3%

106

3. 仮説

3-1. メディア接触量仮説

メディア接触の程度は、有権者の自己へのメディア影響の感じ方に大

きく影響するだろう。メディア接触については2つの側面から見る必要

がある。すなわち以下の2つの接触量である。

・相対的接触量:有権者が選挙情報を入手する全手段に占める「新聞

接触」「テレビ接触」の割合。

・絶対的接触量:選挙情報を入手する際に有権者が新聞・テレビに接

触した量の総和。

絶対的接触量の多い有権者ほど自己へのメディアの影響を感じるだ

ろう。また相対的接触量の大きい有権者、情報の多くをメディアに依存

しているためメディアの影響を感じやすいと推測できる。

3-2. 意見形成能力仮説

政治におけるマス・メディアの役割として世論形成機能が挙げられる。

マス・メディアは「科学的な世論調査の報道のみならず、というよりむ

しろ一般的には、街頭インタビュー、読者・視聴者からの投書・反応、

あるいは記者の分析の中で、世間の反応の大まかな傾向を示す」という

機能を持つ(蒲島・竹下・芹川 2010、121)。

そのため自己意見形成能力を持つ有権者は、「世間の反応」であるマ

ス・メディアの影響を受けにくいと考えられる。具体的には、高学歴者・

管理職者・政治的関心の高い有権者・他者への不信が強い有権者などが

考えられる。高学歴者や政治的関心の高い有権者は、世間(自己より無

知な者というニュアンスを含む)の反応よりも、自分の知識に基づき政

治的意見を形成すると考えられる。管理職者は、人に指示を出す立場で

あり日頃から周囲の意見ではなく自分の考えで動くことが要求されて

いる。そのため政治的意見形成においても、世間に大きく左右されるこ

となく自分の知識や経験に基づいていると推測できる。また他者への不

信が強い有権者は、世間の反応よりも自己の考えを重視して意見形成を

行うため、メディアの影響は低いと考えられる。

3-3. メディアへの信頼仮説

メディアへの信頼度が高い有権者は、メディアの情報を素直に受け

入れるため、メディアの情報に疑問を抱きながら接触を行う有権者に比

べ、メディアの影響を強く受けると考えられる。

信頼には「能力に対する期待としての信頼と、意図に対する期待とし

ての信頼」(山岸 1988、35)の2種類がある。有権者がメディアに対

して、役割を遂行する能力がないと考えているとは予測しがたいため、

ここでいう信頼とは「相手の意図に対する期待としての信頼」(山岸、

1998、35)つまり、有権者に嘘をつくことでメディアが利益を得ること

はないと考えていることだといえる。そのためメディアを信頼する有権

107

者はメディアの報道を疑いなく受け取り、メディア影響を受けやすいと

推測できる。

図2 モデル図

4. 分析

4-1.使用データ

本分析ではJESⅢ(2005年衆議院選挙)を使用する。2 JESⅢは研究

が比較的新しく、また有権者の政治に対する考えや他者との関係等が詳

しく質問されているため、本分析に適していると考えた。

4-2.変数の作成

■相対的接触量

相対的接触量の作成には、「あなたがふだん、政治についての情報を

見たり来たりするメディアはどれですか。」という質問および、「今年

の8月以降、あなたは、選挙に関することに参加したり、働きかけを受

けたりしましたか」という質問を使用する。3

ただし、変数の作成にあたっては「政治情報取得手段への接触量」で

はなく「情報取得手段の種類」に着目する。新聞やテレビに関する質問

が複数選択肢かつ複数回答可能である。それに対してその他の情報取得

手段に関する質問は該当・非該当の2択である(図3・4参照)3。そのた

め両者を単純に比較するのは不適切であると考えたからである。

具体的な手順は次のとおりである。「新聞を1紙以上読んでいる有権

者=1」「全く読まない有権者=0」と置き換え「新聞ダミー」を作成す

る。同様の手順を用いて「テレビニュースを1番組でも見ている有権者

=1」「見ていない有権者=0」と置き換え「テレビダミー」を作成する。

そして「その他の情報取得手段」の8項目に関しても1政党でも該当と選

択していれば「1」、全く接触を行っていなければ「0」と置き換えダミ

ー変数を作成する。「新聞ダミー」「テレビダミー」「その他の情報手

段の8つのダミー」の全てを足し合わせたものが「全政治情報取得手段」

である。「相対的接触量:新聞」とは「全政治情報取得手段」を分母に、

「新聞ダミー」を分子にしたものであり、0、1、1/2、1/3、1/4、1/5、

1/6、1/7、1/8、1/9 の10段階の値をとる。同様に、「相対的接触量:テ

メディアへの接触

(相対的・絶対的)

メディアへの信頼

メディア効果意識 意見形成能力

108

レビ」とは「全政治情報取得手段」を分母に、「テレビダミー」を分子

にしたものであり、10段階の値をとる。

出所:JESⅢ 2005年衆議院選挙面接調査(事前)

図3 メディア接触量(新聞・テレビ)

109

■絶対的接触量

絶対的接触量の作成については、相対的接触量と同様に「あなたがふ

だん、政治についての情報を見たり来たりするメディアはどれですかま

ず新聞についてはどうでしょうか」「次にテレビ番組などについてはど

うでしょうか」という質問を使用する。

ここで表される絶対的接触量とは、有権者が日頃から見ている新聞紙

(テレビニュース番組)の種類の多さを意味する。上記仮説に忠実な変

数を作成するには、有権者は新聞のどの記事を読んでいるか、1日何時

間テレビニュースを視聴するか、といったメディア接触頻度を考慮しな

ければならない。しかし使用するデータに適切な質問文がなかったため、

次善の策として「読んでいる新聞の種類の多さ」「見ているテレビニュ

ース番組の種類の多さ」を絶対的な接触量とみなす。

■教育程度

教育程度の変数作成には「あなたが最後に卒業された学校はどちらで

すか」という質問を使用する。変数は「.新中学校・旧小・旧高小=1」

「.新高校・旧中学=2」「.高専・短大・専修学校=3」「.大学・大学院=

4」とコード化する。

図4 メディア接触量(その他の情報入手手段)

出所:JESⅢ 2005年衆議院選挙面接調査(事後)

110

■年齢(管理職者)

JESⅢの質問からは有権者が管理職者であるかどうかを区別するこ

とが困難であるため、管理職者を「年齢」の変数に置き換える。これは

日本には年功序列制という雇用の特徴があるため、管理職者と年齢には

密接な関係があると考えられるからである。

年齢の変数作成には「あなたは何年何月何日生まれですか、満でおい

くつですか。」という質問を利用する。分析には質問に正確に答えてい

るもののみを扱い、「5」「15」など不明な数字を答えているものは全

て欠損値とする。

■政治への関心度

政治への関心度には「選挙のある、なしに関わらず、いつも政治に関

心を持っている人もいますし、そんなに関心を持たない人もいます。あ

なたは政治上のできごとに、どれくらい注意を払っていますか」という

質問を使用する。変数は「ほとんど注意を払っていない=1」「あまり注

意を払っていない=2」「やや注意を払っている=3」「かなり注意を払っ

ている=4」とコード化する。

■他者への信頼度

他者への信頼度は、「あなたは次にあげることがらについて、どう思

いますか。あなたのお気持ちに最も近いものをそれぞれ1つだけお答え

ください」という質問の中にある以下の5項目への回答を使用する。

(1)ほとんどの人は信頼できる

(2) たいていの人は、人から信頼された場合、同じように相手を信頼す

(3) 自分は信頼できる人と信頼できない人を見分ける自信がある

(4) 人を助ければ、いずれその人から助けてもらえる

(5) 人を助ければ、今度は自分が困っているときに誰かが助けてくれ

るように世の中はできている

他者への信頼度とはこれらの質問への回答の総和である。5項目は何

れも5段階評価で回答されるため、各項目について「そう思わない=0」

「どちらかといえばそう思わない=0.25」「どちらともいえない」=0.5」

「どちらかといえばそう思う=0.75」「そう思う=1」とした5段階尺度

の変数を作成する。このようにして作成した5つの変数を足し合わせた

ものを「他者への信頼度」とし、この変数は0~5の値をとる。

■メディアへの信頼度

メディアへの信頼度は「あなたは、この中にある組織や団体について、

どの程度信頼していますか。『0点』を信頼していない、『10点』を信

頼しているとした場合の点数でお答えください」という質問を使用する。

111

図5 メディアへの信頼度

出所:JESⅢ 2005年衆議院選挙面接調査(事後)

この質問は新聞紙では全国5紙プラス地方紙、テレビでは主要なテレビ5

系列について10段階で信頼度を尋ねている(図5参照)。

本論文では最も信頼できると評価した新聞紙への点数を「メディア信

頼度:新聞」、最も信頼できると評価したテレビへの点数を「メディア

信頼度:テレビ」とする。これは1紙でも強く信頼している新聞(テレ

ビ)があれば、その影響を強く受けると考えたからである。例えば2人

の有権者、AさんとBさんがいると仮定する。

・Aさん:5つのテレビ系列全て5点

・Bさん:1つのテレビ系列のみ10点、その他4つのテレビ系列は2点

2人の信頼度の合計を比較するとAさんが「25点」Bさんが「18点」

であり、Aさんの方が高い値をとっている。しかし、特定のテレビ局の

情報を疑いなく聞き入れるという点でBさんの方がメディアの影響を強

く受けていると推測できる。そのため最高評価の新聞紙・テレビへの値

を「メディアへの信頼度」として使用する。

112

■メディア効果意識

従属変数である「メディア効果意識」の作成については、「郵政民営

化問題に対するあなたのご意見は、新聞やテレビの影響を受けていると

思いますか。この中から1つお答えください。」という質問を利用する。

変数は「ほとんど影響を受けていない=1」「あまり影響を受けていな

い=2」「あまり影響を受けていない=3」「かなり影響を受けている=

4」とコード化する。

■コントロール変数

メディア効果意識に影響を及ぼしている要因は、仮説で提示した要因

の他にも様々なものがあるだろう。そのため本分析では「支持政党の有

無」「争点の重要度」「性別」をコントロール変数としてモデルに加え

る。コントロール変数を加えることにより、これら3つの影響を除外し

た上での「メディアへの接触量」「意見形成能力」「メディアへの信頼」

が従属変数に与える影響を検証することができる。

4-3 回帰分析

これらの変数を用いて重回帰分析を行う(表1)。回帰式は以下の通

りである。

(メディア影響認識)=(定数)

+b1(相対的接触量)

+b2(絶対的接触量)

+c1(教育程度)

+c2(年齢)

+c3(政治への関心度)

+c4(他者への信頼度)

+d(メディアへの信頼度)

+e(政党支持の有無)

+f(争点の重要度)

+g(性別)

113

表1 回帰分析

定数=1.691 調整済みR2=.058

5. 考察

5-1. メディア接触量仮説について

仮説では相対的接触量、絶対的接触量は共にメディアの影響認識にプ

ラスの効果を及ぼすと推測していたが、表1の各説明変数の危険率が非

常に高いことから仮説は統計的に有意でないことが分かる。4

仮説が実証されなかった要因として「変数の作成手順」が考えられる。

この仮説の変数は、変数を作るうえで適した質問項目が見つからず、他

項目で置き換えることが多くった。そのため本来意図していたものとは

別の変数になってしまった可能性が高い。

5-2. 意見形成能力仮説について

仮説では教育程度、年齢(40~50代)、政治への関心度はメディアの

影響認識にマイナスの効果を及ぼし、他者への信頼度はプラスの効果を

及ぼすと推測していた。

「政治への関心度」は危険率が.001 であり、1%水準で統計的に有意

であるといえる。また β値が.109であることから、他の説明変数に比べ

従属変数への影響力が大きいこと、しかし影響の方向が仮説とは反対で

あることが読み取れる。5 すなわち、政治への関心度が高い有権者ほど

従属変数:メディア効果意識

係数 β 危険率

相対的接触量: 新聞 .007 .001 .971

テレビ -.027 -.006 .848

絶対的接触量: 新聞 -.055 -.043 .252

テレビ .013 .041 .160

教育程度 -.030 -.033 .306

年齢 .003 .058 .082

政治への関心度 .122 .109 .001

他者への信頼度 .013 .013 .639

メディアへの信頼度: 新聞 .004 .012 .722

テレビ .036 .075 .017

支持政党の有無 .050 .022 .453

争点の重要度 .149 .133 .000

性別(女性) .098 .054 .067

114

自意見へのメディアの影響を感じているのである。

仮説が実証されたかった要因として、政治関心が強くメディア効果に

抵抗力をもつ、いわゆる「ハードコア」層の人数が尐ないことが考えら

れる。1986年の竹下の研究では、選挙関心が高い有権者の中でも、特に

選挙報道への接触度が高い有権者(ハードコア層)は、メディアの影響

を受けにくいことが示されている。本論文では「政治関心度」という1

変数のみでの分析であったため、このハードコア層を見落としてしまっ

たと考えられる。

5-3. メディアへの信頼仮説について

仮説では、メディアへの信頼はメディアの影響認識にプラスの効果を

及ぼすと予測した。

危険率を見ると、「メディアへの信頼度:新聞」は.722であるため統

計的に有意であるとはいえない。一方「メディアへの信頼度:テレビ」

の危険率は.017であるため、この結果は5%水準で統計的に有意である

といえる。「メディアのへの信頼度:テレビ」は、βの値より従属変数

に正の効果を及ぼしており、特にテレビへの信頼はメディアの影響認識

に比較的大きな影響を及ぼしていることが分かる。以上の事から「メデ

ィアへの信頼度が高い有権者ほど、自意見へのメディアの影響を強く感

じる」という仮説はテレビに限り支持される。

5-4. コントロール変数について

「争点の重要度」は従属変数に正の影響を及ぼしており、この結果は

1%水準で統計的に有意であるといえる。「争点の重要度」のβの絶対値

は本分析モデル中で最も大きく、メディア影響認識に与える影響が高い

ことが読取れる。

5-5 R2値について

表1のモデルの調整済みR2が.058と低い値をとっていることから、

本論文での説明変数は「従属変数:自意見へのメディア影響認識」の一

部分のみを説明していることが読み取れる。

6.まとめ

本論文は、JESⅢの2005年衆議院選挙のデータを用いて、自己へのメ

ディア影響を感じるのはどのような有権者であるかを検証した。

仮説は「メディア接触量仮説」「意見形成能力仮説」「メディアへの

信頼仮説」の3つであり、ここでいうメディアとは新聞紙及びテレビの

ニュース番組を示している。「メディア接触量」では、全ての選挙情報

手段と比較したときのメディアへの依存度を示す「相対的接触量」と、

メディアにどれだけ接触しているかを示す「絶対的接触量」の2つの接

115

触量を、それぞれ新聞とテレビについて変数を作成した。「意見形成能

力」とは、周りの意見に流されず自己意見形成を行う能力であり、「教

育程度」「年齢」「政治への関心度」「他者への信頼度」の4つの変数

を用いる。「メディアへの信頼は」新聞・テレビどちらも最も信頼する

新聞紙・テレビ局への評価を用いて変数作成を行った。

回帰分析を行った結果、(1)本分析においては「メディア接触量」廼従

属変数へ影響は見いだせなかったこと、(2)「意見形成能力仮説」は「政

治への関心度」に限り有意であったが、仮説と反し従属変数にプラスの

影響を与えていたこと、(3)「メディアへの信頼仮説」についてはテレビ

への信頼に関してのみ仮説が支持されたことが分かった。しかし調整済

みR2が.非常に低い値をとっているため、これらの要因はメディア効果意

識の一部のみを説明しているといえる。

本分析結果より「政治への関心度が高く、テレビを信頼しており、特

定の争点を重要だと認識している有権者」がメディアの影響を感じやす

いといえるが、この該当者は進んでメディアの影響を受けている有権者

であると筆者は予想する。なぜなら政治への高関心や争点の重要視は、

どちらも政治に対して積極的な姿勢を示しているからである。そしてこ

の層とは反対の、政治関心が低い有権者や選挙争点を認識していない有

権者はメディアの影響を受けているが無意識である可能性が高いと推

測できる。選挙への積極的な姿勢を持ち「メディアの情報がどのように

自分に影響したか」を客観的に把握する。数多の情報が飛び交う現代社

会では、このような有権者を育てていく必要があると筆者は考える。

(1) メディアの議題設定能力とは「マスメディアで、ある争点やトピ

ックが強調されればされるほど、その争点やトピックに対する人々

の重要性の認識も高まる」(竹下 2008、38)と定義され、1970年代

初頭にアメリカのコミュニケーション研究家であるマコームズとシ

ョーによって提唱された。

(2) なお、ここで利用したデータは JESⅢである。JESⅢは「21 世紀初

頭の投票行動の全国的・時系列的調査研究(JESⅢ SSJDA

版),2001-2005」は、JESⅢ研究会が実施された世論調査である。そ

の個票データについて、東京大学社会科学研究所付属 日本社会研究

情報センター・データアーカイブ(Social Science Japan Date Archive)

より教材としての利用許可(申請者:西澤由隆教授)を得たものを

使用した。本データは同志社大学・法学部の西澤由隆先生のご指導

と便宜により利用ができた。それぞれのデータを公開・寄託され、

利用できるようにしてくださった先生方に感謝いたします。

(3) JESⅢは有権者に質問カードを見せながら回答を得る形式のため本

来ならば質問カードを載せるべきであるが、質問カードの入手が不

可能であったため JESⅢの調査表より質問の構造を示した。

(4) 危険率は有意水準とも呼ばれ、説明変数が従属変数に与える効果が

116

偶然である確率を示す。一般的に 5%以下で統計的に有意であると

いわれる。

(5)βは標準偏回帰係数ともよばれ、各変数が従属変数に及ぼす影

響の大きさと向きを示している。

<補遺>

・質問票(図3・図4・図5の質問は省略している)

話は変わりますが、今回何党に投票するかは別にして、ふだんあなたは

何党を支持していますか。

1自民党 2民主党 3公明党 4社会党 5共産党

6 その他の政党 7支持政党なし

郵政民営化という問題は、あなたにとってどれくらい重要ですか。

1.かなり重要である 2.やや重要である

3.あまり重要ではない 4.ほとんど重要ではない

郵政民営化問題に対するあなたのご意見は、新聞やテレビの影響を受け

ていると思いますか。

1.かなり影響を受けている 2.やや影響を受けている

3.あまり影響を受けていない 4.ほとんど影響を受けていない

選挙のある、なしに関わらず、いつも政治に関心を持っている人もいま

すし、そんなに関心を持たない人もいます。あなたは政治上のでき

ごとに、どれくらい注意を払っていますか。この中ではどれにあた

りますか。

1.かなり注意を払っている 2.やや注意を払っている

3.あまり注意を払っていない 4.ほとんど注意を払っていない

あなたは次にあげることがらについて、どう思いますか。あなたのお気

持ちに最も近いものを、それぞれ1つだけお答えください。

(16)ほとんどの人は信頼できる

(17)たいていの人は、人から信頼され場合、同じように相手を信頼する

(18)自分は信頼できる人と信頼できない人を見分ける自信がある。

(19)人を助ければ、いずれその人から助けてもらえる。

(20)人を助ければ、今度は自分が困っている時に誰かが助けてくれるよ

うに世の中はできている。

1.そう思う 2.どちらかといえばそう思う

3.どちらともいえない 4.どちらかといえばそう思わない

5.そう思わない

117

あなたは何年何月何日生まれですか、満でおいくつですか。

あなたが最後に卒業された学校はどちらですか。

(中退・在学中は卒業とみなす。各種学校は含めない)

1.新中学校・旧小・旧高小 2.新高校・旧中学

3.高専・短大・専修学校 4.大学・大学院

<参考文献>

・内田樹 2008.『ひとりで生きられないのも芸のうち』文芸春秋.

・蒲島郁夫 1988.『政治参加』東京大学出版会.

・蒲島郁夫・竹下俊郎・芹川洋一 2010.『メディアと政治〔改訂版〕』

有斐閣

・鈴木裕久・島崎哲彦 2006.『新版・マス・コミュニケーションの調査

研究法』創風社.

・竹下俊郎 2008.『メディアの議題設定機能―マスコミ効果研究におけ

る理論と実証―〔増補版〕』学文社

・三宅一郎 1989.『投票行動』東京大学出版会

・山岸俊男 1988『信頼の構造』東京大学出版.

118

15 チェコにおける政治

関心

林 香織 廣田 奈央

1. はじめに

チェコ共和国での留学生活を経験して、チェコの人々は政治に高い関

心を持っていると感じた。私の知っている限り、日本の学生同士間では、

政治・経済等々の社会問題が日常会話の話題となることは滅多にない。

しかし、チェコにおいては政治・経済に関する時事問題もひとつの話題

としてよく取り上げられていたように感じた。

チェコ人の政治関心は下の図1から分かるように、比較的高いことが

分かる。

また政治への関心が高いことのほかに、人々のモチベーションの高さ、

および彼らのチャレンジ精神に驚いた。社会のなかでより上の立場につ

きたい、より大きな影響力を持ちたい、または、社会に大きく貢献した

いと考え、努力している人々が多いと強く感じた。

政治関心と同様、上昇志向を持つ人の割合も高いのだろうか。上昇志

向の判断基準として、向上心や人生の捉え方など、様々な価値観を挙げ

ることができるが、ここでは仕事を重要であると考えるかどうかを判断

基準とした。図 2より、生活において仕事を重要であると考える人の割

合は他国に比べチェコでは低いことがわかる。留学生活において、学生

以外と交流する機会はあまりなかったため、私の印象と実態は異なるの

かも知れない。

上昇志向が強い人々ほど政治への関心は高くなるのではないだろう

か。社会の中で活躍したいと考える人々にとって社会の動向を左右する

政治への関心が高いと考え、上昇志向が高い人ほど政治への関心が高い

と考える。また、民主化し 20年あまりの成長過程におけるチェコでは、

上昇志向が強く政治への関心も高いのではないだろうか。そこで本稿の

命題を「上昇志向の強い成長過程にある国家では政治関心が高い」と設

定し、チェコにおける政治関心と上昇志向に関係性があるのかどうかみ

ていきたい。

119

図 1:政治関心

図 2:上昇志向

2. 分析枞組み

2-1. 分析の流れ

「上昇志向が高ければ政治関心が高い」という本稿の命題を確認する

ために従属変数を政治関心とし、独立変数に上昇志向を示す指標を用い

た重回帰分析を行う。教育程度・民主化の歴史・生活水準をコントロー

ル変数として独立変数に投入する。また、国による違いも考慮するため、

国ダミー変数も追加する。

120

2-2. 使用するデータ

命題の検証を行うために、World Values Surveyの「FOUR WAVE

WVS/EVS OFFICIAL INTEGRATED DATASET, data from 1995」を

用いる。1 World Values Surveyは、世界中の国家および地域を対象に

人々の社会的・文化的・宗教的・経済的・政治的価値観調査する国際プ

ロジェクトである。本稿では、チェコにおける最新の調査(1998 年)

を含む 1995年度版を使用する。

2-3. 作業定義

(1)従属変数「政治関心」

政治関心の度合いを測定するにあたり、「あなたはどれくらい政治に

関心を持っているといえますか」という質問への回答を使用する。そし

て、その回答を「全く関心がない」を 1 点、「あまり関心がない」を 2

点、「関心がある」を 3点、「とても関心がある」を 4点と一点刻みにコ

ード化を行った。

(2)独立変数「上昇志向」

上昇志向が高いほど、政治への関心も高くなる。上昇志向を示すもの

さしとして、仕事に対する価値観を用いる。自分の目標を「仕事」を通

じて実現しようとすると考え、仕事の重要性によって上昇志向が高いか

どうかを判断する。

仕事の重要性の程度を測定するために、「あなたの生活において仕事

はどのくらい重要ですか」という質問への回答を使用する。そして、そ

の回答を「全く重要でない」を 1点、「あまり重要でない」を 2点、「重

要である」を 3 点、「とても重要である」を 4 点と一点刻みにコード化

を行った。

(3)コントロール変数

(3)-1 「教育程度」

教育程度が高いほど、政治への関心は高いと考える。教育は、ひとり

ひとりが社会で活躍するための知識を与え、またそのためのスキル及び

能力を高める重要な役割を担っている。すなわち、教育程度が高い人ほ

ど、社会に対する関心が高く、政治への関心も高いと考える。

(3)-2「民主化の歴史」

民主化を近年経験した国家は、国全体が新しい方向に進んでいる最中

であり、人々の政治関心が高いと考える。

そこで、本分析において 1973 年以前に民主化した国家を民主主義の

歴史が長い国とし、1974 年以降に民主化を経験した国家を民主化して

まもない国家とする。

1974 年を境に区切る理由は次の通りである。アメリカの国際政治学

者であるサミュエル・ハンチントンは、彼の著作である「The third

121

Wave: Democratization in the Late Twentieth Century」のなかで「間

接民主制に基づく自由民主主義の採用を民主化ととらえた場合歴史的

に 3つの民主化の波があった」(岩崎、岸川 2004 13)と論じている。

第一の波は、19世紀のはじめから 20世紀はじめの 100年間で、第二の

波は第二次世界大戦後の 20 年間である。そして、 「1974 年における

ポルトガルの独裁の終焉に続く 15 年間に、ヨーロッパ、アジア、そし

てラテン・アメリカにおけるおよそ 30 の国において民主主義体制が権

威主義体制(非民主主義体制)にとって変わった。」(S・P・ハンチント

ン 1995. 21)と主張し、1974年から 1990年までの間の民主主義への

体制移行のことを第三の波と称している。

本変数のコーディングは以下のとおりである。1973 年以前に民主化

をしている場合を 25点とした。そして、74年以後に民主化した国につ

いては、「調査年-民主化した年度」を点数化した。以上のように「民主

化の歴史」をコード化した。

(3)-3 「生活水準」

生活水準の低い国のほうが政治への関心が高いと考える。経済力が低

い国において、国民は自身の経済力と国の経済力をつなげて考えるので

はないか。そこで、国民一人当たりの GDP を上昇志向についての指標

として使用する。GDP はその国の経済の大きさを表す指標である。そ

して国民一人当たりの GDP は、一人がどれだけの価値を生み出したか

を示すと同時に各国民の生活水準を示す。よって、本稿では生活水準を

図る指標として、国民一人当たりのGDPを用いる。なお、データは 1999

年度のものを使用する。

3. 分析結果

上の作業定義に基づいて、分析を行った結果について述べたい。下の

表 1は、従属変数(政治関心)と、独立変数(仕事の重要性)の関係性

についてコントロール変数(民主化の歴史、一人当たり GDP、教育)

を加えて回帰分析を行った結果(国ダミー変数を除く)である。

表 1 分析結果

ベータ 危険率

仕事 0.021 0.000***

教育 0.203 0.000***

民主化の歴史 0.036 0.570

GDP一人当たり 0.688 0.000***

N=1617、調整済みR二乗値0.096

(***は危険率 1%以下を示す)

122

はじめに、仕事の重要性と政治関心についての分析結果をみる。表 1か

ら生活における仕事の重要度が大きい人ほど政治への関心が高いことが

わかる。危険率が 1%以下であることから、この分析は有意であるといえ

る。したがって仮説の通り、仕事の重要度が高いと強く感じる人ほど政治

への関心が高くなることがわかる。

次にコントロール変数の結果をみたい。まず、「民主化の歴史」につい

ての分析結果を見ていただきたい。危険率が 57%であり、有意な結果を

得ることができなかった。したがって、民主化の歴史と政治関心の関係性

についての関係を確認することはできなかった。

そして、生活水準と政治関心の関係について言及したい。危険率が 1%

以下であることから、この分析は有意である。分析の結果、ベータの値は

0.688 であった。一人当たりのGDP の値が大きいほど政治関心が高い場

合に、独立変数が従属変数に与える影響を示すベータの値は大きくなる。

このことから、一人当たりのGDPが高い国家のほうが政治への関心が高

いといえる。したがって、生活水準が低いほうが政治への関心が高いとい

う仮説とは異なる現象が起きていることがわかる。

教育程度については、以下の通りである。最終学歴が高いほど、政治へ

の関心が高い場合にベータの値は大きくなる。ベータは、0.203 であり、

危険率は 1%以下である。よって仮説の通り、教育程度が高いほど政治へ

の関心が高くなることがわかる。

4. 考察

分析の結果を以下にまとめた。上昇志向は政治関心を規定するひとつの

要因であることが分かった。上昇志向が高いほど、政治への関心度も高く

なる。また、教育についても上昇志向と同じことが言え、教育程度が高く

なるほど関心が高まることが分かった。生活水準については、仮説とは異

なる結果が得られた。生活水準が低いほど政治への関心が高くなるのでは

ないかと考えていたが、分析結果はそれとは逆であった。生活水準が高い

ほど関心が高くなる。そして、民主化の歴史については有意な結果を得る

ことができなかったため、歴史と関心の関係性について検証できなかった。

本稿において主題であるチェコにおける関心と上昇志向の関係そのも

のについての分析はできていない。その代替策として、「World Values

Survey」に含まれるチェコを含めた 40ヵ国のデータをもとに政治関心と

上昇志向の関係性を分析した。その結果上述したように、上昇志向があれ

ば、政治への関心が高まることが証明された。チェコを含む 40 ヵ国全体

において関心と上昇志向の間に相関関係がみられることから、チェコにお

いても上昇志向、教育程度、生活水準が政治関心に影響を与える要因の一

部であることがいえるだろう。

また、現在のチェコにおける政治関心および上昇志向は、本分析で用い

たデータが調査された 1999年と比べどう変わっているのだろうか。1999

123

年から現在に至るまで、チェコは EU へ加盟し、経済成長もしている。

そして人々はより豊かな生活を送っている。生活水準が高まれば政治への

関心も高くなることから、現在における関心は高まっているのかもしれな

い。

(1) World Values Survey 「FOUR WAVE WVS/EVS OFFICIAL

INTEGRATED DATASET, data from 1995」

(URL:http://www.worldvaluessurvey.org/)の公開データを利用した。

同志社大学・法学部の西澤由隆先生のご指導と便宜により利用がで

きた。それぞれのデータを公開・寄託され、利用できるようにして

くださったすべての方に感謝いたします。

〈補遺〉

年表(民主化の歴史)

1973年以前 74年

以降

アメリカ 1978 スペイン 1990 アルメニア

オーストラリア 1979

リトアニア

バングラデシュ 1980 ペルー 1991 ベラルーシ

チリ 1981

モルドバ

コロンビア 1982

ラトビア

ドミニカ 1983 アルゼンチン

ウクライナ

フィンランド 1984

ラトビア

インド 1985 ブラジル

ウクライナ

韓国

ウルグアイ 1992

メキシコ 1986 フィリピン 1993

ニュージーランド 1987

1994 エルサルバドル

ノルウェー 1988

1995

スイス 1989 アゼルバイジャン 1996

スウェーデン

ブルガリア 1997 南アフリカ

台湾

チェコ 1998 ナイジェリア

ベネズエラ

ハンガリー

ドイツ

ルーマニア

スロベニア

124

分析結果(国ダミー変数を含む)

B値 ベータ 危険率

仕事 0.02 0.02 0.000***

教育 0.08 0.20 0.000***

民主化の歴史 0.00 0.04 0.570

GDP一人当たり 0.00 0.69 0.000***

バングラデシュ 0.25 0.04 0.000***

アルメニア 0.23 0.04 0.066

ベラルーシ 0.18 0.03 0.056

ブルガリア 0.12 0.01 0.000***

グルジア 0.10 0.02 0.430

モルドバ 0.07 0.01 0.604

ラトビア 0.06 0.01 0.534

マケドニア 0.03 0.00 0.871

ウクライナ 0.01 0.00 0.922

インド 0.01 0.00 0.791

ウルグアイ -0.02 0.00 0.555

南アフリカ共和国 -0.03 -0.01 0.652

エストニア -0.04 -0.01 0.596

アゼルバイジャン -0.06 -0.01 0.597

リトアニア -0.07 -0.01 0.351

ナイジェリア -0.08 -0.01 0.000***

ルーマニア -0.09 -0.01 0.318

ロシア -0.12 -0.02 0.001***

ハンガリー -0.14 -0.01 0.003***

ドミニカ -0.17 -0.01 0.008***

スロバキア -0.17 -0.02 0.000***

ポーランド -0.23 -0.03 0.000***

スロベニア -0.25 -0.03 0.000***

メキシコ -0.25 -0.05 0.013

ペルー -0.30 -0.04 0.000***

韓国 -0.30 -0.04 0.061

ブラジル -0.31 -0.04 0.000***

コロンビア -0.52 -0.11 0.000***

ニュージーランド -0.59 -0.08 0.028

アルゼンチン -0.72 -0.09 0.000***

チリ -0.73 -0.09 0.000***

台湾 -0.75 -0.08 0.000***

125

オーストラリア -0.87 -0.15 0.006***

ベネズエラ -0.93 -0.12 0.000***

ドイツ -1.03 -0.13 0.010***

スペイン -1.31 -0.17 0.000***

フィンランド -1.58 -0.19 0.000***

スウェーデン -1.83 -0.22 0.001***

ノルウェー -2.09 -0.27 0.003***

アメリカ -2.12 -0.32 0.002***

スイス -2.61 -0.34 0.001***

〇分析に使用した質問項目

V117. How interested would you say you are in politics?

1 Very interested

2 Somewhat interested

3 Not very interested

4 Not at all interested

9 Don't know [DO NOT READ OUT]

V008 Important in life: Work Indicate how important it is in your life.

Would you say it is: Work

1 Very important

2 Rather important

3 Not very important

4 Not at all important

9 Don't know [DO NOT READ OUT]

V217 Highest educational level attained

What is the highest educational level that you have attained? (use

functional equivalent of the following, in given society; IF

STUDENT, CODE HIGHEST LEVEL HE/SHE EXPECTS TO

COMPLETE)

1 Inadequately completed elementary education

2 Completed (compulsory) elementary education

3 Incomplete secondary school: technical/vocational type/

(Compulsory) elementary education and basic vocational

qualification

4 Complete secondary school: technical/vocational type/Secondary,

intermediate vocational qualification

5 Incomplete secondary: university-preparatory type/Secondary,

intermediate general qualification

6 Complete secondary: university-preparatory type/Full secondary,

maturity level certificate

7 Some university without degree/Higher education - lower-level

tertiary certificate

8University with degree/Higher education - upper-level tertiary

certificate

126

9 Don't know [DO NOT READ OUT]

〈参考文献〉

・ジョセフ・ロスチャイルド1999.『現代東欧史 多様性への回帰』 羽

場久浘子 水谷驍訳 共同通信社

・恒川惠一 2008. 『比較政治 中南米』 放送大学

・サミュエル・ハンチントン 1995.『第三の波 20世紀後半の民主化』

中道寿一 藪野祐三訳 三嶺書房

・岸川毅 岩崎正洋編 2004.『アクセス 地域研究 民主化の多様な姿』

日本経済評論社

・財団法人 矢野恒太記念会編 2002.『世界がわかるデータブック

2002・2003世界国勢図解第13版』財団法人 矢野恒太記念会

127

16 選挙制度改革によっ

て候補者投票と政党

投票の効果はどのよ

うに変化したのか 廣田 奈央

1. はじめに

1994 年に選挙制度改革が成立し、1996 年の衆議院議員選挙で小

選挙区比例代表並立制が実施された。改革の柱となった「政治改革

大綱」と選挙制度審議会答申」によると、「候補者本位」の選挙に代

わる「政党本位」・「政策本位」の選挙をもたらすことを選挙制度改

革は 1 つの目標としていた。すでに選挙制度改革が実施されて 10

年以上経つので、その目標としていた効果が現れる時期である。そ

れにもかかわらず、選挙制度改革に関する研究において明確な効果

はあまり確認されていない。

たとえば、三宅(2001)は候補者投票・政党投票・政策投票の 3

要因投票決定モデルのそれぞれに注目し、選挙制度改革による効果

を明らかにしようと試みている。その試みの中で、1972 年から 1996

年の計 9 回にわたって、明るい選挙推進協会による衆議院選挙調査

を用いて、「政党重視」か「候補者個人重視」かの回答の比率を示し

た。中選挙区制下では、「政党」対「候補者」の比率は 5:4 で安定

しており、1996 年調査の小選挙区制下であっても 1:1 で釣り合っ

ていることを確認している。つまり、選挙制度改革によって期待さ

れた政党を重視するという傾向は見られず、選挙制度が変わっても

中選挙区制下のパターンとほとんど変化していない。これは、選挙

制度改革が目指したものとは異なる結果であろう。このように、選

挙制度が中選挙区制から小選挙区制になったからといってすぐに期

待される効果がはっきりと確認されたわけではない。

だが、本当に三宅(2001)の言うように、政党要因の重要性は選

挙制度改革を通じて変化していないのだろうか。ここ数年、政党を

中心とした選挙戦が展開されているように私は感じる。2009 年の衆

議院選挙で、民主党が第一党となる政権交代が起こって以来、自民

128

党と民主党という二大政党が選挙をリードする激しい選挙戦を繰り

広げている。また、選挙区ごとの選挙戦を見ても、候補者個人同士

の争いというよりもむしろ候補者の所属する政党同士の争いになっ

ているように感じる。誰が当選したかではなく、何党の候補者が当

選し、政党がどれだけ議席を獲得できたかということがより重要に

なっている。

このような政党中心の選挙戦や二大政党の現れは、選挙制度改革

の効果の 1 つとして浅野(2006)が掲げているものである。ここ数

年の選挙の様子を踏まえると、小選挙区制が導入されたことによっ

て政党中心の選挙戦や政権交代が現れたのではないだろうかと私は

考える。

それを確かめるために、本稿では、異なる選挙制度下における有

権者の投票行動に注目する。そしてその中でも、先の三宅(2001)

の 3 要因投票決定モデルのうち候補者投票と政党投票を取り上げる。

この 2 つに限定したのは、有権者が投票行動を決定する主な判断基

準となっていると考えるからだ。有権者は、「候補者」と所属してい

る「政党」を承知の上で投票するだろう。また、投票に行く有権者

であれば、政党についてもある程度イメージや評価があるだろう。

以上のことを踏まえて、本稿のリサーチクエスチョンを「選挙制

度改革は政党投票と候補者投票のバランスにどのような影響を与え

たのだろうか」と設定する。そして、上記のような選挙制度改革が

目標としていた効果が現れているのかを検討していきたい。つまり、

選挙制度改革の前後で候補者投票から政党投票の方へ割合がシフト

していることを明らかにし、選挙制度改革の効果を示すことを試み

る。

2. 先行研究

本稿で比較をする 2つの選挙制度の違いとその効果についてはじ

めに概観する。そして、選挙制度改革に関する研究をここでは二つ

紹介する。

まず、選挙制度に関して中選挙区制と小選挙区制の違いを確認し

たい。中選挙区制では、同一選挙区で同一政党から複数の候補者が

立候補することができる。そのため、政党名だけを頼りに選挙を戦

うわけにはいかない。候補者にとって、選挙は政党や政策の争いと

いうより個人同士の争いとならざるを得ない。したがって、投票す

るにあたって候補者個人の要因が重要になってくる。一方で、小選

挙区制では、すべての選挙区で定数が 1 なので、一般的に各政党は

同一選挙区で 1 人しか候補者を出さない。よって、選挙区に個人的

な基盤のない候補者でも、政党の後ろ盾のもと選挙を戦うことがで

きる。つまり、選挙は政党中心に展開されるようになり、有権者は

129

投票する際に候補者自身に対する評価やイメージがなくても、政党

名だけを見て判断することができる。小選挙区制において、政党の

果たす役割が重要であることが分かる。

上記のような選挙制度の違いと、それがもたらす効果について紹

介している研究の 1 つに浅野(2006)がある。その中で、中選挙区

制と小選挙区制での選挙の違いについて、「中選挙区制では、候補者

は個人票に依存し、政党内競争を伴った複数政党制につながる一方

で、小選挙区制では、二大政党制を促進する傾向があり、選挙にお

いて党内競争は存在せず、政党中心の選挙戦が展開されると期待さ

れている」と浅野(2006)は述べている。

次に、選挙制度改革を主たる関心とした鹿毛(1997)と三宅(2001)

の研究を紹介する。

鹿毛(1997)は 1996 年の衆議院議員選挙に注目し、「制度の理解・

認識」という問題について各アクターが選挙制度改革の実現以降に

とった対応の違いを比較し、その対応の違いが選挙制度の作用に及

ぼしている影響を検討している。「各政党は従来以上に『公約』作り

に力を入れ、政党単位の政策論争は全般的に低調ではあったが、制

度的基盤はある程度形成されたといえそうだという点で、選挙運動

における『政党中心』化は一定程度進んだ」と鹿毛(1997)は述べ

ている。しかしながら、あくまで各政党の認識と対応の結果を論理

的にまとめた記述であり、実証的に検討をしていない。

また、三宅(2001)は、候補者投票と政党投票を個別に検討し、

その2つのバランスについてJESⅠとJESⅡというパネル調査デー

タを用いて総合的に分析している。候補者個人要因の効果は 1996

年選挙において、「中選挙区制下の選挙と同様に重要、あるいはそれ

よりもさらに重要である」(三宅 2001, p.38)ことを前職・新人対

立構造で確認している。三宅(2001)は、選挙制度改革の予想に反

して、小選挙区制は候補者個人要因の効果を高める構造があると考

えている。一方、政党要因の効果は「小選挙区制に特有の二大政党

対立選挙区と無競争に近い安全選挙区は、政党投票率」(三宅 2001,

p.84)が確かに高くなることを党派的対立構造で示している。だが、

二大政党対立選挙区と安全選挙区の 2つ以外の対立パターンの選挙

区では、政党投票率が抑えられており、この 2 方向への分化が大き

いと指摘している。そして、「小選挙区制の政党投票率は低いところ

でより低く、高いところでより高い」(三宅 2001, p.69)という点

で中選挙区制と変わりがないことも確認された。さらに、候補者投

票と政党投票のバランスは 1996 年選挙で 1:1 と釣り合っており、

三宅(2001)は『制度均衡』ではないがこのバランスは選挙個別状況

に応じて変化すると述べている。

三宅(2001)は 1990 年代の衆院選しか扱っていないが、2000

年代に入っていよいよ選挙制度の効果が見られるようになったかも

しれない。そこで本稿では、三宅(2001)の分析枞組みを参考にし

130

て、選挙制度改革による候補者個人要因と政党要因の効果の違いを

三宅(2001)とは異なるデータも用いて検討する。つまり、選挙制

度改革による候補者投票と政党投票の割合の変化について示してい

きたい。

3. 仮説

ここまでの内容を踏まえた上で、本稿の仮説を「より最近の選挙

では、候補者投票と政党投票のバランスは政党投票の方向に傾く」

と設定する。この仮説を検証するために、「候補者投票」と「政党投

票」の割合を比較し、そのバランスがどのように変化しているのか

を明らかにしていこう。

この仮説を設定した理由は、1996 年に小選挙区制が導入されたこ

とによって政党要因の重要性がそもそも高まったと思うからだ。前

章で紹介した通り、一般的に小選挙区制では、同一政党から 1 人の

候補者が出る。つまり、すべての候補者は各選挙区における政党の

代表ということになる。よって、候補者の選挙活動は自身の政策を

掲げるというよりも、むしろ政党のマニフェストに準じた主張に重

点をおいて行われることになるだろう。浅野(2000)や鹿毛(1997)

が述べていたように、選挙戦は政党単位で展開され、「候補者同士の

争い」イコール「政党同士の争い」と置き換えることができるだろ

う。したがって、小選挙区制の導入によって政党の役割が大きくな

ったのではないかと考える。

中選挙区制の下では、有権者は政党名で判断するというよりも候

補者自身を評価して投票行動を決めていた。なぜなら、同一選挙区

から同じ政党に所属する候補者が複数出るからだ。そのため、候補

者は独自の政策を掲げて政党に頼らない選挙活動をしていた。一方、

有権者はそれぞれの候補者の主張を踏まえた上で、地元に最も利益

を還元してくれる政策を掲げる候補者を選択することになるだろう。

つまり、有権者が投票先を決定するにおいて、政党だけではなく候

補者自身の評価やイメージを考慮せざるをえない状況であった。

以上のように、中選挙区制下で行われた 1990 年代の選挙と比べ

ると、2000 年代の小選挙区制における選挙の方が政党要因は高まっ

ているだろう。なお、1996 年から小選挙区制は導入されているが、

この年は選挙制度が変わって初めての選挙であったため、政党要因

の効果は1993年の時とあまり変わらないのではないかとも考える。

したがって、「より最近の選挙」と仮説では設定した。

131

4. 分析枞組み

4-1. 使用するデータ

前節で提示した仮説の検証を行うためには、選挙制度改革が行わ

れた 1996 年前後の衆院選を調査しており、かつ候補者投票と政党

投票を定義することができる質問項目に含むサーベイ・データが必

要となる。なぜなら、候補者投票と政党投票を通じて選挙制度改革

の効果を見るためには、制度が変わる前後の調査で変化を見なけれ

ば比較のしようがないからだ。また、仮説の「より最近の選挙」を

検討ために、2000 年以降の衆院選調査も複数必要になる。2000 年

以降の複数回の衆院選における候補者投票と政党投票の変化を見る

ことによって、1990 年代と比較をしながら選挙ごとにどのように変

化をしているかを確認できる。これらの条件を満たすのは JESⅡ・

Ⅲである。1 そして、1993 年から 2005 年の計 4 回の選挙における

衆議院議員選挙事前調査用いる。2

事前調査で明らかになることは、有権者が投票する前の選挙に対

する意見や意図である。今回の分析で使用した 1993 年から 2005

年の事前調査において、約 50%以上もの有権者が選挙前から投票に

行く意思があり、そのうちの約 90%以上が特定の候補者に投票する

つもりであることが分かった。3 この投票に行くつもりである有権

者が、投票先として決めている特定の候補者のことを投票意図候補

者と呼ぶことにする。

4-2. 作業定義

三宅(2001)にならって、投票意図候補者を決めている有権者を

対象に「候補者投票」と「政党投票」に関係する一連の指標を以下

のように作業定義する。

(1) 投票意図候補者の指標

投票意図候補者の指標は、「個人の候補者に投票する小選挙区では

誰に投票するか決めていますか」という質問に対して、有権者が回

答した候補者の所属する政党で定義されている。4 なお、全回答者

が候補者名を挙げているのではない。「あなたは今度の衆議院選挙で

投票に行きますか」という質問に対して「必ず投票するつもり」と

「たぶん投票するつもり」と回答した場合にのみデータが存在する。

(2) 候補者評価の最高値の指標

候補者評価の指標は感情温度計評価を用いて定義されている。そ

れは、候補者について好き・嫌いを 0 度から 100 度で表したもので

ある。ただし、評価を得たのはすべての立候補者ではなく、有権者

が「知っている候補者」で挙げた候補者に限られている。

なお、「知っている候補者」は 1993 年調査では 12 人、1996 年調

査では 8 人、2003・2005 年調査では 6 人まで答えることができた。

132

したがって、感情温度計評価は有権者が答えた候補者の数だけデー

タがある。1 つまたは複数ある感情温度計評価の中で、最も高い温

度を与えられた候補者を候補者評価の最高値として使用する。ただ

し、最高の評価を与えられた候補者が複数いる場合、「知っている候

補者」で挙げた順番の早い方の候補者の評価を採用する。なぜなら、

池田(2010)の言うように、人は「簡単に思い出せたかどうかとい

う想起の容易さ」を手がかりにして物事を簡便的に判断するという

強い傾向があるからだ。このような傾向は選挙でも同様に起こりう

るだろう。つまり、投票先を有権者が簡単に思い出すことのできた

1 番初めの候補者に決定する確率が高いということだ。そして、(1)

の投票意図候補者の時と同様に候補者の所属政党を用いる。

(3) 支持政党の指標

支持政党の指標は「話はかわりますが,今回何党に投票するかは

別にして,ふだんあなたは何党を支持していますか」という質問へ

の回答で定義されている。5

以上の(1)~(3)の指標を用いて「候補者投票」と「政党投票」をそ

れぞれ次のように定義する。

候補者投票とは、候補者評価で最高の評価を与えた候補者に対し

て投票することである。よって、候補者投票変数は(1)の政党と(2)

の政党を組み合わせて作成した。(1)と(2)が一致した場合に「候補者

投票」とした。一方政党投票とは、支持政党の候補者に対して投票

することである。よって、政党投票変数は(1)と(3)を組み合わせて作

成した。(1)と(3)が一致した場合に「政党投票」とした。そして最後

に、2 つの変数を「投票の仕方」としてまとめた。具体的には、候

補者投票の場合に「0」、政党投票の場合に「1」としてダミー変数

を作成した。

[図 1]

図 1 を見ていただきたい。この図は、「候補者投票」と「政党投

票」の変数を「投票の仕方」として作業定義した場合に起こりうる

場合を、ベン図を用いて表したものである。ベン図の左側の円は「候

補者投票」をしたと上記の手順により判定をした有権者、右側の円

は「政党投票」をしたと判定した有権者の集合を表す。この 2 つの

円の重なる網掛けの部分は「候補者投票」をしてかつ「政党投票」

している有権者を示す。つまり、この部分については「候補者投票」

なのか「政党投票」なのかの判定ができないことになる。

そこで本稿では、純粋な候補者投票と政党投票の割合から 2 つの

バランスを検証するため、この網掛け部分に該当する有権者は分析

から排除した。つまり、有権者の支持政党と「知っている候補者」

で最高の評価を与えた候補者の所属政党とが一致するケースは分析

から除いている。その上で、純粋な「候補者投票」と「政党投票」

の比率において、候補者投票から政党投票への時系列的変化を追う

133

ことで、選挙制度の影響を見ることにしたい。

5. 仮説の検証

前節で示した分析枞組みで分析をした結果、表 1・図 2 のような

結果が得られた。

[表 1・図 2]

1993 年から 2005 年を通じて、一貫して候補者投票の割合は政党

投票のそれに上回ることが分かる。つまり、多くの有権者は支持政

党をもとにした政党投票よりも、候補者を重視した候補者投票をし

ている。政党中心の選挙戦になってきたと感じていても、依然とし

て候補者要因が投票に強く表れているということだろう。

次に年ごとに見ていきたい。1993 年と 1996 年の候補者投票の値

を見ていただきたい。この 2 回の選挙では 7 割以上もの人が候補者

投票をしている。上記にもあるように、ここで示されている候補者

投票は、自身の支持政党とは違う政党の候補者へ投票する割合であ

る。したがって、中選挙区制の下ではいかに候補者を重視して投票

していたかが見てとれる。なお、1996 年は小選挙区制の下で選挙が

行われているが、1993 年の結果と変わらない。それどころか、候補

者投票の割合は 1.1%増加している。やはり予想をした通り、小選挙

区制の効果は見られず、中選挙区制の時のなごりがある。

次に、2003 年の候補者投票・政党投票の値を見ていただきたい。

1996年から 2003年にかけて、候補者投票と政党投票の割合は 5.1%

分、政党投票の方へ傾いている。このことから、1990 年代に比べて

政党要因の効果が高まったといえるであろう。つまり、小選挙区制

の導入の効果が尐しではあるが表れている。仮説の通り、2000 年代

の「より最近の選挙」から小選挙区制の効果が見られるようになっ

たといえるだろう。

続いて、2005 年の候補者投票・政党投票の値を見ていただきたい。

1990 年代の選挙と比べると、2~3%分ではあるが候補者投票から

政党投票の方へ比率がシフトしている。したがって、ここからも

2000 年代の選挙で小選挙区制の効果をわずかに確認することがで

きる。ただ、2003 年と比べると 1.7%分だけ候補者投票の割合が増

えてしまっている。選挙を重ねることによって徐々に政党要因の効

果が高まっていくであろうと考えていたが、その効果の変化は直線

的ではないようである。

このような結果になった原因の 1 つとしては、2005 年の選挙が郵

政民営化を政策争点とする郵政選挙であったからだと考えられる。

郵政民営化について、特に自民党では党内で一貫した方針が定まっ

ておらず、党内分裂が起こっていた。これを受けて、支持政党に基

づいて投票をしようとしていた有権者は、郵政民営化に賛成・反対

134

かという基準で候補者を選択したので、候補者要因の効果が 2003

年に比べて尐し高まったのではないかと推測する。

最後に、表 1 の一番右側の「候補者∩政党」の値を見ていただき

たい。これは、図 1 の網掛け部分にあたる、政党投票もしてかつ政

党投票もしている有権者の数を表す。数値を見て分かる通り、どち

らもしている有権者は圧倒的に多い。候補者投票と政党投票のケー

スの総数(N)よりも多くなっている。このことから、有権者はふ

だん支持している政党に所属する候補者に投票しようとする可能性

が高いということがいえる。

6. おわりに

本稿の分析結果を踏まえて仮説を再検討すると 2つのことが明ら

かになった。

1 つめに、1990 年代の 2 つの選挙と 2000 年代の 2 つの選挙の比

較を通じて、選挙制度の効果は直線的ではないが尐しずつ高まって

きていることが分かった。1996 年に導入された小選挙区制ではある

が、その効果は仮説の通り「より最近の選挙」である 2000 年代か

ら表れ始めている。分析結果(表 1・図 2)を振り返ると、1996 年

から 2003 年には 5.1%、2005 年には 3.4%だけ候補者投票から政党

投票の方へ割合がスライドしている。このことから、小選挙区制導

入の際に目標としていた政党要因の効果が、2000 年代という「より

最近の選挙」で尐し高まってきているということが分かった。ただ、

2005 年には、予想に反して 1.4%分候補者投票の方へ比率が戻って

しまった。このことから、1990 年代と比べると政党要因の効果はわ

ずかに上昇しているが、比率の変化の仕方が直線的ではないといえ

る。

ただ、純粋な「候補者投票」から「政党投票」の比率において、

大幅な変化を見ることができなかった。1990 年代と 2000 年代の差

は最大 5.1%であり、割合の変化としては尐し小さかった。よって、

選挙制度改革で期待されていた小選挙区制が、必ずしも有権者の中

でしっかりと定着したとはいえないだろう。

以上のことを踏まえて 2 つめに、選挙制度改革の目標が達成され

るまでにはまだ長い時間がかかるのではないだろうかと考える。分

析では1993年から2005年の計4回の選挙を対象にして検討してき

た。1996年に小選挙区制が導入されておよそ 10年経つというのに、

制度の効果をはっきりと確認することができなかった。制度の定着

と有権者の投票行動の変化を見るためには、何十年とかけて小選挙

区制の下での選挙を何回もする必要があるようだ。つまり、小選挙

区制が期待する政党要因の効果の上昇には、有権者の同じ制度下で

の複数回にわたる選挙経験が必要になってくると考えられる。

135

図 1 投票の仕方

表1 候補者投票と政党投票の割合

候補者投票 政党投票 N 候補者∩政党

% N % N % N N

1993 年 70.7 203 29.3 84 100 287 557

1996 年 71.8 290 28.2 114 100 404 522

2003 年 66.7 279 33.3 139 100 418 578

2005 年 68.4 234 31.6 108 100 342 567

出所:JESⅡ・Ⅲ

図2 候補者投票と政党投票の割合

68.4

31.6

70.7

71.866.7

33.328.2

29.3

20

40

60

80

1993 1996 2003 2005 (年)

(%)

候補者投票

政党投票

候補者投票 政党投票

136

(1) ここで使用したデータは JESⅡ・JESⅢである。JESⅡは「投

票行動の全国的・時系列的調査研究」で JESⅡ研究会が実施さ

れた世論調査である。JESⅢは、平成 14~18 年度文部科学省

研究費特別推進研究「21 世紀初頭の投票行動の全国的・時系列

的調査研究」に基づく「JESⅢ研究プロジェクト」(参加者・池

田謙一:東京大学教授、小林良彰:慶應義塾大学教授、平野

浩:学習院大学教授)が行った研究結果である。それをレヴァ

イアサン・データバンクより同志社大学が代表して利用申請

したものを西澤由隆先生のご指導と便宜の下、利用させてい

ただいた。この場を借りて、それぞれのデータを公開・寄託

され、利用できるようにしてくださった先生方に感謝いたし

ます。

(2) ただし、2000 年衆院選調査はデータ不足のため分析から外し

た。

(3) これは、全サンプルの約 45%にあたる。

(4) ここで候補者の所属政党を用いたのは、選挙区ごとに候補者

名が違うので定義が煩雑になってしまうからである。

(5) 無所属とその他の政党は 1 つにまとめて分析をした。

〈補遺〉

・使用した質問文(2003 年衆議院選挙事前調査のみ掲載)

Q1 さっそくですが,今回の衆議院選挙で,この選挙区から立候

補の届け出をすませた方々のことについてお伺いします。

(1)この中で,どなたのお名前をご存知ですか。

(2)その方をどの程度ご存知ですか。

(3)温度計の絵があります。この候補者を好きでも嫌いでもなけれ

ば 50 度、もし好きでしたらその強さに応じて 50 度から 100

度の間の数字を答えてください。もし嫌いでしたら、やはり

その強さに応じて 0 度から 50 度のどこかの数字を答えてくだ

さい。

→候補者の名前が出なくなるまで上記(1)~(3)を繰り返す。(最大

6 人まで答えることができる)

Q8 話はかわりますが,今回何党に投票するかは別にして,ふだ

んあなたは何党を支持していますか。

1 自民党 2 民主党 3 公明党 4 社民党 5 共産党 6 保守新党

8 その他政党 9 支持政党なし 10 わからない 11 答えない

Q12 あなたは今度の衆議院選挙で投票に行きますか。

1 必ず投票するつもり 2 たぶん投票するつもり

3 たぶん棄権する 4 棄権する 5 決めていない,わからない

6 答えない

SQ1 個人の候補者に投票する小選挙区では誰に投票するか決めて

いますか。

1 候補者を決めている 2 政党は決めている

137

3 政党も候補者も決めていない 4 わからない 5 答えない

SQ2 それは誰ですか。

〈参考文献〉

・浅野正彦 2006. 『市民社会における制度改革―選挙制度と候

補者リクルート』 慶應義塾大学出版会.

・池田謙一・唐沢穣・工藤恵理子・村本由喜子・江草貞治 2010.

『社会心理学』 有斐閣.

・鹿毛利枝子 「制度認識と政党システム再編」 大嶽秀夫編

1997. 『政界再編の研究:新選挙制度による総選挙』 有斐

閣.

・三宅一郎 2001. 『選挙制度変革と投票行動』 木鐸社

多岐亡羊 Vol.15 2011 年度西澤ゼミ学生論文集

2012 年 3 月 20 日 初版

著 者 2011 年度西澤ゼミ生

発行者 西澤由隆

発行所 同志社大学法学部・政治学科 西澤由隆研究室

〒602-8580 京都市上京区今出川通東入る

TEL: 075-251-3597

E-mail: [email protected]

HP: http://ynishiza.doshisha.ac.jp/

製本所 ナカバヤシ株式会社

Copyright © 2012 Nishizawa Seminar Printed in Japan